Beta
Logo of the podcast Cardionerds: A Cardiology Podcast

Cardionerds: A Cardiology Podcast (CardioNerds)

Explore every episode of Cardionerds: A Cardiology Podcast

Dive into the complete episode list for Cardionerds: A Cardiology Podcast. Each episode is cataloged with detailed descriptions, making it easy to find and explore specific topics. Keep track of all episodes from your favorite podcast and never miss a moment of insightful content.

Rows per page:

1–50 of 417

Pub. DateTitleDuration
07 Jan 2022171. Narratives in Cardiology: Innovation, Excellence and Leadership in Interventional Cardiology with Dr. Samir Kapadia – Ohio Chapter00:43:30
CardioNerds (Amit Goyal and Daniel Ambinder), Dr. Zarina Sharalaya (Interventional Cardiology Fellow at the Cleveland Clinic), and Dr. Simrat Kaur (General Cardiology Fellow at the Cleveland Clinic) join Dr. Samir Kapadia, the Chair of the Robert and Suzanne Tomsich Department of Cardiovascular Medicine, Sydell and Arnold Miller Family Heart, Vascular & Thoracic Institute at Cleveland Clinic. They discuss future advancements in the field of structural interventional cardiology. Dr. Kapadia sheds light on his journey starting as an international medical graduate from India and speaks about his mentors that helped shape his career and his life. We later delve into several advancements in the field of structural and interventional cardiology, along with the amalgamation of different sub-specialities with intervention such as heart failure and critical care cardiology. We also discuss the measures being taken to reduce the occupational hazards associated with interventional cardiology and how to make this field more appealing to women in cardiology. Special message by Ohio ACC State Chapter Governor, Dr. Kanny Grewal. The PA-ACC & CardioNerds Narratives in Cardiology is a multimedia educational series jointly developed by the Pennsylvania Chapter ACC, the ACC Fellows in Training Section, and the CardioNerds Platform with the goal to promote diversity, equity, and inclusion in cardiology. In this series, we host inspiring faculty and fellows from various ACC chapters to discuss their areas of expertise and their individual narratives. Join us for these captivating conversations as we celebrate our differences and share our joy for practicing cardiovascular medicine. We thank our project mentors Dr. Katie Berlacher and Dr. Nosheen Reza. Video Version • Notes • References • Production Team Claim free CME just for enjoying this episode! There are no relevant disclosures for this episode. The PA-ACC & CardioNerds Narratives in Cardiology PageCardioNerds Episode PageCardioNerds AcademyCardionerds Healy Honor Roll CardioNerds Journal ClubSubscribe to The Heartbeat Newsletter!Check out CardioNerds SWAG!Become a CardioNerds Patron! Tweetorial on Innovation, Excellence and Leadership in Interventional Cardiology with by Dr. Gurleen Kaur https://twitter.com/gurleen_kaur96/status/1484205728663576590?s=21 Video version - Innovation, Excellence and Leadership in Interventional Cardiology with Dr. Samir Kapadia https://youtu.be/BfqnRkaVGkk Quotables - Innovation, Excellence and Leadership in Interventional Cardiology with Dr. Samir Kapadia “A very important thing for all international medical graduates and for everybody, for that matter - it is important to recognize that the opportunities are what you perceive and not what others perceive.”Dr. Samir Kapadia Show notes - Innovation, Excellence and Leadership in Interventional Cardiology with Dr. Samir Kapadia How do international medical graduates contribute to the work force in medicine across the United States of America? International medical graduates account for 25% of the physician work force, with over 85% being involved in direct patient care.IMGs are usually accomplished, consummate and highly motivated physicians who often have to overcome challenges such as language proficiency, acculturation and difficulties with obtaining a visa status in the United States.IMGs also help fill gaps in health care by working in geographical areas that are otherwise not desirable by US or Canadian medical graduates.IMGs contribute to diversity of the field which provides a richer training environment, improved access to health care for underrepresented minorities, as well as better patient outcomes. What are key qualities of a good mentor? A good mentor is responsible for enhancing the education of his or her mentees along with motivating them to challenge their limits.Qualities of a good mentor extend beyond mere mentorship to s...
11 Jan 2022172. CCC: The Hemodynamic Evaluation of Cardiogenic Shock with Dr. Nosheen Reza00:42:53
The hemodynamic evaluation of cardiogenic shock obtained via a Swan-Ganz catheter plays an essential role in the characterization of cardiogenic shock patients. Join Dr. Nosheen Reza, (Assistant Professor of Medicine and Advanced Heart Failure and Transplant cardiologist at the Hospital of the University of Pennsylvania), episode fellow lead Dr. Brian McCauley (Interventional and Critical Care Fellow at the Hospital of the University of Pennsylvania), Dr. Mark Belkin (Cardiac Critical Care Series Co-Chair and AHFT fellow at University of Chicago), and CardioNerds Co-Founders, Amit Goyal and Dan Ambinder, for this tour through the heart aboard the Swan-Ganz catheter. In this episode, we evaluate three separate admissions for a single patient to highlight pearls regarding waveform assessment, evaluating cardiac output, phenotyping hemodynamic profiles, targeted therapies based on hemodynamics and so much more. Episode introduction and audio editing by Dr. Gurleen Kaur (Director of the CardioNerds Internship). Claim free CME for enjoying this episode! Disclosures: None Pearls • Notes • References • Guest Profiles • Production Team CardioNerds Cardiac Critical Care PageCardioNerds Episode PageCardioNerds AcademyCardionerds Healy Honor Roll CardioNerds Journal ClubSubscribe to The Heartbeat Newsletter!Check out CardioNerds SWAG!Become a CardioNerds Patron! Pearls and Quotes - Hemodynamic Evaluation of Cardiogenic Shock Swan-Ganz catheters are not dead #ReviveTheSwan!  They remain a useful tool to characterize cardiac patients & to help direct therapy, especially in Cardiogenic Shock.When looking at Swan-Ganz catheter data, it is important to always interpret your own tracings, to know what values are acquired directly, and which values are derived.It is important to understand the strengths and weakness of hemodynamic characterization by Swan-Ganz cathetersAdvanced metrics such as cardiac power output, pulmonary artery pulsatility index, and aortic pulsatility index are extremely useful in further phenotyping patients as well as guiding mechanical support platforms“The data will be wrong if the preparation is not right” Show notes - Hemodynamic Evaluation of Cardiogenic Shock 1. Swan-Ganz catheters are a useful tool to characterize cardiac patients and to direct therapy.  With the ESCAPE trial in 2004, Swan-Ganz catheter utilization dropped drastically outside transplant centers across the United States (2). While the ESCAPE trial did demonstrate the possibility of harm when using a Swan-Ganz catheter, many of the truly ill cardiac patients we care for would have been excluded from the trial. For instance, patients on dobutamine at doses above 3 µg/kg/min or any dose of milrinone during the hospitalization were excluded from the trial.This is a classic example of “throwing the baby out with the bath water.”In a recent large, multicenter cardiogenic shock registry, complete hemodynamic assessment using pulmonary artery catheters prior to MCS is associated with lower in-hospital mortality compared with incomplete or no assessment (3). 2. When looking at Swan-Ganz catheter data, it is important to always interpret your own tracings, to know what values are acquired directly, and which values are derived. Incomplete or incorrect data can lead to mischaracterization of our patients. Therefore, it is essential to review all of the tracings, calculations, and data acquired for each individual patient before any clinical adjustments are made (1). An incomplete pulmonary capillary wedge tracing is an example from clinical practice (causing the PCWP, and therefore the left-sided filling pressures to be overestimated).  It is equally important to know the limitations of cardiac output equations, and that no one measurement is perfect.Foibles of the Fick equation include assumed rather than measured oxygen consumption and variations in hemoglobin concentration. Traditionally,
17 Jan 2022173. Case Report: A Block and a Leak Lead to Shock – Weill Cornell01:14:01
CardioNerds (Amit Goyal and Daniel Ambinder) join Dr. Jaya Kanduri, Dr. Dan Lu, and Dr. Joe Wang from Weill Cornell Cardiology for Levain cookies in Central Park. The ECPR is provided by Dr. Harsimran Singh (Cardiology Program Director and Interventional Cardiologist with expertise in ACHD). Episode introduction by CardioNerds Clinical Trialist Dr. Jeremy Brooksbank. We discuss a case of a 24-year-old female with a history of unicuspid aortic valve with associated aortopathy status post mechanical aortic valve replacement and Bentall procedure at age 16 presents with acute onset substernal chest pain and shortness of breath. She was found to have mechanical aortic valve obstruction and severe aortic regurgitation resulting in cardiogenic shock. Unfortunately, the shock quickly progressed to refractory cardiac arrest requiring mechanical support with VA-ECMO before valve debridement was performed in the operating room. The differential for mechanical prosthetic valve stenosis includes pannus, thrombus, or vegetation. She was eventually found to have thrombus obstructing the outflow tract and holding the mechanical leaflets open leading to torrential regurgitation. She underwent successful surgical debridement. We discuss unicuspid aortic valve and associated aortopathy, surgical considerations regarding AVR, diagnosis and management of prosthetic valve dysfunction, approach to cardiogenic shock and considerations around activating and managing VA-ECMO. With this episode, the CardioNerds family warmly welcomes Weill Cornell Cardiology to the CardioNerds Healy Honor Roll. The CardioNerds Healy Honor Roll programs support and foster the the CardioNerds spirit and mission of democratizing cardiovascular education. Healy Honor Roll programs nominate fellows from their program who are highly motivated and are passionate about medical education. The Weill Cornell fellowship program director, Dr. Harsimran Singh has nominated Dr. Jaya Kanduri for this position. Claim free CME just for enjoying this episode!  Disclosures: NoneJump to: Pearls - Notes - References CardioNerds Case Reports PageCardioNerds Episode PageCardioNerds AcademyCardionerds Healy Honor Roll CardioNerds Journal ClubSubscribe to The Heartbeat Newsletter!Check out CardioNerds SWAG!Become a CardioNerds Patron! Case Media ECGCXREchoRHC PSL AP3 Color LHC - LCA LHC - LCA RCA Aortogram TEE TEE 2 Episode Teaching Pearls - Mechanical Valve Thrombosis (1) Unicuspid aortic valves present with aortic stenosis earlier in life. There can be concurrent aortic regurgitation and, like bicuspid aortic valves, unicuspids can be associated with aortopathy as well as other congenital anomalies. (2) Prosthetic valve stenosis is assessed with different echocardiographic parameters than what we use for native valves. The differential for mechanical valve stenosis includes pannus, thrombus, or vegetation. Patient prosthesis mismatch may also lead to elevated gradients. (3) VA-ECMO provides robust flow in the setting of cardiogenic shock as well as gas exchange. While this flow may improve end-organ perfusion, it also increases left ventricular afterload, thereby potentially worsening LV ischemia and impeding LV recovery. Elevated afterload may also decrease innate contractility and prevent aortic valve leaflets from opening. Therefore, if a patient with a mechanical valve is on VA-ECMO, ensuring valve opening to prevent valve (or ventricular) thrombosis is paramount. (4) Venting is sometimes necessary to decrease the left ventricular end diastolic pressure from the high afterload imposed by VA-ECMO. A microaxial temporary LVAD (example – Impella device) directly unloads the left ventricle, but cannot be used in the setting of a mechanical aortic valve. TandemHeart is also a consideration (inflow cannula placed across the interatrial septum in the left atrium) to unload the LV,
21 Jan 2022174. Cardio-Obstetrics: Black Maternal Health with Dr. Rachel Bond01:11:45
CardioNerds (Amit Goyal), Dr. Natalie Stokes (Cardiology Fellow at UPMC and Co-Chair of the Cardionerds Cardio-Ob series), fellow lead Dr. Victoria Thomas (Cardionerds Ambassador, Vanderbilt University Medical Center), join Dr. Rachel Bond (Women's Heart Health Systems Director at Dignity Health, Arizona) for a cardio-obstetrics discussion about Black maternal health. Episode introduction by CardioNerds Clinical Trialist Dr. Chistabel Nyange. Audio editing by CardioNerds Academy Intern, Christian Faaborg-Andersen. This episode was developed in collaboration with the Association of Black Cardiologists. ABC is a 501(c)3 nonprofit organization whose mission is to promote the prevention and treatment of cardiovascular disease, including stroke, in Black persons and other minority populations, and to achieve health equity for all through the elimination of disparities. Learn more at https://abcardio.org/. Notes • References • Guest Profiles • Production Team CardioNerds Cardio-Obstetrics Series PageCardioNerds Episode PageCardioNerds AcademyCardionerds Healy Honor Roll CardioNerds Journal ClubSubscribe to The Heartbeat Newsletter!Check out CardioNerds SWAG!Become a CardioNerds Patron! Show notes 1. Why does Black Maternal Health need to be deliberately highlighted episode on CardioNerds? Black women are three-four times more likely to die during their pregnancy. The deaths are primarily tied to cardiomyopathy and cardiovascular conditions such as coronary artery disease, pulmonary hypertension, chronic hypertension, preeclampsia, and eclampsia.63-68% of this cardiovascular mortality is preventable depending on one’s racial identity.  As CardioNerds, we must educate ourselves on why this occurs and identifying diseases that may place patients at increased risk.Studies have shown the Black maternal mortality crisis exist irrespective of one’s education or socioeconomic status.We must recognize and admit that some patients are being treated differently because of their race and ethnicity alone. 2.     When we consider or acknowledge a patient’s race, what should CardioNerds think about? Race is an important factor to think about, but we must remember that it is an imperfect variable. We should not focus on biology or genetic make-up. We should think about social determinants of health. 60% of the time social and personal aspects dictate one’s health.Unconscious biases and structural racism are likely playing a major role in race-based health inequities. 3.     What are other vulnerable groups that have increased mortality rates related to cardioobstetric care? Native American women have similar maternal mortality rates to Black populations.Women who are veterans, live in rural communities, and/or are currently incarcerated have increased risk of mortality 4.     What are some of the social determinants of health that should be considered for these patients? Food deserts or having poor access to nutrient rich/quality foods make these vulnerable patients have increased risk factors for high cholesterol, high blood pressure, obesity, and diabetes which increase the risk for pregnancy complications and infertility.The above vulnerable populations can have less access to higher levels of care for high-risk pregnancies. 5.     What are some of the preventable causes of maternal mortality? Clinicians should actively listen to their patients' concerns. There have been several media stories in the news and on CardioNerds episodes where women’s concerns were not acknowledged or taken seriously.Preconception counseling is important to provide to all patients. 50% of women have one risk factor for cardiovascular disease when entering pregnancy. We should have discussions with patients regarding their lifestyles, with an emphasis on exercise and diet. 6.     What are some of the psychosocial or health related differences we see in black mothers when compared to other...
27 Jan 2022175. ACHD: Single Ventricle Circulation and Fontan Palliation with Dr. Yuli Kim00:53:07
CardioNerds (Amit Goyal and Daniel Ambinder), ACHD series co-chair Dr. Daniel Clark (Vanderbilt University), and ACHD FIT lead Dr. Danielle Massarella (Toronto University Health Network) join ACHD expert Dr. Yuli Kim (Associated Professor of Medicine & Pediatrics at the University of Pennsylvania), to discuss single ventricular heart disease and Fontan palliation. They cover the varied anatomical conditions that can require 3-step surgical palliation culminating in the Fontan circulation, which is characterized by passive pulmonary blood flow, high venous pressures, and low cardiac output. Audio editing by Dr. Gurleen Kaur (Director of the CardioNerds Internship and CardioNerds Academy Fellow).  The CardioNerds Adult Congenital Heart Disease (ACHD) series provides a comprehensive curriculum to dive deep into the labyrinthine world of congenital heart disease with the aim of empowering every CardioNerd to help improve the lives of people living with congenital heart disease. This series is multi-institutional collaborative project made possible by contributions of stellar fellow leads and expert faculty from several programs, led by series co-chairs, Dr. Josh Saef, Dr. Agnes Koczo, and Dr. Dan Clark. The CardioNerds Adult Congenital Heart Disease Series is developed in collaboration with the Adult Congenital Heart Association, The CHiP Network, and Heart University. See more Claim free CME for enjoying this episode! Disclosures: None Pearls • Notes • References • Guest Profiles • Production Team CardioNerds Adult Congenital Heart Disease PageCardioNerds Episode PageCardioNerds AcademyCardionerds Healy Honor Roll CardioNerds Journal ClubSubscribe to The Heartbeat Newsletter!Check out CardioNerds SWAG!Become a CardioNerds Patron! Pearls There are various forms of unpalliated ‘single ventricle’ congenital heart disease. The three main hemodynamic issues that need to be addressed in any form are unbalanced flow, pulmonary over-circulation, and blood mixing.  The Fontan palliation is a series of operations for congenital heart disease patients in whom biventricular repair is not feasible.  In the completed Fontan circulation, systemic venous blood is surgically routed directly to the lungs, effectively bypassing the heart, and creating passive pulmonary blood flow.  The hallmarks of the Fontan circulation (and Fontan failure) are elevated central venous pressure and low cardiac output.  Patients with Fontan circulation may experience significant morbidity in the long term from both cardiac and non-cardiac sequelae, and require lifelong specialist care.  Show notes 1. Why do some patients require Fontan palliation?  Many different types of anatomies may ultimately require single ventricular palliation via the Fontan procedure due to inadequate biventricular function to support both pulmonary and systemic circulations. Some examples include Tricuspid Atresia (hypoplastic RV), Double Inlet Left Ventricle (DILV; hypoplastic RV), Hypoplastic Left Heart Syndrome (HLHS; hypoplastic LV), and atrioventricular septal defects (AVSD; either RV or LV may be inadequate based on “commitment” of the common AV valve). The Fontan procedure was first described in 1971; at this time, mortality of single ventricular patients exceeded 90% in the first year of life.  2. What are the stages of Fontan palliation?  Effective pulmonary blood flow/balancing flow to the pulmonary and systemic circulations: for many conditions, this involves retrograde pulmonary blood flow from a systemic -> PA shunt (i.e. Blalock-Taussig-Thomas “BTT” shunt in which the subclavian artery is turned down and anastomosed to the pulmonary artery). In infants, the pulmonary vascular resistance (PVR) is high perinatally and gradually lowers over the first 3 months of life to adult levels with exposure to the atmosphere’s natural pulmonary vasodilator: oxygen. Thus, in the first 3 months of life babies have an intri...
04 Feb 2022176. Narratives in Cardiology: Interventional Cardiology, Cardioobstetrics, & Work Life Integration with Dr. Ki Park – Florida Chapter00:48:23
CardioNerd (Amit Goyal), Dr. Zarina Sharalaya (Interventional cardiology fellow at the Cleveland Clinic), Dr. Ashley Mohadjer (Interventional cardiology fellow, Vanderbuilt Heart and Vascular Institute), and Dr. Laurie Mbuntum (Cardiology fellow, UTSW) join Dr. Ki Park (Associate professor of medicine and an interventional cardiologist at the University of Florida and Malcom Randall VA Medical Center in Gainesville, FL.) for a a well-rounded discussion on all things ‘Women-in-Cardiology' #WIC . Dr. Ki Park discusses how she nurtured her interest in interventional cardiology, and further shares her thoughts and passion for cardio-obsetrics. She shares her advice for trainees thinking about interventional or cardioobetrics and anecdotes from her training as a successful woman in the field. We discuss the need for education on pregnancy outcomes and long-term cardiovascular risk, ideas to lower maternal mortality, how to start a women’s cardiovascular clinic, and her thoughts on how the field may look in the future. Special message by Florida ACC State Chapter Governor, Dr. David Perloff. Episode introduction and audio editing by CardioNerds Academy Intern, Shivani Reddy. The PA-ACC & CardioNerds Narratives in Cardiology is a multimedia educational series jointly developed by the Pennsylvania Chapter ACC, the ACC Fellows in Training Section, and the CardioNerds Platform with the goal to promote diversity, equity, and inclusion in cardiology. In this series, we host inspiring faculty and fellows from various ACC chapters to discuss their areas of expertise and their individual narratives. Join us for these captivating conversations as we celebrate our differences and share our joy for practicing cardiovascular medicine. We thank our project mentors Dr. Katie Berlacher and Dr. Nosheen Reza. Video Version • Notes • Production Team Claim free CME just for enjoying this episode! There are no relevant disclosures for this episode. The PA-ACC & CardioNerds Narratives in Cardiology PageCardioNerds Episode PageCardioNerds AcademyCardionerds Healy Honor Roll CardioNerds Journal ClubSubscribe to The Heartbeat Newsletter!Check out CardioNerds SWAG!Become a CardioNerds Patron! Video version - Interventional Cardiology, Cardioobstetrics, & Work Life Integration with Dr. Ki Park https://youtu.be/_oYUc-_sdfU Tweetorial - Interventional Cardiology, Cardioobstetrics, & Work Life Integration with Dr. Ki Park https://twitter.com/gurleen_kaur96/status/1495921275545563136?s=21 Quotables - Interventional Cardiology, Cardioobstetrics, & Work Life Integration with Dr. Ki Park “I like the work life integration as opposed to work life balance. Balance just implies that you always have everything aligned perfectly at all times and that is just not doable.”Dr. Ki Park Show notes - Interventional Cardiology, Cardioobstetrics, & Work Life Integration with Dr. Ki Park Why is screening for OB-GYN history for cardiovascular risk is important, and who should be responsible? Pregnancy is nature’s stress test and in some women can unmask someone’s predisposition to cardiac diseaseYearly screening for diabetes, hypertension, dyslipidemiaBig interdisciplinary effort in attempt to try to capture all women at risk, as many will not present with manifestation of disease initially How did you nurture your interest in cardioobsetrics? In interventional cardiology? Meetings and societiesConnect with those who work in the field, social mediaRegarding interventional cardiology – having interest in procedures, do as many cases “hands on” as possible, learning from mistakes What advise do you have to achieve work and life balance? It's important to understand the various occupational hazards of radiation exposure which include but are not limited to brain tumors, cataracts, thyroid disease, cardiovascular diseases, musculosketal problems and reproductive side effects. Have grace,
07 Feb 2022177. CCC: Cardiac Arrest, E-CPR, & Post-Arrest Care with Dr. Jason Bartos01:05:29
Approximately 350,000 adults per year in the US experienced out-of-hospital cardiac arrest (OHCA). Only about 10% of such patients survive their initial hospitalization. The key drivers of successful resuscitation from OHCA are bystander cardiopulmonary resuscitation (CPR) and public use of an automated external defibrillator (AED). Survival rates from OHCA vary dramatically between US regions. For instance, the extracorporeal CPR (eCPR) program at the University of Minnesota has over a 40% survival rate in patients with OHCA and refractory ventricular fibrillation (VF) based on data published in the ARREST trial. In this episode, we are joined by experts from the University of Minnesota, including Dr. Jason Bartos (Interventional and Critical Care Faculty) and Dr. Julie Power (Chief Fellow at University of Minnesota and CardioNerds Academy Fellow), along with Dr. Yoav Karpenshif (Co-Chair Critical Care Series, University of Pennsylvania) and CardioNerds Co-Founders (Amit Goyal and Dan Ambinder) to discuss cardiac arrest, E-CPR, & post-arrest care. This includes targeted temperature management, coronary angiography and revascularization, as well as the growing field of eCPR and VA ECMO.  Episode introduction by CardioNerds Clinical Trialist Dr. Jason Feinman. Audio editing by CardioNerds Academy Intern, Shivani Reddy. The CardioNerds Cardiac Critical Care Series is a multi-institutional collaboration made possible by contributions of stellar fellow leads and expert faculty from several programs, led by series co-chairs, Dr. Mark Belkin, Dr. Eunice Dugan, Dr. Karan Desai, and Dr. Yoav Karpenshif. Claim free CME for enjoying this episode! Disclosures: None Pearls • Notes • References • Guest Profiles • Production Team CardioNerds Cardiac Critical Care PageCardioNerds Episode PageCardioNerds AcademyCardionerds Healy Honor Roll CardioNerds Journal ClubSubscribe to The Heartbeat Newsletter!Check out CardioNerds SWAG!Become a CardioNerds Patron! Abbreviations - Cardiac Arrest, E-CPR, & Post-Arrest Care eCPR- extracorporeal cardiopulmonary resuscitation VA ECMO- veno-arterial extracorporeal membrane oxygenation VT/VF- ventricular tachycardia/ventricular fibrillation ACLS- advanced cardiovascular life support ROSC- return of spontaneous circulation- OHCA- out-of-hospital cardiac arrest IHCA- in-hospital cardiac arrest TTM- targeted temperature management Pearls and Quotes - Cardiac Arrest, E-CPR, & Post-Arrest Care The ARREST trial showed early VA ECMO-facilitated resuscitation for patients with OHCA and refractory VF significantly improved survival to hospital discharge when compared to standard ACLS treatment.Coronary artery disease is common in the setting of cardiac arrest, with up to 96% of patients with STEMI on post resuscitation EKG and up to 85% of refractory out-of-hospital VT/VF arrests.Guidelines recommend emergent coronary angiography for patients with ST-segment elevation on the post-ROSC ECG.The role of timing of revascularization after ROSC in patients without STEMI or shock is unknown.The role of coronary angiography in cardiac arrest with nonshockable rhythms is also unclear.The current AHA guidelines recommend initiation of targeted temperature management between 32°C and 36°C for at least 24 hours for all patients who do not follow commands after ROSC in both OHCA and IHCA. Show notes - Cardiac Arrest, E-CPR, & Post-Arrest Care 1. What are early post arrest management considerations? The key drivers of successful resuscitations from OHCA: CPR and public use of AEDs in the field. After initial stabilization, care of the critically ill post-arrest patient hinges on hemodynamic support, mechanical ventilation, temperature management, attending to adverse sequelae of arrest, and diagnosis and treatment of underlying causes of arrest. Coronary artery disease is common in the setting of VT/VF cardiac arrest,
10 Feb 2022178. Case Report: Occam’s Razor or Hickam’s Dictum? Cardiogenic Shock With Severe Biventricular Heart Failure – Northwestern University00:57:43
CardioNerds (Amit Goyal and Daniel Ambinder) join Dr. Loie Farina (Northwestern University CardioNerds Ambassador), Dr. Josh Cheema, and Dr. Graham Peigh from Northwestern University for drinks along the shores of Lake Michigan at North Avenue Beach. They discuss a case of a 52-year-old woman with limited cutaneous systemic sclerosis who presents with progressive symptoms of heart failure and is found to have a severe, non-ischemic cardiomyopathy. The etiology of her cardiomyopathy is not clear until her untimely death. She is ultimately diagnosed with cardiac AL amyloidosis with isolated vascular involvement a real occam’s razor or hickam’s dictum conundrum. We discuss the work-up and management of her condition including a detailed discussion of the differential diagnosis, the underlying features of systemic sclerosis with cardiac involvement as well as cardiac amyloidosis, the role of a shock team in managing cardiogenic shock, and how to identify those with advanced or stage D heart failure. Advanced heart failure expert Dr. Yasmin Raza (Northwestern University) provides the ECPR segment. Episode introduction by CardioNerds Clinical Trialist Dr. Liane Arcinas. Audio editing by CardioNerds Academy Intern, Christian Faaborg-Andersen. Claim free CME just for enjoying this episode!  Disclosures: NoneJump to: Pearls - Notes - References CardioNerds Case Reports PageCardioNerds Episode PageCardioNerds AcademyCardionerds Healy Honor Roll CardioNerds Journal ClubSubscribe to The Heartbeat Newsletter!Check out CardioNerds SWAG!Become a CardioNerds Patron! Case Summary - Occam’s Razor or Hickam’s Dictum? This is a case of a 52-year-old woman with limited cutaneous systemic sclerosis who presented with progressive dyspnea on exertion and weight loss over the course of 1 year. Her initial work-up was notable for abnormal PFTs and finding of interstitial pneumonia on high-resolution CT, an ECG with frequent PVCs and normal voltage, a transthoracic echocardiogram with a mildly reduced ejection fraction of 40%, and a right/left heart catheterization with normal coronary arteries, filling pressures, and cardiac output. Scleroderma-related cardiac involvement is suspected. She is placed on GDMT, but her condition worsens over the next several months, and repeat echocardiogram shows severely reduced biventricular function, reduced LV global longitudinal strain (GLS) with apical preservation of strain, severely reduced mitral annular tissue Doppler velocities, and a normal left ventricular wall thickness. Scleroderma-related cardiac involvement remains highest on the differential, but because of some findings on the echo that are concerning for cardiac amyloidosis, an endomyocardial biopsy was obtained. It showed vascular amyloid deposition without interstitial involvement. The diagnosis of cardiac amyloid was discussed but deemed unlikely due to lack of interstitial involvement. However, a serologic work-up soon revealed a monoclonal serum lambda light chain and a follow-up bone marrow biopsy showed 20% plasma cells. She was discharged with very near-term follow-up in oncology clinic with a presumptive diagnosis of AL amyloidosis, but she unfortunately returned in shock and suffered a cardiac arrest. She initially survived and underwent emergent veno-arterial extracorporeal membrane oxygenation (VA ECMO) cannulation with subsequent left ventricular assist device placement (LVAD). However, she passed away due to post-operative hemorrhage. Autopsy was consistent with a final diagnosis of cardiac AL amyloidosis with isolated vascular involvement.  Case Media - Occam’s Razor or Hickam’s Dictum? EKG CXR TTE Pathology CMR Episode Teaching -Occam’s Razor or Hickam’s Dictum? Pearls Scleroderma causes repeated focal ischemia-reperfusion injuries which result in patchy myocardial fibrosis. Cardiac involvement in scleroderma is frequent but often not clinicall...
16 Feb 2022179. Lipids: Polyunsaturated Fatty Acids, Omega-3 Fatty Acids, Eicosapentaenoic acid – Mechanisms of Action with Dr. Ty Gluckman00:51:13
CardioNerds Tommy Das (Program Director of the CardioNerds Academy and cardiology fellow at Cleveland Clinic), Rick Ferraro (Director of CardioNerds Journal Club and cardiology fellow at the Johns Hopkins Hospital), and CardioNerds Healy Honor Roll Ambassador Dr. Justice Oranefo (UConn cardiology fellow) discuss omega-3 fatty acids acid with Dr. Ty Gluckman, preventive cardiologist and medical director of the Center for Cardiovascular Analytics, Research, and Data Science (CARDS) at the Providence St. Joseph Heart Institute in Portland, Oregon. Audio editing by CardioNerds Academy Intern, Christian Faaborg-Andersen. In the recent years, purified omega 3 fatty acids and its esters have emerged as a potential new tool in our arsenal for management of hypertriglyceridemia and atherosclerotic coronary artery disease. In this episode we review the sources and basic structure of these compounds, as well as their metabolic effects as it pertains to cardiovascular disease. Using hypothetical patient cases, we also discuss scenarios in which these therapies can be useful. This episode is part of the CardioNerds Lipids Series which is a comprehensive series lead by co-chairs Dr. Rick Ferraro and Dr. Tommy Das and is developed in collaboration with the American Society For Preventive Cardiology (ASPC). Relevant disclosures: None Pearls • Notes • References • Guest Profiles • Production Team CardioNerds Lipid Series PageCardioNerds Episode PageCardioNerds AcademyCardionerds Healy Honor Roll CardioNerds Journal ClubSubscribe to The Heartbeat Newsletter!Check out CardioNerds SWAG!Become a CardioNerds Patron! Pearls Omega 3 (n-3) fatty acids are a class of polyunsaturated fatty acids [PUFA]. The most studied n-3 fatty acids include eicosapentaenoic acid [EPA], docosahexaenoic acid [DHA] and alpha linoleic acid [ALA]. ALA is found in certain vegetable oils while EPA and DHA are abundant in fish sources.Cardiovascular benefits of n-3 fatty acids include blood pressure reduction, enhanced diastolic function, triglyceride reduction, and immunomodulatory properties.Inflammation plays a major role in the atherogenic process and plaque rupture. Inflammatory marker hs-CRP is a risk enhancing factor for predicting future ASCVD risk. Ongoing trials are investigating therapy that target the inflammatory process in treatment of atherosclerotic heart disease.Prevention and management of ASCVD require aggressive lifestyle modifications and medical therapy addressing risk factors and underlying inflammatory conditions.Purified forms of n-3 fatty acids are approved for the treatment of severe hypertriglyceridemia and as an adjunct therapy to statins for reduction of coronary events in high-risk individuals. Show notes 1. What are omega 3 (n-3) fatty acids? What are the natural sources of n-3 fatty acids? n-3 fatty acids are class of polyunsaturated fatty acids [PUFA]. PUFA are types of unsaturated fats that have more than one double bond in their backbone. PUFAs are important constituents of the phospholipids of all cell membranes.The most studied n-3 fatty acids include eicosapentaenoic acid [EPA], docosahexaenoic acid [DHA] and alpha linoleic acid [ALA].ALA is found in certain vegetable oils including walnuts, flaxseeds, chia seeds. EPA and DHA are abundant in cold water fish oils such as salmon, mackerel, tuna sardines. Interestingly, farm raised fish usually have higher levels of EPA and DHA than wild caught fish; however, this depends on what the fish are fed. Another important class of PUFAs are omega 6 (n-6) fatty acids, found in vegetable oils (1,2).  2. What are the metabolic effects of omega 3 fatty acids? Multiple early studies have demonstrated the anti-inflammatory properties of n-3 fatty acids.The typical Western diet with a high arachidonic acid (an n-6 fatty acid) content promotes atherogenesis leading to the high incidence of CAD in this population.
17 Feb 2022180. Narratives in Cardiology: Raising Women Leaders in Academic Cardiology with Dr. Anu Lala00:54:18
CardioNerds (Amit Goyal and Daniel Ambinder), Dr. Leticia Helms (Internal medicine resident at Columbia University), Dr. Silia DeFilippis (AHFT FIT at Columbia University), and Dr. Anu Lala (AHFT faculty and program director at Mount Sinai Hospital) to discuss diversity and inclusion in academic cardiology and more in this installment of the Narratives in Cardiology Series. The President of the New York ACC Chapter Dr. Hima Vidula discusses D&I initiatives at her chapter. Episode introduction and audio editing by CardioNerds Academy Intern, Shivani Reddy. Although women compose 50% of medical students in the United States, cardiology remains a male dominated field. Gender disparity is even more prominent when we look at leadership positions. In this episode we discuss why and how cardiology (and advanced heart failure) can be such a rewarding field for women. The episode reflects on the significant contributions women have made to the field in the past and how they continue to move the field with respect to clinical care and research. The PA-ACC & CardioNerds Narratives in Cardiology is a multimedia educational series jointly developed by the Pennsylvania Chapter ACC, the ACC Fellows in Training Section, and the CardioNerds Platform with the goal to promote diversity, equity, and inclusion in cardiology. In this series, we host inspiring faculty and fellows from various ACC chapters to discuss their areas of expertise and their individual narratives. Join us for these captivating conversations as we celebrate our differences and share our joy for practicing cardiovascular medicine. We thank our project mentors Dr. Katie Berlacher and Dr. Nosheen Reza. Video Version • Notes • Production Team Claim free CME just for enjoying this episode! There are no relevant disclosures for this episode. The PA-ACC & CardioNerds Narratives in Cardiology PageCardioNerds Episode PageCardioNerds AcademyCardionerds Healy Honor Roll CardioNerds Journal ClubSubscribe to The Heartbeat Newsletter!Check out CardioNerds SWAG!Become a CardioNerds Patron! Tweetorial - Raising Women Leaders in Academic Cardiology with Dr. Anu Lala https://twitter.com/Gurleen_Kaur96/status/1509286469051031556?s=20&t=9BFDUXbSnWLcAc5-daXK2A Video version - Raising Women Leaders in Academic Cardiology with Dr. Anu Lala https://youtu.be/vVuhUCbqwYk Quotables - Raising Women Leaders in Academic Cardiology with Dr. Anu Lala “We all have multiple roles – and those roles don’t always have to be completely distinct and separate from one another. You know, our ability in one role, maybe it makes us better at another.”“I felt like being there was like in medical Disneyland for research.” - Anu Lala“Heart failure is unique in that it truly requires that cross disciplinary collaboration at the precipice of what is often life and death.” - Anu Lala“It points to the importance of seeing people do things before you who look like you.” - Anu Lala“All human beings want to feel heard. They want to feel seen and they want to feel like their voice matters.”  - Anu Lala“consciously deliberately disruptive”“I like the idea of calling it work-life harmony.” Show notes - Raising Women Leaders in Academic Cardiology with Dr. Anu Lala How do women compare to men in cardiology in 2022?Although nearly 50% of US medical graduates and more than 40% of internal medicine graduates are women, the field of cardiology remains male dominated.Approximately 20% of general cardiology fellows are women, which is comparable to women in surgical subspecialties like neurosurgery (17%), thoracic surgery (21%), and orthopedic surgery (15%) (Reza 2021).Additionally, women hold significantly fewer leadership positions and are less likely to be promoted to senior academic ranks.Out of LBCT presented at ACC 2021, zero had a female first author and zero were presented by women (Kaur 2021).What may make heart failure unique with respect to the recruitment of ...
21 Feb 2022181. Aortic Stenosis and the Story of TAVR – Historical Perspective & Future Directions with Dr. Jon Resar00:48:33
CardioNerds, Daniel Ambinder and CardioNerds Academy Program Director, Dr. Tommy Das (Cardiology fellow, Cleveland Clinic), Dr. Jacqueline Latina (Structural heart fellow, Johns Hopkins) discuss aortic stenosis and the story of TAVR from both the historical perspective and in terms of future directions with Dr. Jon Resar, Professor of Medicine and Director of the Adult Catheterization Laboratory and Interventional Cardiology at the Johns Hopkins Hospital. This episode is brought to you for Heart Valve Disease Awareness Day. Audio editing by CardioNerds Academy Intern, Shivani Reddy. As many as 11 million Americans have heart valve disease (HVD)—a potentially disabling and deadly disease—yet 3 out of 4 Americans know little to nothing about heart valve disease. Learn more about valve disease. Pearls • Notes • References • Guest Profiles • Production Team CardioNerds Aortic Stenosis SeriesCardioNerds Episode PageCardioNerds AcademyCardionerds Healy Honor Roll CardioNerds Journal ClubSubscribe to The Heartbeat Newsletter!Check out CardioNerds SWAG!Become a CardioNerds Patron! Pearls and Quotes - Aortic Stenosis and the Story of TAVR In the previous century, patients with severe aortic stenosis who were treated “medically” had 50% mortality over 2 years after developing symptoms. Balloon aortic valvuloplasty was initially touted as extremely “efficacious” for aortic stenosis but follow-up studies showed that the improvement in symptoms were not durable, and long-term prognosis was dismal. The PARTNER Trial started enrolling in 2007 in extreme risk patients – patients who were not surgical candidates. In 2010, the PARTNER trial was published and TAVR blew away the “standard of care” in inoperable patients at the time, cutting outcomes in half (composite of death and repeat hospitalization). The PARTNER trial studied balloon expandable intra-annular valve implantation. The CoreValve trial studied self-expanding supra-annular valve implantation and was published in 2014. The “Heart Team” approach entails collaborative decision making between cardiologists and cardiac surgeons to personalize management for patients. Both intra-annular and supra-annular valves show non-inferior outcomes to surgery in intermediate and low risk patients. Revascularization prior to TAVR is an evolving arena; the trend has been interventionalists performing fewer PCIs prior to TAVR given the benefit is not clear if angina is not a prominent symptom. Show notes - Aortic Stenosis and the Story of TAVR (TAVR/TAVI are using interchangeably) CardioNerds Aortic Stenosis, updated 1.20.21 1. In the 1990s, patients with severe aortic stenosis (AS) who were deemed to be at high surgical risk would weigh the risks of surgery and prolonged recovery. Balloon Aortic Valvuloplasty (BAV) was first performed by Dr. Alain Cribier in 1986. The technique was based on the foundation of pulmonary valvuloplasty performed initially in 1982 by Drs. Jean Kan and Bob White, and mitral valvuloplasty in 1984. BAV was initially touted as an efficacious cure for aortic stenosis, but unfortunately it had a high restenosis rate as well as high risks for stroke and vascular complications (no closure devices at that time) with an overall poor long-term prognosis. Balloon aortic valvuloplasty was primarily used for decompensated Class IV heart failure in non-surgical candidates. 2. Transcatheter aortic valve replacement (TAVR/TAVI) was developed and first performed in human in 2002.(1) This was performed by Dr. Alain Cribier in France in 2002, initially by trans-septal approach and then by retroaortic approach. Here is a representative diagram of the procedure. Figure: Transcatheter Aortic-Valve Replacement. The transcatheter valve is positioned at the level of the native aortic valve during the final step of valve replacement, when the balloon is inflated within the native valve during a brief period of rapid ven...
27 Feb 2022182. Case Report: Dyspnea with an LVAD: A Tale of Hypoxia and Hemodynamics – Temple University00:47:14
CardioNerds (Amit Goyal & Karan Desai)  join Dr. Matthew Delfiner (Cardiology fellow, Temple University Hospital) and Dr. Katie Vanchiere (Internal medicine resident, Temple University Hospital) in the beautiful Fairmount Park in Philadelphia. They discuss a case of a 53-year-old man with an LVAD who presents with progressive dyspnea since LVAD implant due to right-to-left shunting due to a PFO. Dr. Val Rakita (Assistant professor of medicine and advanced heart failure and transplant specialist at Temple University Hospital) provides the E-CPR for this episode. Episode introduction by CardioNerds Clinical Trialist Dr. Anthony Peters (Duke Heart Center). This case has been published by Circulation: Heart failure. See Invasive Hemodynamic Study Unmasks Intracardiac Shunt With Ventricular Assist Device. Claim free CME just for enjoying this episode!  Disclosures: NoneJump to: Pearls - Notes - References CardioNerds Case Reports PageCardioNerds Episode PageCardioNerds AcademyCardionerds Healy Honor Roll CardioNerds Journal ClubSubscribe to The Heartbeat Newsletter!Check out CardioNerds SWAG!Become a CardioNerds Patron! Case Summary - Dyspnea with an LVAD: A Tale of Hypoxia and Hemodynamics A 53-year-old man with an LVAD placed 3 months prior presents with progressive dyspnea since LVAD implant, though it has acutely worsened over the past 2 weeks. Two weeks ago, he had a hemodynamic and echocardiographic ramp study, where the LVAD speed was increased. By increasing the speed, his LV was more adequately decongested, and flow improved. In the Emergency Department, he was hypoxic on room air, and remained so with escalation ultimately with intubation. Even then he remained severely hypoxic requiring cannulation to veno-venous ECMO. Chest imaging was normal, and LVAD parameters were normal without any alarms. An astute clinician noticed that when the patient became hypertensive, his oxygen saturation improved. A subsequent echocardiogram revealed a patent foramen ovale, with right to left shunting. The patient then went to the cath lab, where simultaneous right atrial and left atrial pressures and oxygen pressures were measured, along with trans-esophageal echocardiography, while adjusting LVAD speed. It became evident that right-to-left shunting occurred only when there was high LVAD speed and low peripheral blood pressure. Essentially, faster LVAD speeds (sucking blood from the LV) and low systemic blood pressure (reducing LV afterload) increased right to left shunting by decreasing the left atrial pressure relative to the right atrial pressure. The PFO was closed at that time, drastically improving oxygenation. He was decannulated and extubated the following day. Invasive Hemodynamic Study Unmasks Intracardiac Shunt With Ventricular Assist Device | Circulation: Heart Failure (ahajournals.org) Episode Teaching -Dyspnea with an LVAD: A Tale of Hypoxia and Hemodynamics Pearls PFOs are present in up to 25% of individuals, including those with LVADs.LV unloading, and therefore LA decompression, depends on both LVAD speed and systemic vascular resistance.Blood pressure dependent hypoxia may be suggestive of a right-to-left intracardiac shunt.Hypoxia refractory to mechanical ventilation should raise suspicion for intracardiac shunt.Patients with LVADs can suffer from the same diseases that anyone can. Notes - Dyspnea with an LVAD: A Tale of Hypoxia and Hemodynamics 1. What factors influence LVAD flow? Factors that influence LVAD flow include pump speed, blood pressure, volume status, RV function, cardiac rhythm, and some other variables. The faster the pump is spinning, the more flow you should provide (to an extent). However, if your LV is underfilled, either from systemic hypovolemia or an RV not providing the needed LV preload, then you have no blood to flow! If you have high systemic vascular resistance, then you will have less forward flow,
06 Mar 2022183. Cardio-Obstetrics: The Fourth Trimester: Postpartum and Long-term Cardiovascular Care after Hypertensive Disorders of Pregnancy with Dr. Malamo Countouris and Dr. Alisse Hauspurg01:01:01
CardioNerds (Amit Goyal), Dr. Natalie Stokes (Cardiology Fellow at UPMC and Co-Chair of the Cardionerds Cardio-Ob series), and episode lead Dr. Priya Freaney (Northwestern University cardiology fellow) discuss “The Fourth Trimester” with Dr. Malamo Countouris and Dr. Alisse Hauspurg, from the University of Pittsburgh Departments of Cardiology and Obstetrics and Gynecology, respectively. We discuss the cardiovascular considerations after adverse pregnancy outcomes in the postpartum and long-term follow-up periods. The discussion is focused mainly on hypertensive disorders of pregnancy (HDP), guided by a series of clinical vignettes. We cover a wide range of topics from cardiovascular complications and management considerations in the immediate postpartum period after a HDP, postpartum outpatient follow-up, long term cardiovascular morbidity related to HDP and related preventive strategies, contraceptive considerations for the cardiologist, and interdisciplinary care management pearls for cardiologists working in a cardio-obstetrics team. Notes • References • Guest Profiles • Production Team CardioNerds Cardio-Obstetrics Series PageCardioNerds Episode PageCardioNerds AcademyCardionerds Healy Honor Roll CardioNerds Journal ClubSubscribe to The Heartbeat Newsletter!Check out CardioNerds SWAG!Become a CardioNerds Patron! Pearls - The Fourth Trimester Blood pressures >160/110 should be treated like a true emergency during pregnancy and the postpartum period, as the cerebrovascular circulation is more sensitive to hypertension, due to hormonal changes related to pregnancy.Women with pre-eclampsia are at higher risk for peripartum cardiomyopathy. Have a low threshold to do a clinical heart failure evaluation (i.e., natriuretic peptides, echocardiogram), and administer diuretics as appropriate to improve volume status and blood pressure.Women with HDP should have their blood pressures monitored closely after discharge, ideally with a home BP monitoring program, as they can have exacerbations of their HTN for up to 2 weeks postpartum.The American Rescue Plan Act of 2021 included a landmark policy to extend postpartum Medicaid coverage up to a year postpartum (from 60 days).Remember to take a reproductive history for every woman you see in cardiology clinic! This can be done in one minute. At a minimum, include obstetric history [number of pregnancies, outcome of each pregnancy, gestational age and weight at delivery, pregnancy complications (HDP, GDM, etc), and delivery method] and menopausal history (age at menarche, age at menopause).The Pooled Cohort Equations may underestimate ASCVD risk for a woman who has had pregnancy complications or premature menopause – consider obtaining a CAC score to aid in risk-stratification in middle-aged women who may have underestimated risk.Low dose aspirin during pregnancy in women who have risk factors for pre-eclampsia reduces the risk of development of HDP by 15-20%. Quotables - The Fourth Trimester “Some of our traditional approaches to caring for women in the postpartum period just aren’t realistic…we need to think about how we can improve care from a policy standpoint to ensure women have access to care and think about how we deliver care.” – Dr. Alisse Hauspurg “Silos are never good. Cardio-obstetrics is a space where you really want to have open communications, be truly collaborative – taking into consideration the expertise of multiple disciplines…because it’s really hard to do it alone.” – Dr. Malamo Countouris Show notes - The Fourth Trimester For more on hypertensive disorders of pregnancy enjoy: Episode #128: Cardio-Obstetrics: Hypertensive Disorders of Pregnancy with Dr. Jennifer LeweyEpisode #66: Case Report: Severe Pre-eclampsia & Cardio-Obstetrics – UPMC Hypertensive Disorders of Pregnancy 1.     What are some of the immediate postpartum cardiovascular risks and complications following a hypertensive disorder of pregnancy (H...
14 Mar 2022184. CardioNerds Rounds: Challenging Cases of Cardiovascular Prevention with Dr. Martha Gulati00:46:01
CardioNerds Rounds Co-Chairs, Dr. Karan Desai and Dr. Natalie Stokes and CardioNerds Academy Fellow, Dr. Najah Khan, join Dr. Martha Gulati – President-Elect of the American Society for Preventive Cardiology (ASPC) and prior Chief of Cardiology and Professor of Medicine at the University of Arizona – to discuss challenging cases in cardiac prevention. As an author on numerous papers regarding cardiac prevention and women’s health, Dr. Gulati provides many prevention pearls to help guide patient care. Come round with us today by listening to the episodes now and joining future sessions of #CardsRounds! This episode is supported with unrestricted funding from Zoll LifeVest. A special thank you to Mitzy Applegate and Ivan Chevere for their production skills that help make CardioNerds Rounds such an amazing success. All CardioNerds content is planned, produced, and reviewed solely by CardioNerds. Case details are altered to protect patient health information. CardioNerds Rounds is co-chaired by Dr. Karan Desai and Dr. Natalie Stokes.  Speaker disclosures: None Cases discussed and Show Notes • References • Production Team CardioNerds Rounds PageCardioNerds Episode PageCardioNerds AcademyCardionerds Healy Honor Roll CardioNerds Journal ClubSubscribe to The Heartbeat Newsletter!Check out CardioNerds SWAG!Become a CardioNerds Patron! Show notes - CardioNerds Rounds: Challenging Cases of Cardiovascular Prevention with Dr. Martha Gulati Case #1 Synopsis: A 55-year-old South Asian woman presents to prevention clinic for an evaluation of an elevated LDL-C. Her prior history includes hyperlipidemia, hypertension, obesity, and pre-eclampsia. She was told she had “high cholesterol” a few years prior and would need medication. She started exercising regularly and cut out sweets from her diet. Before clinic, labs showed: Total Cholesterol (mg/dL) of 320, HDL 45, Triglycerides 175, and (directly measured) LCL-C 180. Her Lipoprotein(a) is 90 mg/dL (ULN being ~ 30 mg/dL). Her HbA1C is 5.2% and her 10-year ASCVD Risk (by the Pooled Cohorts Equation) is 5.4%. Her recent CAC score was 110. She prefers not to be on medication and seeks a second opinion. Takeaways from Case #1 As Dr. Gulati notes, in the 2019 ACC/AHA Guideline on the Primary Prevention of Cardiovascular Disease, South Asian ethnicity is considered a “risk enhancing factor.” The pooled cohort equations (PCE) may underestimate risk in South Asians. Furthermore, risk varies within different South Asian populations, with the risk for cardiovascular events seemingly higher in those individuals of Bangladeshi versus Pakistani or Indian origin. There are multiple hypotheses for why this may be the case including cultural aspects, such as diet, physical activity, and tobacco use. A better understanding of these factors could inform targeted preventive measures.In the same 2019 ACC/AHA Guideline on the Primary Prevention of Cardiovascular Disease mentioned above, history of an adverse pregnancy outcome (APO) increases later ASCVD risk (e.g., preeclampsia) and is also included as a “risk-enhancing factor.” Studies have shown that preeclampsia is an independent risk factor for developing early onset coronary artery calcification. Recent data has shown that the risk for developing preeclampsia is not the same across race and ethnicity, with Black women more likely to develop preeclampsia. Black women also had the highest rates of peripartum cardiomyopathy, heart failure, and acute renal failure. After adjustment for socioeconomic factors and co-morbidities, preeclampsia was associated with increased risk of CVD events in all women, the risk was highest among Asian and Pacific Islander women. Listen to Episode #174. Black Maternal Health with Dr. Rachel Bond to learn more about race-based disparities in cardio-obstetric care and outcomes.Our patient thus has multiple risk-enhancing factors to help in shared decision making and personalize her decision...
16 Mar 2022185. ACHD: Tetralogy of Fallot with Dr. George Lui01:00:26
CardioNerds (Daniel Ambinder), ACHD series co-chair,  Dr. Josh Saef (ACHD fellow at University of Pennsylvania), and ACHD FIT lead Dr. Charlie Jain (Mayo Clinic) join ACHD expert Dr. George Lui (Medical Director of The Adult Congenital Heart Program at Stanford and Program Director for the ACGME adult congenital heart disease fellowship at Stanford) to discuss Tetrology of Fallot. Audio editing by CardioNerds Academy Intern, Dr. Leticia Helms. Tetralogy of Fallot (ToF) is the most common cyanotic heart disease and one of the most common congenital heart diseases that we see in adults overall. The anatomy includes a ventricular septal defect (VSD), an overriding aorta, and infundibular hypertrophy with subpulmonic +/- pulmonic valvular +/- supravalvular stenosis, which causes severe RV outflow obstruction and subsequent RV hypertrophy. Patients require surgery during childhood, which includes patching the VSD and relieving RV outflow obstruction. This results in pulmonic regurgitation (usually severe) and patients can live with this for decades. Adults with ToF commonly will require pulmonic valve replacement, potential relief of subvalvular or supravalvular stenoses, and tricuspid valve repair (for functional tricuspid regurgitation caused by RV dilation). These patients are at increased risk of atrial and ventricular arrhythmias and may warrant prophylactic ICDs. The CardioNerds Adult Congenital Heart Disease (ACHD) series provides a comprehensive curriculum to dive deep into the labyrinthine world of congenital heart disease with the aim of empowering every CardioNerd to help improve the lives of people living with congenital heart disease. This series is multi-institutional collaborative project made possible by contributions of stellar fellow leads and expert faculty from several programs, led by series co-chairs, Dr. Josh Saef, Dr. Agnes Koczo, and Dr. Dan Clark. The CardioNerds Adult Congenital Heart Disease Series is developed in collaboration with the Adult Congenital Heart Association, The CHiP Network, and Heart University. See more Disclosures: None Pearls • Notes • References • Guest Profiles • Production Team CardioNerds Adult Congenital Heart Disease PageCardioNerds Episode PageCardioNerds AcademyCardionerds Healy Honor Roll CardioNerds Journal ClubSubscribe to The Heartbeat Newsletter!Check out CardioNerds SWAG!Become a CardioNerds Patron! Pearls - Tetralogy of Fallot Tetralogy of Fallot is the most common cyanotic heart disease and the 4 anatomic features are: VSD, infundibular hypertrophy (with RVOT obstruction), overriding aorta, and RV hypertrophy. The most common lesion you will see in adults with repaired Tetralogy of Fallot is pulmonic regurgitation.Pulmonic regurgitation (PR) can be easy to miss on exam as the murmur is brief and even shorter when the PR is severe. In patients with PR and aortic regurgitation, remember PR is clearest when laying supine, in comparison to aortic regurgitation which is loudest while leaning forward.Patients with ToF may also have coronary anomalies (e.g. LAD off RCA), right-sided aortic arches, and also left-sided heart disease (LV diastolic or systolic dysfunction).Patients with ToF are at risk for atrial and ventricular arrhythmias, and clinicians should consider prophylactic ICD for those with multiple risk factors for sudden death (e.g. QRS >180ms, scar on MRI).In all patients with congenital heart disease, inspection is a key part of the physical exam (e.g. right thoracotomy could clue you into a prior BTT shunt) and in patients with prior BTT shunts and/or prior brachial cut-downs (look in the antecubital fossa for scars), radial arterial access is discouraged. Show notes - Tetralogy of Fallot LesionTTETEECardiac MRICardiac CTTetralogy of Fallot(1) Routine assessment of RV and LV size and function (2) Routine semiquantitative assessment of pulmonic valve regurgitation (3) Evaluation of PVR/conduit gradients,
20 Mar 2022186. Case Report: Coronary Artery Bypass Grafting: An Iatrogenic Left to Right Cardiac Shunt – SUNY Downstate00:47:27
CardioNerds (Amit Goyal and Daniel Ambinder) and guest host, Dr. Priya Kothapalli (UT Austin fellow and CardioNerds Ambassador), join SUNY Downstate cardiology fellows, Dr. Eric Kupferstein and Dr. Gautham Upadhya to discuss a case about a patient who had coronary artery bypass grafting that was complicated by a LIMA grafted to the great cardiac vein. Dr. Alan Feit (Professor of Medicine, SUNY Downstate) provides the E-CPR for this episode. Dr. Moritz Wyler von Ballmoos (Director, robotic cardiac and vascular surgery for Houston Methodist Cardiovascular Surgery Associates) provides a special perspective regarding coronary artery bypass grafting as it relates to this case. Episode introduction with CardioNerds Clinical Trialist Dr. Jana Lovell (Johns Hopkins). Left Internal Mammary Artery (LIMA) to Left Anterior Descending (LAD) artery anastomosis is the cornerstone of Coronary Artery Bypass Graft (CABG) surgery. Anastomosis of the LIMA to the Great Cardiac Vein (GCV) is a known but rare complication of the surgery. Currently there are no clear guidelines in regard to further management. We report a case of a LIMA to GCV anastomosis managed with a drug eluting stent (DES) to the mid LAD after ruling out a significant left to right heart shunt. Jump to: Case media - Case teaching - References CardioNerds Case Reports PageCardioNerds Episode PageCardioNerds AcademyCardionerds Healy Honor Roll CardioNerds Journal ClubSubscribe to The Heartbeat Newsletter!Check out CardioNerds SWAG!Become a CardioNerds Patron! Case Media - Coronary Artery Bypass Grafting: An Iatrogenic Left to Right Cardiac Shunt Angiography Episode Schematics & Teaching - Coronary Artery Bypass Grafting: An Iatrogenic Left to Right Cardiac Shunt Pearls - Coronary Artery Bypass Grafting: An Iatrogenic Left to Right Cardiac Shunt Listen to the patient's story. The patient determines when the angina is no longer stable angina.The placebo effect of our interventions should not be discounted.LIMA to GCV anastomosis creates a left to right cardiac shunt. A Qp:Qs greater than 1.5 signifies a significant shunt.Increasing the pressure in the coronary sinus may actually be beneficial to the patient.LIMA-LAD is remains the most efficacious and long lasting graft but why not other arterial grafts? Notes - Coronary Artery Bypass Grafting: An Iatrogenic Left to Right Cardiac Shunt Iatrogenic anastomosis of the LIMA to the GCV is a rare but noted complication of CABG surgery. Review of the literature has reported under 40 such cases of arteriovenous fistula formation in the coronary system. Detection of the anastomosis generally stems from recurrent angina which can be attributed to unresolved ischemia or coronary steal syndrome but also can be detected with new heart failure (namely right sided heart failure due to left to right shunting). Diagnosis is usually made with coronary angiography, but CT coronary angiography has also been reported. Due to the rarity of this complication, no clear guidelines are in place directing the management leaving it to the discretion of the various Heart Teams. Evaluating for signs of heart failure and/or ischemia, and measuring the Qp:Qs have been the most common signs directing management. Various options are available for closing the fistula and include coil or balloon embolization, vascular plugs, venous ligation or a covered stent. Redoing the surgery is also an option. Spontaneous closure of the fistula has also been reported. Lastly, if redo surgery is not performed then regardless of fistula closure, coronary intervention for the native diseased artery may be pursued to relieve symptoms. References Boden et al; COURAGE Trial Research Group. Optimal medical therapy with or without PCI for stable coronary disease. N Engl J Med. 2007 Apr 12;356(15):1503-16. doi: 10.1056/NEJMoa070829. Epub 2007 Mar 26. PMID: 17387127.
21 Mar 2022187. Guidelines: 2021 ESC Cardiovascular Prevention – Question #1 with Dr. Eugene Yang
This question refers to Sections 3.2 and 3.3 of the 2021 ESC CV Prevention Guidelines. The question is asked by CardioNerds Academy Intern, student Dr. Hirsh Elhence, answered first by Ohio State University Cardiology Fellow Dr. Alli Bigeh, and then by expert faculty Dr. Eugene Yang. Dr. Yang is professor of medicine of the University of Washington where he is medical director of the Eastside Specialty Center and the co-Director of the Cardiovascular Wellness and Prevention Program. Dr. Yang is former Governor of the ACC Washington Chapter and current chair of the ACC Prevention of CVD Section. The CardioNerds Decipher The Guidelines Series for the 2021 ESC CV Prevention Guidelines represents a collaboration with the ACC Prevention of CVD Section, the National Lipid Association, and Preventive Cardiovascular Nurses Association. Question #1 A 48-year-old Pakistani woman with rheumatoid arthritis comes to your clinic asking how she can reduce her risk of ASCVD. Her mother died of an MI at age 45, her father is healthy at age 79. Her calculated 10-year risk based on SCORE2 is 3%. SBP is 120 mmHg, LDL is 120 mg/dL. What is the next best step? A. Order an echocardiogram B. Schedule a follow-up appointment in 1 year C. Discuss initiating a statin D. Repeat lipid panel in 3-5 years  Answer #1 Answer: C. Discuss Initiating a statin The absolute benefit derived from risk factor modification depends on the absolute risk of CVD and the absolute improvements in each risk factor category. Risk factor treatment recommendations are based on categories of CVD risk (“low-to-moderate”, “high”, and “very high”). The cut-off risk levels for these categories are numerically different for various age groups to avoid undertreatment in the young and to avoid overtreatment in the elderly. As age is a major driver of CVD risk, but lifelong risk factor treatment benefit is higher in younger people, the risk thresholds for considering treatment are lower for younger people as per the ESC guidelines. Treatment decisions should be made with shared decision-making valuing patient preference.  Option A is INCORRECT- there is a lack of convincing evidence that echocardiography improves CVD risk reclassification, and it is NOT recommended to improve CV risk prediction. (Class III, LOE B) Option B is INCORRECT- simply doing nothing is not appropriate for this patient with elevated CVD risk.  Option C is CORRECT- This patient has a seemingly low 10-year CVD risk based on SCORE 2 of 3% and her SBP is controlled; however, given her age she is considered as having high CVD risk, therefore treatment should be considered. Stepwise approach involves targeting LDL <100 (class IIa) so initiating a statin would be appropriate. This patient also carries several risk enhancing modifiers including Pakistani ethnicity, family history of premature CVD, and inflammatory comorbidity. All patients should be counseled on smoking cessation, lifestyle modifications, and target SBP <160 mmHg. Option D is INCORRECT- repeating a lipid panel without risk factor modification will not change treatment recommendations for this patient with elevated CVD risk.  Main Takeaway In summary, when a patient <50 years old without established ASCVD has an estimated 10-year risk 2.5 to <7.5% they are considered high CVD risk and risk factor treatment should be considered. Risk modifiers should also be taken into consideration. *Of note- ACC/AHA guidelines recommend the ASCVD risk calculator to estimate 10-year risk and do not restructure CVD risk groups according to age groups. High risk in the ACC/AHA guidelines is considered to be >20%. Guideline Location Table 5 and Figure 5, Page 32513.2.3.4, Page 32533.2.3, Figure 6 page 32523.3, Pages 3258-3259  CardioNerds Decipher the Guidelines - 2021 ESC Prevention SeriesCardioNerds Episode PageCardioNerds AcademyCardionerds Healy Honor RollCardioNerds Journal ClubSubscribe to The Heartbea...
22 Mar 2022188. Guidelines: 2021 ESC Cardiovascular Prevention – Question #2 with Dr. Allison Bailey00:14:00
The following question refers to Section 3.3 of the 2021 ESC CV Prevention Guidelines. The question is asked by CardioNerds Academy Intern student Dr. Adriana Mares, answered first by Brigham & Women’s medicine intern & Director of CardioNerds Internship Dr. Gurleen Kaur, and then by expert faculty Dr. Allison Bailey.Dr. Bailey is an advanced heart failure and transplant cardiologist at Centennial Heart. She is the editor-in-chief of the American College of Cardiology's Extended Learning (ACCEL) editorial board and was a member of the writing group for the 2018 American Lipid Guidelines. The CardioNerds Decipher The Guidelines Series for the 2021 ESC CV Prevention Guidelines represents a collaboration with the ACC Prevention of CVD Section, the National Lipid Association, and Preventive Cardiovascular Nurses Association. Question #2 Mr. Early M. Eye is a 55-year-old man with a history of GERD who is seeing you in clinic as he is concerned about his family history of early myocardial infarction and would like to discuss if he should be taking a statin for cardiovascular prevention. He has never smoked tobacco. His 10-year CVD risk is estimated to be 8%. Which imaging modality is recommended by the ESC guidelines to reclassify his CVD risk?A. Coronary Artery Calcium (CAC) scoringB. Echocardiography C. Ankle brachial index D. Contrast enhanced computed tomography coronary angiography (CCTA)E. None of the above Answer #2 The correct answer is A.Coronary artery calcium (CAC) scoring can reclassify CVD risk upwards and downwards and should specifically be considered in patients with calculated risk scores that are around decision thresholds. CAC scores which are high-than-expected for age and sex increase estimated future CVD risk. Notably, CAC scoring may also be used to “de-risk” if CAC is absent or lower-than-expected. The 2021 ESC Prevention Guidelines give a Class IIb (LOE B) recommendation to consider CAC scoring to improve risk classification around treatment decision thresholds. However, one limitation of CAC is that it does not provide direct information on total plaque burden or stenosis severity. In addition, there is also a Class IIb (LOE B) recommendation to use plaque detection by carotid ultrasound as an alternative when CAC scoring is unavailable or not feasible. Plaque assessed through carotid ultrasound is defined as presence of wall thickening that is >50% greater than the surrounding vessel wall or a focal region with intima-media thickness measurement >1.5mm that protrudes into the lumen.Similar to the ESC Prevention Guidelines, the 2019 ACC/AHA guidelines on primary prevention of CVD also have a Class IIa recommendation for using CAC score, and explicitly mention its use for adults at intermediate risk (>7.5% to <20% 10-year ASCVD risk) with cut-offs including >100 Agatson units to reclassify risk upwards and CAC of 0 to reclassify risk downwards. However, the guidelines also mention that clinicians should not down-classify risk in patients who have CAC of 0 if they are current smokers, have diabetes, have a family history of ASCVD, or have chronic inflammatory conditions. Furthermore, the 2018 ACC/AHA Cholesterol guidelines have a Class IIa recommendation that if CAC is 0, it is reasonable to withhold statin therapy and reassess risk in 5 to 10 years, as long as higher risk conditions that we just discussed are absent. If CAC is 1-99, it is reasonable to initiate statin therapy for patients ≥ 55 years of age.Choice B is incorrect. Echocardiography is not recommended to improve CV risk prediction due to lack of convincing evidence that it improves CVD risk reclassification.Choice C is incorrect. While the 2013 ESC guidelines mentioned that ABI may be considered as a risk modifier in CVD risk estimation, the newer 2021 guidelines state that ankle brachial index has limited potential in terms of reclassification risk, though an individual patient data meta-analysis showed th...
23 Mar 2022189. Guidelines: 2021 ESC Cardiovascular Prevention – Question #3 with Dr. Kim Williams00:14:11
The following question refers to Section 4.3 of the 2021 ESC CV Prevention Guidelines. The question is asked by CardioNerds Academy Intern Dr. Maryam Barkhordarian, answered first by medicine resident CardioNerds Academy House Chief Dr. Ahmed Ghoneem, and then by expert faculty Dr. Kim Williams. Dr. Williams is Chief of the Division of Cardiology and is Professor of Medicine and Cardiology at Rush University Medical Center. He has served as President of ASNC, Chairman of the Board of the Association of Black Cardiologists (ABC, 2008-2010), and President of the American College of Cardiology (ACC, 2015-2016). The CardioNerds Decipher The Guidelines Series for the 2021 ESC CV Prevention Guidelines represents a collaboration with the ACC Prevention of CVD Section, the National Lipid Association, and Preventive Cardiovascular Nurses Association. Question #3 Mrs. B is a 56-year-old African American woman with a past medical history significant for type 2 diabetes (HbA1C 7.6) and hypercholesterolemia. Her calculated ASCVD risk score today is 12.5% and her BMI is 24kg/m2. She is concerned about her high cholesterol levels despite being on a statin and feels that her diet is “not healthy enough.” She is interested in making dietary changes to help reduce her ASCVD risk. Which of the following recommendations is appropriate?A. Sodium restriction to <3g /day will be of no benefit because she is not hypertensive.B. Isocaloric substitution of saturated fat with polyunsaturated fat is associated with reduction of CHD risk.C. Dietary fiber intake is associated with GI benefits but has no CV risk reduction benefit.D. Supplementing diet with vitamins A, B, C and E helps reduce ASCVD risk. Answer #3 The correct answer is B.Risk of CHD is reduced when dietary saturated fats are replaced with other foods having similar caloric values. The greatest reduction was observed when saturated fats were isocalorically replaced with polyunsaturated fats (↓25%), followed by monounsaturated fats (↓15%) and carbohydrates from whole grains (↓9%). This is a class 1a recommendation in the ESC guidelines and a class IIa recommendation in the 2019 ACC/AHA guidelines. Conversely, increased trans fatty acid intake is associated with increased CHD risk. A regulation of the European Union (EU) Commission has set the upper limit of trans fats to 2 g per 100 g of fat. The ACC/AHA guidelines recommend that the intake of trans fats should be avoided (a class III: harm).Choice A is incorrect because dietary sodium restriction is recommended not only for control of blood pressure, but also for reduction of ASCVD risk. In a meta-analysis, salt reduction of 2.5 g/day resulted in a 20% reduction of ASCVD events (RR 0.80). Reduction of salt intake is a class 1 recommendation in the ESC guidelines compared to a class IIa recommendation in the 2019 ACC/AHA guidelines.Choice C is incorrect because a 10 g/day higher fiber intake was associated with a 16% lower risk of stroke (RR 0.84) and a 6% lower risk of type 2 DM (RR 0.94). A high fiber intake may reduce postprandial glucose responses after carbohydrate-rich meals and also lower triglyceride levels. The Mediterranean diet is rich in fiber (it includes high intakes of fruits, vegetables, pulses and wholegrain products) and is a class I recommendation.Choice D is incorrect because while vitamin supplementation has been associated with reduction in ASCVD risk in observational studies, intervention trials have failed to show any benefit. Main Takeaway:A healthy diet is recommended as a cornerstone of CVD prevention in all individuals, independent of their underlying co-morbidities. Replacing saturated with unsaturated fats, reducing salt intake, and choosing a more plant-based diet that is rich in fiber can lower risk of CVD.Guideline LocationSection 4.3.2, Page 3270 CardioNerds Decipher the Guidelines - 2021 ESC Prevention SeriesCardioNerds Episode PageCardioNerd...
24 Mar 2022190. Guidelines: 2021 ESC Cardiovascular Prevention – Question #4 with Dr. Roger Blumenthal00:10:36
The following question refers to Section 4.7 and figure 16 of the 2021 ESC CV Prevention Guidelines. The question is asked by CardioNerds Academy Intern Student Dr. Shivani Reddy, answered first by Fellow at Johns Hopkins Dr. Rick Ferraro, and then by expert faculty Dr. Roger Blumenthal.Dr. Roger Blumenthal is professor of medicine at Johns Hopkins where he is Director of the Ciccarone Center for the Prevention of Cardiovascular Disease. He was instrumental in developing the 2018 ACC/AHA CV Prevention Guidelines. Dr. Blumenthal has also been an incredible mentor to CardioNerds from our earliest days.The CardioNerds Decipher The Guidelines Series for the 2021 ESC CV Prevention Guidelines represents a collaboration with the ACC Prevention of CVD Section, the National Lipid Association, and Preventive Cardiovascular Nurses Association. Question #4 Ms. K.M. is a 40-year-old woman presenting to the outpatient clinic for a routine physical exam required for her employment as an airline stewardess. She states she has been in her usual good health but does experience occasional headaches and lightheadedness while in flight. On exam her BP was noted to be 170/90. The diagnosis of hypertension is confirmed during a subsequent clinic visit. What would be the most appropriate initial therapy recommendation(s) for Ms. K.M.?A. Initiate single drug therapy with a beta-blocker.B. Discuss and initiate lifestyle interventionsC. Initiate two-drug combination therapy with a thiazide-like diuretic, BB, CCB, or an ARB.D. Both B and C  Answer #4 The correct answer is D. Both B (lifestyle interventions) and C (initial combination therapy) are appropriate at this time. Lifestyle interventions are indicated for all patients with high-normal BP or hypertension because they can delay the need for drug treatment or complement the BP-lowering effect of drug treatment (Class 1). Moreover, most lifestyle interventions have health benefits beyond their effect on BP. Single-drug therapy will rarely achieve optimal BP control. Therefore, initial antihypertensive therapy with a combination of two drugs, preferably as a single-pill combination, is recommended for the management of HTN (Class 1). The only exceptions would be patients with a baseline BP close to the recommended target, who might achieve that target with a single drug, or very old (>80 years) or frail patients who may better tolerate a gentler reduction of BP. Five major classes of BP-lowering drug therapy have shown benefit in reducing CV events; angiotensin-converting enzyme (ACE) inhibitors, angiotensin receptor blockers (ARBs), beta-blockers, calcium channel blockers (CCBs), and thiazide or thiazide-like diuretics. A combination of an ACE inhibitor or ARB with a CCB or thiazide/thiazide-like diuretic is the preferred initial therapy for most patients with hypertension (Class 1). For those in whom treatment requires escalation to three drugs, a combination of an ACE inhibitor or ARB with a CCB and a thiazide/thiazide-like diuretic should be used (Class 1). Resistant hypertension is defined as BP being uncontrolled despite treatment with optimal or best-tolerated doses of three or more drugs including a diuretic, and confirmed by ABPM or HBPM. Spironolactone is the most effective drug for lowering BP in resistant hypertension when added to existing treatment; however, the risk of hyperkalaemia is increased in patients with CKD. When spironolactone is not tolerated, amiloride, alpha-blockers, beta-blockers, or centrally acting drugs, such as clonidine, have evidence supporting their use. Renal denervation and device-based therapy may be considered for specific cases. Beta-blockers should be used when there is a specific indication (e.g. angina, post myocardial infarction, arrythmia, HFrEF, or as an alternative to an ACE inhibitor or ARB in women of child-bearing potential). Combinations of an ACE inhibitor and an ARB are not recommended because of no added ben...
25 Mar 2022191. Guidelines: 2021 ESC Cardiovascular Prevention – Question #5 with Dr. Laurence Sperling00:11:22
The following question refers to Section 4.10 of the 2021 ESC CV Prevention Guidelines. The question is asked by CardioNerds Academy Intern student Dr. Christian Faaborg-Andersen, answered first by UCSD fellow Dr. Patrick Azcarate, and then by expert faculty Dr. Laurence Sperling. Dr. Laurence Sperling is the Katz Professor in Preventive Cardiology at the Emory University School of Medicine and Founder of Preventive Cardiology at the Emory Clinic. Dr. Sperling was a member of the writing group for the 2018 Cholesterol Guidelines, serves as Co-Chair for the ACC's Cardiometabolic and Diabetes working group, and is Co-Chair of the WHF Roadmap for Cardiovascular Prevention in Diabetes. The CardioNerds Decipher The Guidelines Series for the 2021 ESC CV Prevention Guidelines represents a collaboration with the ACC Prevention of CVD Section, the National Lipid Association, and Preventive Cardiovascular Nurses Association. Question #5 The European Society of Cardiology Prevention guidelines currently recommend that low-dose colchicine (0.5mg/day) may be considered for the primary prevention of cardiovascular disease.A. TrueB. False Answer #5 The correct answer is False. The correct answer is False.The European Society of Cardiology recommends that low-dose colchicine may be considered as an adjunctive therapy for secondary rather than primary prevention of cardiovascular disease in individuals whose risk factors are otherwise insufficiently controlled (Class IIb, LOE A). A broad evidence base currently supports that inflammation has pro-atherosclerotic effects and that reducing inflammation may reduce atherogenesis in high-risk patients.The initial LoDoCo trial in 2013 first demonstrated a 10.7% absolute risk reduction in acute coronary syndrome, out of hospital cardiac arrest, and non-cardioembolic ischemic stroke with daily low-dose colchicine; however, results were clouded by small sample size. Subsequently, the CANTOS trial in 2017 demonstrated a 15% relative reduction in non-fatal myocardial infarction, non-fatal stroke, and cardiovascular death with Canakinumab, an anti-inflammatory monoclonal antibody inhibitor of interleukin-1. More recently, the COLCOT trial in 2019 studying patients with recent AMI and LoDoCo2 trial in 2021 studying patients with stable chronic CAD both demonstrated reductions in myocardial infarction, cardiovascular mortality, CVA, and ischemia-driven revascularization with colchicine 0.5mg/day. In the LoDoCo2 trial, stable CAD was defined either angiographically, by coronary CT, CAC >400, or history of CABG >10 years prior with evidence of failed grafts or angioplasty since that time.In high-risk individuals with stable ischemic heart disease, the most recent evidence suggests that once daily low dose colchicine may reduce myocardial infarction and other ischemic events. Future studies may assess the biochemical markers including the trend of lipids and inflammatory markers to identify subpopulations that may benefit most from this therapy.Main Takeaway:Based upon the 2021 ESC Prevention Guidelines, clinicians may consider initiating low-dose colchicine (0.5mg/day) for secondary prevention of cardiovascular disease, particularly if other risk factors are insufficiently controlled or if recurrent CVD events occur despite optimal therapy.Guideline Location: Section 4.10, page 3291. CardioNerds Decipher the Guidelines - 2021 ESC Prevention SeriesCardioNerds Episode PageCardioNerds AcademyCardionerds Healy Honor RollCardioNerds Journal ClubSubscribe to The Heartbeat Newsletter!Check out CardioNerds SWAG!Become a CardioNerds Patron!
25 Mar 2022192. Guidelines: 2021 ESC Cardiovascular Prevention – Question #6 with Dr. Melissa Tracy00:11:34
The following question refers to Section 4.8 of the 2021 ESC CV Prevention Guidelines. The question is asked by CardioNerds Academy Intern student Dr. Christian Faaborg-Andersen, answered first by UCSF resident Dr. Jessie Holtzman, and then by expert faculty Dr. Melissa Tracy. Dr. Tracy is a preventive cardiologist, echocardiographer, Director of Cardiac Rehabilitation, and solid organ transplant cardiologist at Rush University. The CardioNerds Decipher The Guidelines Series for the 2021 ESC CV Prevention Guidelines represents a collaboration with the ACC Prevention of CVD Section, the National Lipid Association, and Preventive Cardiovascular Nurses Association. Question #6 A 62-year-old man with a history of non-obstructive coronary artery disease, heart failure with reduced ejection fraction (EF 30-35%), stage III chronic kidney disease, and type II diabetes mellitus presents to your clinic to establish care. His only medications are aspirin 81 mg daily and metformin 1000 mg BID, which he has taken since being diagnosed with diabetes mellitus 5 years ago. His hemoglobin A1c is 6.8%. What changes would you recommend to his medications at this time?A. Start glipizideB. Start saxagliptinC. Start empagliflozinD. No changes Answer #6 The correct answer is C – start empagliflozin.The Trials involving SGLT-2 inhibitors and GLP-1R agonists have shown cardiovascular benefits independent of glycemic control and metformin use.The ADA recommends metformin as a first-line therapy for all patients with type 2 DM. The ESC also recommends metformin as first-line therapy but only in patients without ASCVD, CKD, or HF (Class I, LOE B). If a patient has ASCVD, metformin can be considered (Class IIa, LOE B). Rather, for those patients with type 2 DM and ASCVD, the ESC recommends the use of GLP-1R agonist or SGLT-2 inhibitors with proven outcome benefits to reduce CV and/or cardiorenal outcomes (Class I, LOE A). Additionally, for those with type 2 DM and either CKD or HFrEF, the ESC recommends the use of SGLT-2 inhibitor to improve outcomes (Class I, LOE A).In contrast to the ADA, the view of the ESC is that metformin should be considered but is not mandatory first-line treatment in patients with diabetes and ASCVD or evidence of target organ damage. The initiation of metformin in such patients should not forego or delay the initiation of evidence-based SGLT2 inhibitors or GLP-1RAs.Therefore, the next best step for our patient is to start an SGLT-2 inhibitor given his history of CAD, HF, and CKD. While this patient’s A1c goal is within the range recommended for patients with Type 2 DM and ASCVD (<7%), given his CAD, HF, and CKD an SGLT-2 inhibitor should still be added.Saxagliptin is a DPP-4 inhibitor, a class of drugs that showed no effect of MACE but increased risk of HF hospitalization in patients with DM and existing.Lifestyle management is a top priority for ASCVD prevention and management of DM. Lifestyle intervention lowers future microvascular and macrovascular risks as well as mortality in the longer term. Intensive lifestyle changes with low-calorie diets and mean weight losses in the region of 10 kg leads to remission of type 2 DM in around 46% of cases at 1 year and 36% by 2 years. Smoking cessation, a diet low in saturated fat and high in fiber, aerobic physical activity, strength training, and reduction in energy intake for weight optimization are all recommended for patient with diabetes mellitus (Class I).Main TakeawayIn patients with Type 2 DM and ASCVD or end organ dysfunction, SGLT-2 inhibitors or GLP-1R agonists should be recommended regardless of background therapy or glycemic control. For patients with type 2 diabetes mellitus and CKD or HFrEF, SGLT-2 inhibitor is recommended.Guideline LocationSection 4.8.1, Pages 3289-90. CardioNerds Decipher the Guidelines - 2021 ESC Prevention SeriesCardioNerds Episode PageCardioNerds AcademyCardionerds Healy Honor RollC...
27 Mar 2022193. CCC: Cardiogenic Shock and Valvular Heart Disease with Dr. Paul Cremer
In this episode we discuss cardiogenic shock due to valvular heart disease. Join Dr. Pranoti Hiremath (Interventional cardiology fellow, Johns Hopkins), Dr. Karan Desai (CN Critical Care Series Co-Chair, Cardiology fellow, University of Maryland), Dr. Yoav Karpenshif (CN Critical Care Series Co-Chair, Chief cardiology fellow, University of Pennsylvania), and Amit Goyal (CardioNerds Co-Founder) as they interview Dr. Paul Cremer (Associate Director of the Cardiac Intensive Care Unit and Associate Director of the Cardiovascular Fellowship at the Cleveland Clinic) in this broad overview of valvular shock. We discuss the nuances in diagnosis, differing presentations and how physical exam, multi-modality imaging, and invasive hemodynamics can inform management. Audio editing by Dr. Gurleen Kaur (Director of the CardioNerds Internship and CardioNerds Academy Fellow). The CardioNerds Cardiac Critical Care Series is a multi-institutional collaboration made possible by contributions of stellar fellow leads and expert faculty from several programs, led by series co-chairs, Dr. Mark Belkin, Dr. Eunice Dugan, Dr. Karan Desai, and Dr. Yoav Karpenshif. Pearls • Notes • References • Guest Profiles • Production Team CardioNerds Cardiac Critical Care PageCardioNerds Episode PageCardioNerds AcademyCardionerds Healy Honor Roll CardioNerds Journal ClubSubscribe to The Heartbeat Newsletter!Check out CardioNerds SWAG!Become a CardioNerds Patron! Pearls and Quotes - Cardiogenic Shock and Valvular Heart Disease Shock due to valve disease is the result of a structural abnormality that may be temporized with medical therapy and circulatory support devices. However, it is ultimately best treated with a structural solution in the form of either percutaneous valvular therapies or cardiac surgery.When treating a patient with cardiogenic shock with normal or hyperdynamic ventricular function, we should keep a high index of suspicion for valvular disease. The cardiac output may be reduced due to a stenotic lesion “blocking” forward flow or regurgitant lesion causing backward flow.Acute mitral and aortic regurgitation will typically not manifest as a loud murmur on physical exam. The combination of hypotension and rapid flow of regurgitant blood on an “unprepared” cardiac chamber results in rapid equalization of chamber pressures, shortening the intensity and duration of the murmur. On transthoracic echocardiogram, for instance with acute MR, color Doppler may not show a large turbulent jet, and thus the MR may be underestimated or not appreciated at all.Echocardiography is critical to understand the etiology and severity of valvular shock, and invasive hemodynamics are often needed to guide medical and mechanical interventions.In multi-valve disease with severe aortic stenosis and functional mitral regurgitation, we typically treat the aortic stenosis first, since the mitral regurgitation may improve from the reduction in afterload associated with treating aortic stenosis. Show notes - Cardiogenic Shock and Valvular Heart Disease 1. Shock due to valve disease arises due to a structural problem that may be temporized with medical therapy and circulatory support devices, but is ultimately best treated with a structural solution in the form of either percutaneous valvular therapies or cardiac surgery. Stabilizing therapies for acute mitral regurgitation include afterload reduction with vasodilators, diuresis as needed to reduce pulmonary edema, and mechanical circulatory support including intra-aortic balloon pumps.Therapies for acute aortic regurgitation are typically more limited and include vasopressors such as epinephrine.  Bradycardia should be avoided with agents such as dobutamine or temporary pacing to reduce time in diastole. Temporary mechanical circulatory support options are limited in the setting of acute AR, though case reports of techniques such as LAVA ECMO (left atrial venoarterial extracorpeal membr...
01 Apr 2022194. Lipids: Omega-3 Fatty Acids & The Battle Of The Oils with Dr. Pam Taub00:39:55
CardioNerds, Amit Goyal, Dr. Tommy Das (Program Director of the CardioNerds Academy and Cardiology fellow at Cleveland Clinic), Dr. Rick Ferraro (Director of CardioNerds Journal Club and Cardiology fellow at the Johns Hopkins Hospital), Dr. Patrick Zakka (CardioNerds Academy Chief fellow of House Jones and Cardiology fellow at UCLA) discuss omega-3 fatty acids & the battle of the oils with Dr. Pam Taub, Director of Step Family Foundation Cardiovascular Rehabilitation and Wellness Center and Professor of Medicine at UC San Diego. Learn all about the different types of omega-3 fatty acids and the differences between prescription omega-3 fatty acids and dietary supplement fish oils. Audio editing by CardioNerds Academy Intern, Shivani Reddy. This episode is part of the CardioNerds Lipids Series which is a comprehensive series lead by co-chairs Dr. Rick Ferraro and Dr. Tommy Das and is developed in collaboration with the American Society For Preventive Cardiology (ASPC). Relevant disclosures: None Pearls • Notes • References • Guest Profiles • Production Team CardioNerds Cardiovascular Prevention PageCardioNerds Episode PageCardioNerds AcademyCardionerds Healy Honor Roll CardioNerds Journal ClubSubscribe to The Heartbeat Newsletter!Check out CardioNerds SWAG!Become a CardioNerds Patron! Pearls - Omega-3 Fatty Acids & The Battle Of The Oils Coming soon! Show notes - Omega-3 Fatty Acids & The Battle Of The Oils Coming soon! References - Omega-3 Fatty Acids & The Battle Of The Oils Arnett DK, Blumenthal RS, Albert MA, et al. 2019 ACC/AHA Guideline on the Primary Prevention of Cardiovascular Disease: A Report of the American College of Cardiology/American Heart Association Task Force on Clinical Practice Guidelines [published correction appears in Circulation. 2019 Sep 10;140(11):e649-e650] [published correction appears in Circulation. 2020 Jan 28;141(4):e60] [published correction appears in Circulation. 2020 Apr 21;141(16):e774].Grundy SM, Stone NJ, Bailey AL, et al. 2018 AHA/ACC/AACVPR/AAPA/ABC/ACPM/ADA/AGS/APhA/ASPC/NLA/PCNA Guideline on the Management of Blood Cholesterol: A Report of the American College of Cardiology/American Heart Association Task Force on Clinical Practice Guidelines [published correction appears in Circulation. 2019 Jun 18;139(25):e1182-e1186]. Circulation. 2019;139(25):e1082-e1143.Authors/Task Force Members; ESC Committee for Practice Guidelines (CPG); ESC National Cardiac Societies. 2019 ESC/EAS guidelines for the management of dyslipidaemias: Lipid modification to reduce cardiovascular risk [published correction appears in Atherosclerosis. 2020 Jan;292:160-162] [published correction appears in Atherosclerosis. 2020 Feb;294:80-82]. Atherosclerosis. 2019;290:140-205.Bhatt D, Steg P, Miller M et al., 2019. Cardiovascular Risk Reduction with Icosapent Ethyl for Hypertriglyceridemia. The New England journal of medicine, 380(1), pp.11–22.Budoff M, Bhatt D, Kinninger A et al. Effect of icosapent ethyl on progression of coronary atherosclerosis in patients with elevated triglycerides on statin therapy: final results of the EVAPORATE trial. Eur Heart J. 2020;41(40):3925-3932.Nicholls S, Lincoff A, Garcia M et al. Effect of High-Dose Omega-3 Fatty Acids vs Corn Oil on Major Adverse Cardiovascular Events in Patients at High Cardiovascular Risk: The STRENGTH Randomized Clinical Trial. JAMA. 2020;324(22):2268-2280. Guest Profiles Dr. Pam Taub Dr. Pam Taub, Professor of Medicine, is the founding director of the StepFamily Foundation Cardiac Rehabilitation and Wellness Center at the University of California, San Diego. Dr. Taub is a leader in preventive cardiology and has authored over one hundred publications, abstracts and book chapters. Dr. Taub is a leader in multiple professional societies, including board membership for the American Society of Preventive Cardiology. Dr. Patrick Zakka Dr. Patrick Zakka completed his medical school at th...
06 Apr 2022195. 1st Sanjay V Desai Lecture: Growth Mindset, Power of Yet, & Pursuit of Mastery with Dr. Keri Shafer and Dr. David Hirsh01:15:55
The CardioNerds Academy Class of 2021 graduation ceremony kicked off the inaugural Sanjay V Desai Lecture: Growth Mindset, Power of Yet, & Pursuit of Mastery. Join us as Dr. Tommy Das (CardioNerds Academy Program Director), and Dr. Saman Nematollahi (CardioNerds Academy Director of Research) discuss Growth Mindset with Dr. Keri Shafer and Dr. David Hirsh. Terrific acting by Dr. Patrick Zakka, Dr. Teodora Donisan, Dr. Ahmed Ghoneem, and Dr. Jessie Holtzman. Dr. Sanjay V Desai serves as the Chief Academic Officer, The American Medical Association and is the former Program Director of the Osler Medical Residency at The Johns Hopkins Hospital. Dr. Keri Shafer is an adult congenital heart disease specialist at Boston Children’s Hospital, and an assistant professor of pediatrics within Harvard Medical School. She completed internal medicine residency at Beth Israel Deaconess Medical Center, before completing cardiology fellowship at UT Southwestern and Adult Congenital and Pulmonary Hypertension subspecialty training at Boston Children’s and BWH.    Dr. David Hirsh is an associate professor of Medicine within Harvard Medical School, as well as the director of the HMS Academy fellowship in medical education and the associate dean of undergraduate medical education. Relevant disclosures: None CardioNerds Episode PageCardioNerds AcademyCardionerds Healy Honor Roll CardioNerds Journal ClubSubscribe to The Heartbeat Newsletter!Check out CardioNerds SWAG!Become a CardioNerds Patron!
24 Apr 2022196. Case Report: What I C, I Remember: A Case of Acute Heart Failure – Lahey Hospital and Medical Center00:59:12
CardioNerds (Amit Goyal and Daniel Ambinder) join Dr. Sonu Abraham (Cardiology fellow, Lahey Hospital and Medical Center), Dr. Amitoj Singh (Internal Medicine Resident, Lahey Hospital and Medical Center), Dr. Ahmed Ghoneem (Internal Medicine Resident, Lahey Hospital and Medical Center, CardioNerds Academy Chief) and Dr. Aanika Balaji (Internal Medicine Resident, Johns Hopkins) for a scrumptious meal on the Boston Harbor as they discuss a case of a young woman with metastatic melanoma on immune checkpoint inhibitors presenting with dyspnea. The presentation, risk factors, work up and management of patients with immune checkpoint inhibitor induced myocarditis are described. The E-CPR segment is provided by Dr. Sarju Ganatra, the founding director of the cardio-oncology program at Lahey Clinic.  CardioNerds Clinical Trialist Dr. Carrie Mahurin (University of Vermont Medical Center) is introduced at the beginning of the episode. A 41-year-old woman presented with mild dyspnea on exertion and non-productive cough. She had a history of Hashimoto thyroiditis, nodular thyroid s/p resection on levothyroxine, and metastatic melanoma on immune checkpoint inhibitor therapy with ipilimumab and nivolumab. She also had a history of obesity and underwent gastric bypass surgery several years prior. Though she lost weight after the surgery, she regained a significant amount and was 244 lbs with a BMI of 42. Her exam findings were remarkable for tachycardia, bilateral pulmonary rales, elevated JVP, and symmetric pedal edema. Investigations revealed a mild troponin elevation, non-specific EKG changes, and TTE with severely reduced left ventricular function (EF 15%) and a low GLS. Cardiac MRI showed patchy delayed myocardial enhancement in a non-ischemic distribution with marked global hypokinesis and EF of 11%. Endomyocardial biopsy confirmed the diagnosis of immune checkpoint inhibitor (ICI) associated myocarditis. The ICI therapy was discontinued and she was treated with high dose intravenous corticosteroids followed by a prolonged oral steroid taper with clinical improvement and complete recovery of left ventricular function. Jump to: Case media - Case teaching - References CardioNerds Case Reports PageCardioNerds Episode PageCardioNerds AcademyCardionerds Healy Honor Roll CardioNerds Journal ClubSubscribe to The Heartbeat Newsletter!Check out CardioNerds SWAG!Become a CardioNerds Patron! Case Media - immune checkpoint inhibitor myocarditis Episode Schematics & Teaching CardioNerds Myocarditis, updated 1.20.21 Pearls - immune checkpoint inhibitor myocarditis ICI-associated myocarditis has a high mortality rate necessitating a high degree of clinical suspicion. When in doubt, check it out! The initial 4 diagnostic pillars include EKG, troponin, BNP and TTE. Cardiac MRI and endomyocardial biopsy help to confirm the diagnosis.Left ventricular function is normal in 50% of these patients with ICI-associated myocarditis, so the ejection fraction is not a sensitive test for ruling this out.Endomyocardial biopsy should be considered in patients with a high clinical suspicion but negative or ambiguous non-invasive imaging.Early initiation of corticosteroids within 24 hours of presentation is associated with better outcomes.ICIs should be discontinued indefinitely in those with Grade 3 or 4 disease. Notes - immune checkpoint inhibitor myocarditis 1. Immune checkpoint inhibitors – What are they and why should we as cardiologists know about them? Immune checkpoint inhibitors (ICI) boost the host immune response against tumor cells by inhibiting the intrinsic brakes of the immune response.There are currently 7 FDA approved drugs in this group: one CTLA-4-blocking antibody called ipilimumab; three PD-1-blocking antibodies [nivolumab, pembrolizumab, and cemiplimab]; and three PD-L1-blocking antibodies [atezolizumab, avelumab, and durvalumab].Like a car,
27 Apr 2022197. Narratives in Cardiology: Empowering the LGBTQIA+ Community of Cardiovascular Patients & Professionals with Dr. Stephen Cook & Dr. Katie Berlacher – Indiana Chapter00:58:39
CardioNerds (Amit Goyal and Daniel Ambinder), join Dr. Kara Denby (Interventional cardiology fellow, Cleveland Clinic), Dr. Tony Pastor (ACHD fellow, Harvard Medical School), Dr. Katie Berlacher (Cardiology program director, UPMC), and Dr. Stephen Cook (ACHD cardiologist, Indiana University) to discuss empowering the LGBTQIA+ community of cardiovascular patients & professionals and more in this installment of the Narratives in Cardiology Series. This episode features the Indiana ACC Chapter. Episode introduction and audio editing by CardioNerds Academy Intern, Pace Wetstein. This discussion was inspired by this perspective piece on ACC.org titled: Finding Our Voices: Building an LGBTQIA+ Community Within Cardiology. To learn more about diversity and equity among the LGBTQIA+ population, check out this webinar organized by the ACC. The PA-ACC & CardioNerds Narratives in Cardiology is a multimedia educational series jointly developed by the Pennsylvania Chapter ACC, the ACC Fellows in Training Section, and the CardioNerds Platform with the goal to promote diversity, equity, and inclusion in cardiology. In this series, we host inspiring faculty and fellows from various ACC chapters to discuss their areas of expertise and their individual narratives. Join us for these captivating conversations as we celebrate our differences and share our joy for practicing cardiovascular medicine. We thank our project mentors Dr. Katie Berlacher and Dr. Nosheen Reza. Video Version • Notes • Production Team Claim free CME just for enjoying this episode! There are no relevant disclosures for this episode. The PA-ACC & CardioNerds Narratives in Cardiology PageCardioNerds Episode PageCardioNerds AcademyCardionerds Healy Honor Roll CardioNerds Journal ClubSubscribe to The Heartbeat Newsletter!Check out CardioNerds SWAG!Become a CardioNerds Patron! Tweetorial - Empowering the LGBTQIA+ Community of Cardiovascular Patients & Professionals https://twitter.com/Gurleen_Kaur96/status/1526334939830034432?s=20&t=wMk75ORn1_KJtMTOY1IAdw Video version - Empowering the LGBTQIA+ Community of Cardiovascular Patients & Professionals Coming soon Production Team Dr. Gurleen Kaur Amit Goyal, MD Daniel Ambinder, MD
03 May 2022198. ACHD: Cardiovascular Multimodality Imaging in Congenital Heart Disease with Dr. Eric Krieger00:59:56
CardioNerds (Daniel Ambinder), ACHD series co-chairs,  Dr. Josh Saef (ACHD fellow, University of Pennsylvania) Dr. Daniel Clark (ACHD fellow, Vanderbilt University), and ACHD FIT lead Dr. Jon Kochav (Columbia University) join Dr. Eric Krieger (Director of the Seattle Adult Congenital Heart Service and the ACHD Fellowship, University of Washington) to discuss multimodality imaging in congenital heart disease. Audio editing by CardioNerds Academy Intern, Dr. Maryam Barkhordarian. Special introduction to CardioNerds Clinical Trialist Dr. Shiva Patlolla (Baylor University Medical Center). In this episode we discuss the strengths and weaknesses of the imaging modalities most commonly utilized in the diagnosis and surveillance of patients with ACHD.  Specifically, we discuss transthoracic and transesophageal echocardiography, cardiac MRI and cardiac CT. The principles learned are then applied to the evaluation of two patient cases – a patient status post tetralogy of Fallot repair with a transannular patch, and a patient presenting with right ventricular enlargement of undetermined etiology. The CardioNerds Adult Congenital Heart Disease (ACHD) series provides a comprehensive curriculum to dive deep into the labyrinthine world of congenital heart disease with the aim of empowering every CardioNerd to help improve the lives of people living with congenital heart disease. This series is multi-institutional collaborative project made possible by contributions of stellar fellow leads and expert faculty from several programs, led by series co-chairs, Dr. Josh Saef, Dr. Agnes Koczo, and Dr. Dan Clark. The CardioNerds Adult Congenital Heart Disease Series is developed in collaboration with the Adult Congenital Heart Association, The CHiP Network, and Heart University. See more Disclosures: None Pearls • Notes • References • Guest Profiles • Production Team CardioNerds Adult Congenital Heart Disease PageCardioNerds Episode PageCardioNerds AcademyCardionerds Healy Honor Roll CardioNerds Journal ClubSubscribe to The Heartbeat Newsletter!Check out CardioNerds SWAG!Become a CardioNerds Patron! Pearls - Cardiovascular Multimodality Imaging in Congenital Heart Disease Transthoracic echocardiography (TTE) is the first line diagnostic test for the diagnosis and surveillance of congenital heart disease due to widespread availability, near absent contraindications, and ability to perform near comprehensive structural, functional, and hemodynamic assessments in patients for whom imaging windows allow visualization of anatomic areas of interest.Transesophageal echocardiography (TEE) use in ACHD patients is primarily focused on similar indications as in acquired cardiovascular disease patients: the assessment of endocarditis, valvular regurgitation/stenosis severity and mechanism, assessment of interatrial communications in the context of stroke, evaluation for left atrial appendage thrombus, and for intraprocedural guidance. When CT or MRI are unavailable or contraindicated, TEE can also be used when transthoracic imaging windows are poor, or when posterior structures (e.g. sinus venosus, atrial baffle) need to be better evaluated.Cardiac MRI (CMR) with MR angiography imaging is unencumbered by imaging planes or body habitus and can provide comprehensive high resolution structural and functional imaging of most cardiac and extracardiac structures. Additional key advantages over echocardiography are ability to reproducibly quantify chamber volumes, flow through a region of interest (helpful for quantifying regurgitation or shunt fraction), assess for focal fibrosis via late gadolinium enhancement imaging, and assess the right heart.Cardiac CT has superior spatial resolution in a 3D field of view which makes it useful for clarifying anatomic relationships between structures, visualizing small vessels such as coronary arteries or collateral vessels, and assessing patency of larger vessels (e.
04 May 2022199. Case Report: The Perfect Storm of Complications Post-Partum – Summa Health00:40:20
CardioNerds (Amit Goyal and Daniel Ambinder) join Dr. Phoo Pwint Nandar (former FIT Ambassador), Dr. Deep Shah (current FIT Ambassador), and Dr. Sugat Wagle from the Summa Health Cardiology Department for an afternoon at Cuyahoga National Valley Park. We discuss a case of a post-partum woman who presented with ventricular fibrillation arrest due to SCAD. She had ongoing advanced cardiac life support (ACLS) for nearly 60 minutes before obtaining return of spontaneous circulation. We discuss the broad differential of VF arrest, including acute coronary syndrome and spontaneous coronary artery dissection (SCAD) – among many others. We also go over the etiology and management of SCAD as well the complications. Pregnancy is a crucial stressor to the cardiovascular system and understanding its hemodynamic changes is crucial to all physicians. The E-CPR segment is provided by Dr. Grace Ayafor, Interventional cardiology faculty, Summa Health. Jump to: Case media - Case teaching - References CardioNerds Case Reports PageCardioNerds Episode PageCardioNerds AcademyCardionerds Healy Honor Roll CardioNerds Journal ClubSubscribe to The Heartbeat Newsletter!Check out CardioNerds SWAG!Become a CardioNerds Patron! Case Media Episode Schematics & Teaching - SCAD Pearls - SCAD SCAD remains underdiagnosed. It has a wide range of clinical presentations, including chest pain, ACS, VT/VF arrest, and cardiogenic shock.Underlying etiologies of SCAD include autoimmune diseases, connective tissue disorders, fibromuscular dysplasia, external stressors, and pregnancy.There are 3 types of SCAD, and coronary angiogram is the gold standard for diagnosis.Common areas of involvement of SCAD include left anterior descending and left circumflex arteries; however, SCAD can manifest in any coronary artery as well as simultaneously in multiple coronary arteries. Left main trunk involvement is rare, more likely to be associated with the peri-partum state, and requires complex management decisions guided by a heart team approach.Most SCAD cases are benign and treated conservatively, however, some require intervention (PCI or CABG) depending on clinical severity and course.Recurrent SCAD has been reported in 10-30% of the patients and aggressive management of hypertension is recommended.Guidelines regarding SCAD management are largely based on expert consensus due to a dearth of high-quality data. Efforts to raise awareness and study this syndrome are of paramount importance. Notes - SCAD 1. What is SCAD and how does it present? Spontaneous coronary artery dissection (SCAD) is defined as an epicardial coronary dissection that is not associated with atherosclerosis or instrumentation.This occurs with hematoma formation within the tunica media,  thereby potentially compressing the arterial true lumen leading to ischemia.There are two proposed mechanisms of hematoma formation: “inside-out” and “outside-in”. The inside-out hypothesis posits that the hematoma arises from the true lumen via a dissection flap – an endothelial-intimal disruption. Conversely the outside-in hypothesis posits that the hematoma arises de novo within the media through disruption of traversing microvessels.There is a wide range of clinical presentation for SCAD varying in severity including asymptomatic / benign presentation, anginal syndromes, acute myocardial infarction, VT/VF arrest, and cardiogenic shock. Our patient presented with VF arrest and ACS.SCAD epidemiology is confounded by a lack of awareness. A high index of suspicion is warranted. Diagnosis can be missed in young or mid-life without CV risk factors who would present with atypical/mild chest pain. 2. What are the etiologies of SCAD? SCAD is associated with the peripartum state (presumed due to combination of hormonal mediated vessel wall integrity changes and hemodynamic stressors), illicit substance use, autoimmune disorders,
06 May 2022200. 2022 AHA/ACC/HFSA Guideline for The Management of Heart Failure – Hot Takes from The Journal of Cardiac Failure Family00:45:33
CardioNerds (Amit Goyal, Daniel Ambinder) and special co-host Dr. Mark Belkin, join the Journal of Cardiac Failure Family to discuss the 2022 AHA/ACC/HFSA Guideline for The Management of Heart Failure. The JCF Editor-In-Chief Dr. Robert Mentz, Deputy Editor Dr. Anu Lala, and FIT editors -- Dr. Vanessa Bluemer, Dr. Ashish Corrhea, and Dr. Quinton Youmans -- share their hot takes and practical takeaways from the guidelines. At JCF, we’re privileged to share this important document that will support improved care for those living with heart failure,” stated Editor-in Chief Dr. Robert J. Mentz and Deputy Editor Anu Lala. “The 2022 guidelines convey patient-centered updates regarding the language we use to communicate disease considerations (e.g., stages of HF) and practice-changing guidance around the diagnosis and management of HF including newer therapeutics (e.g., SGLT2i). There is an emphasis not only on managing HF but also on how to treat important comorbidities as part of the holistic care for patients living with HF." 2022 AHA/ACC/HFSA Guideline for the Management of Heart Failure Executive Summary A Clinician's Guide to the 2022 ACC/AHA/HFSA Guideline for the Management of Heart Failure by Dr. Michelle Kittleson CardioNerds Heart Success Series PageCardioNerds Episode PageCardioNerds AcademyCardionerds Healy Honor Roll CardioNerds Journal ClubSubscribe to The Heartbeat Newsletter!Check out CardioNerds SWAG!Become a CardioNerds Patron! Guideline Top 10 Take-Home Messages - Guideline for The Management of Heart Failure 1. Guideline-directed medical therapy (GDMT) for heart failure (HF) with reduced ejection fraction (HFrEF) now includes 4 medication classes that include sodium-glucose cotransporter-2 inhibitors (SGLT2i). 2. SGLT2i have a Class of Recommendation 2a in HF with mildly reduced ejection fraction (HFmrEF). Weaker recommendations (Class of Recommendation 2b) are made for ARNi, ACEi, ARB, MRA, and beta blockers in this population. 3. New recommendations for HFpEF are made for SGLT2i (Class of Recommendation 2a), MRAs (Class of Recommendation 2b), and ARNi (Class of Recommendation 2b). Several prior recommendations have been renewed including treatment of hypertension (Class of Recommendation 1), treatment of atrial fibrillation (Class of Recommendation 2a), use of ARBs (Class of Recommendation 2b), and avoidance of routine use of nitrates or phosphodiesterase-5 inhibitors (Class of Recommendation 3: No Benefit). 4. Improved LVEF is used to refer to those patients with previous HFrEF who now have an LVEF >40%. These patients should continue their HFrEF treatment. 5.Value statements were created for select recommendations where high-quality, cost-effectiveness studies of the intervention have been published. 6. Amyloid heart disease has new recommendations for treatment including screening for serum and urine monoclonal light chains, bone scintigraphy, genetic sequencing, tetramer stabilizer therapy, and anticoagulation. 7. Evidence supporting increased filling pressures is important for the diagnosis of HF if the LVEF is >40%. Evidence for increased filling pressures can be obtained from noninvasive (e.g., natriuretic peptide, diastolic function on imaging) or invasive testing (e.g., hemodynamic measurement). 8. Patients with advanced HF who wish to prolong survival should be referred to a team specializing in HF. A HF specialty team reviews HF management, assesses suitability for advanced HF therapies, and uses palliative care including palliative inotropes where consistent with the patient's goals of care. 9. Primary prevention is important for those at risk for HF (stage A) or pre-HF (stage B). Stages of HF were revised to emphasize the new terminologies of “at risk” for HF for stage A and pre-HF for stage B. 10.Recommendations are provided for select patients with HF and iron deficiency, anemia, hypertension, sleep disorders,
29 Dec 20194. Practical Approach to Hypertrophic Cardiomyopathy with Dr. Jose Madrazo00:26:49
Clinical and echo expert Dr. Jose Madrazo discusses a high yield and practical approach to diagnosis and management of hypertrophic cardiomyopathy. Show notes and images: https://www.cardionerds.com/hypertrophic-cardiomyopathy/
08 May 2022201. Guidelines: 2021 ESC Cardiovascular Prevention – Question #7 with Dr. Wesley Milks00:10:18
The following question refers to Section 3.4 of the 2021 ESC CV Prevention Guidelines. The question is asked by student Dr. Adriana Mares, answered first by early career preventive cardiologist Dr. Dipika Gopal, and then by expert faculty Dr. Michael Wesley Milks. Dr. Milks is a staff cardiologist and assistant professor of clinical medicine at the Ohio State University Wexner Medical Center where he serves as the Director of Cardiac Rehabilitation and an associate program director of the cardiovascular fellowship. He specializes in preventive cardiology and is a member of the American College of Cardiology's Cardiovascular Disease Prevention Leadership Council. The CardioNerds Decipher The Guidelines Series for the 2021 ESC CV Prevention Guidelines represents a collaboration with the ACC Prevention of CVD Section, the National Lipid Association, and Preventive Cardiovascular Nurses Association. Question #7 While you are on holiday break visiting your family, your aunt pulls you aside during the family gathering to ask a few questions about your 70-year-old uncle. He has hypertension, hyperlipidemia, type 2 diabetes mellitus, and moderate chronic obstructive pulmonary disease. His medications include Fluticasone/Salmeterol, Tiotropium, Albuterol, Lisinopril, Simvastatin, and Metformin. She is very concerned about his risk for heart disease as he has never had his “heart checked out.” She asks if the presence of COPD increases his chance of having heart disease. Which of the following statements would best answer her question? A. Systemic inflammation and oxidative stress caused by COPD promote vascular remodeling and a paradoxical ‘anticoagulant’ state affecting all vasculature types. B. Although chronic COPD is associated with increased cardiovascular events, individual exacerbations have no impact on risk of cardiovascular events. C. Patients with mild-moderate COPD are 8-10x more likely to die from atherosclerotic cardiovascular disease than respiratory failure. D. Cardiovascular mortality increases proportionally with an increase in forced expiratory volume in 1 second (FEV1) Answer #7 The correct answer is C. Patients with mild-moderate COPD are 8-10x more likely to die from atherosclerotic cardiovascular disease than respiratory failure. Patients with COPD have a 2-3-fold increased risk of CV events compared to age-matched controls even when adjusted for tobacco smoking, a shared risk factor. This can be partly explained by other common risk factors including aging, hypertension, hyperlipidemia, and low physical activity. Interestingly, CVD mortality increases proportionally with a decrease (rather than increase) in FEV1, making answer choice D wrong (28% increase CVD mortality for every 10% decrease in FEV1). Additionally, COPD exacerbations and related infections are associated with a 4x increase in CVD events, making answer choice B incorrect. COPD has several effects on the vasculature which creates a ‘procoagulant’ not ‘anticoagulant’ effect on all vascular beds. This is associated with increased risk of cognitive impairment due to cerebral microvascular damage as well as increased risk of ischemic and hemorrhagic stroke. Main Takeaway The presence of COPD (even mild to moderate) has a significant impact on the incidence of non-fatal coronary events, stroke, and cardiovascular mortality mediated by inherent disease process and progression, risk factors (smoking, aging, hypertension, and hyperlipidemia), and systemic inflammation altering vasculature creating a ‘procoagulant’ effect. The ESC gives a Class I indication (LOE C) to investigate for ASCVD and ASCVD risk factors in patients with COPD. Guideline Location 3.4.5, Page 3264. CardioNerds Decipher the Guidelines - 2021 ESC Prevention Series CardioNerds Episode Page CardioNerds Academy Cardionerds Healy Honor Roll CardioNerds Journal Club Subscribe to The Heartbeat Newsletter! Check out CardioNerds SWAG! Become a CardioNerds Patron!
09 May 2022202. Guidelines: 2021 ESC Cardiovascular Prevention – Question #8 with Dr. Eugene Yang00:09:19
This question refers to Sections 3.1 of the 2021 ESC CV Prevention Guidelines. The question is asked by CardioNerds Academy Intern, student Dr. Hirsh Elhence, answered first by internal medicine resident at Beaumont Hospital and soon to be Mayo Clinic cardiology fellow and Dr. Teodora Donisan and then by expert faculty Dr. Eugene Yang.Dr. Yang is professor of medicine of the University of Washington where he is medical director of the Eastside Specialty Center and the co-Director of the Cardiovascular Wellness and Prevention Program. Dr. Yang is former Governor of the ACC Washington Chapter and current chair of the ACC Prevention of CVD Section.The CardioNerds Decipher The Guidelines Series for the 2021 ESC CV Prevention Guidelines represents a collaboration with the ACC Prevention of CVD Section, the National Lipid Association, and Preventive Cardiovascular Nurses Association. Question #8 Please read the following patient vignettes and choose the FALSE statement.A. A 39-year-old man who comes for a regular physical, has normal vitals and weight, denies any significant past medical or family history – does not need systematic cardiovascular disease (CVD) assessment.B. A 39-year-old woman who comes for a regular physical, has normal vitals and weight, and has a history of radical hysterectomy (no other significant past medical or family history) – could benefit from systematic or opportunistic CVD assessment.C. A 39-year-old woman who comes for a regular physical, has normal vitals except for a BMI of 27 kg/m2 and a family history of hypertension – requires a systematic global CVD assessment.D. A 39-year-old man who comes for a regular physical, has normal vitals and weight, and has a personal history of type I diabetes – requires a systematic global CVD assessment. Answer #8 The correct answer is C.Option A is an accurate statement, as systematic CVD risk assessment is not recommended in men < 40 years-old and women < 50 years-old, if they have no known cardiovascular (CV) risk factors. (Class III, level C)Option B is an accurate statement, as this patient had a radical hysterectomy, which means the ovaries have been removed as well and she is considered postmenopausal. Systematic or opportunistic CV risk assessment can be considered in men > 40 years-old and women > 50 years-old or postmenopausal, even in the absence of known ASCVD risk factors. (Class IIb, level C)Option C is a false statement and thus the correct answer, as the recommendations for global screening in this patient are not as strong and would require shared decision making. Opportunistic screening of blood pressure can be considered in her, as she is at risk for developing hypertension. Blood pressure screening should be considered in adults at risk for the development of hypertension, such as those who are overweight or with a known family history of hypertension. (Class IIa, level B)Option D is an accurate statement, as systematic global CVD risk assessment is recommended in individuals with any major vascular risk factor (i.e., family history of premature CVD, familial hyperlipidemia, CVD risk factors such as smoking, arterial hypertension, DM, raised lipid level, obesity, or comorbidities increasing CVD risk). (Class I, level C)Additional learning points:Do you know the difference between opportunistic and systematic CVD screening?Opportunistic screening refers to screening without a predefined strategy when the patient presents for different reasons. This is an effective and recommended way to screen for ASCVD risk factors, although it is unclear if it leads to benefits in clinical outcomes.Systematic screening can be done following a clear strategy formally evaluating either the general population or targeted subpopulations (i.e., type 2 diabetics or patients with significant family history of CVD). Systematic screening results in improvements in risk factors but has no proven effect on CVD outcomes.
10 May 2022203. Guidelines: 2021 ESC Cardiovascular Prevention – Question #9 with Dr. Noreen Nazir00:12:03
The following question refers to Section 4.3 of the 2021 ESC CV Prevention Guidelines. The question is asked by Dr. Maryam Barkhordarian, answered first by pharmacy resident Dr. Anushka Tandon, and then by expert faculty Dr. Noreen Nazir. Dr. Noreen Nazir is Assistant Professor of Clinical Medicine at the University of Illinois at Chicago, where she is the director of cardiac MRI and the preventive cardiology program. The CardioNerds Decipher The Guidelines Series for the 2021 ESC CV Prevention Guidelines represents a collaboration with the ACC Prevention of CVD Section, the National Lipid Association, and Preventive Cardiovascular Nurses Association. Question #9 Mr. A is a 28-year-old man who works as an accountant in what he describes as a “desk job” setting. He shares that life got “a little off-track” for him in 2020 between the COVID-19 pandemic and a knee injury. His 2022 New Years’ resolution is to improve his overall cardiovascular and physical health. He has hypertension and a family history of premature ASCVD in his father, who died of a heart attack at age 50. Prior to his knee injury, he went to the gym 3 days a week for 1 hour at a time, split between running on the treadmill and weightlifting. He has not returned to the gym since his injury and has been largely sedentary, although he is trying to incorporate a 20-minute daily walk into his routine. Which of the following exercise-related recommendations is most appropriate? A. A target of 75-150 minutes of vigorous-intensity or 150-300 minutes of moderate-intensity aerobic physical exercise weekly is recommended to reduce all-cause mortality, CV mortality, and morbidity. B. Bouts of exercise less than 30 minutes are not associated with favorable health outcomes. C. Exercise efforts should be focused on aerobic activity, since only this type of activity is associated with mortality and morbidity benefits. D. Light-intensity aerobic activity like walking is expected to have limited health benefits for persons with predominantly sedentary behavior at baseline. Answer #9 The correct answer is A. There is an inverse relationship between moderate-to-vigorous physical activity and CV morbidity/mortality, all-cause mortality, and incidence of type 2 diabetes, with additional benefits accrued for exercise beyond the minimum suggested levels. The recommendation to “strive for at least 150-300 min/week of moderate-intensity, or 75-150 min/week of vigorous-intensity aerobic physical activity, or an equivalent combination thereof” is a Class 1 recommendation per the 2021 ESC guidelines, and a very similar recommendation (at least 75 minutes of vigorous-intensity or 150 minutes of moderate-intensity activity) is also Class 1 recommendation per 2019 ACC/AHA primary prevention guidelines. Both the ESC and ACC/AHA provide examples of activities grouped by absolute intensity (the amount of energy expended per minute of activity), but the ESC guidelines also offer suggestions for measuring the relative intensity of an activity (maximum/peak associated effort) in Table 7, which allows for a more individualized, customizable approach to setting activity goals. Importantly, individuals who are unable to meet minimum weekly activity recommendations should still be encouraged to stay as active as their abilities and health conditions allow to optimize cardiovascular and overall health. Choice B is incorrect, as data suggests physical activity episodes of any duration, including <10 min, are associated with favorable outcomes like all-cause mortality benefit. The duration of a single exercise bout is less correlated with health benefits than the total physical activity time accumulated per week. Choice C is incorrect. Per the ESC guidelines, it is a class 1 recommendation to perform resistance exercise, in addition to aerobic activity, on 2 or more days per week to reduce all-cause mortality. Data indicate that the addition of resistance exercise to aerobic activity is assoc...
11 May 2022204. Guidelines: 2021 ESC Cardiovascular Prevention – Question #10 with Dr. Eileen Handberg00:09:05
The following question refers to Section 4.6 of the  2021 ESC CV Prevention Guidelines. The question is asked by student Dr. Shivani Reddy, answered first by NP Carol Patrick, and then by expert faculty Dr. Eileen Handberg. Dr. Handberg is an Adult Nurse Practitioner, Professor of Medicine, and Director of the Cardiovascular Clinical Trials Program in the Division of Cardiovascular Medicine at the University of Florida. She has served as Chair of the Cardiovascular Team Section and the Board of Trustees with the ACC and is the President Elect for the PCNA. The CardioNerds Decipher The Guidelines Series for the 2021 ESC CV Prevention Guidelines represents a collaboration with the ACC Prevention of CVD Section, the National Lipid Association, and Preventive Cardiovascular Nurses Association. Question #10 Ms. DW is a 67-year-old woman with a history of coronary artery disease and prior percutaneous coronary intervention in 2019 with a drug-eluting stent to the proximal left anterior descending artery. They have transitioned to your clinic from a previous provider, and their LDL is 134 mg/dL. What would be the ESC recommended goal LDL-C level for this patient?  A. <30 mg/dL B. <55 mg/dL C. <70 mg/dL D. <100 mg/dL Answer #10 The correct answer is B. The ESC guidelines outline a robust LDL-C reduction goal of <55mg/dL (<1.4mmol/L) and ≥50% reduction from baseline in those with known atherosclerotic cardiovascular disease, with the highest possible Class I (LOE A) recommendation for this goal. A goal LDL-C <55mg/dL with ≥50% reduction from baseline should also be considered in apparently healthy persons <70 years of age who are at very high risk (Class IIa, LOE C).  To achieve these goals, the guidelines recommend a stepwise approach to treatment including dietary, lifestyle, and medical management. Recognizing that lower LDL-C is better, the guidelines recommend liberal intensification of treatment especially if using submaximal doses of generic or low-cost statins and side effects are not apparent. High-intensity statin is recommended to be prescribed to the highest tolerated dose to reach LDL-C goals set for each specific risk group (Class I, LOE A). If these goals are not achieved with the maximum tolerated dose of a statin, combination therapy with ezetimibe is recommended (Class I, LOE B). Choice A is incorrect. An LDL-C <30mg/dL is a more significant reduction than that recommended by the guidelines, even for patients with known ASCVD. Notably, for patients with ASCVD who experience a second vascular event within 2 years while taking maximum tolerated statin-based therapy, an LDL-C goal of <1.0 mmol/L (40 mg/dL) may be considered. Choice C is incorrect. The ESC prevention guidelines recommend considering a goal of <70mg/dL for patients in the primary prevention setting at high risk who are <70 years of age (Class IIa, LOE C). (Recall again that for those at very high risk the primary prevention recommendation is target LDL-C <55 mg/dL). Choice D is incorrect. LDL-C <100mg/dL was a frequently cited goal in older iterations of various prevention and lipid guidelines. As the data has shifted to support lower LDL-C goals, this is not a noted goal within the 2021 ESC prevention guidelines for patients <70 years of age. Main Takeaway Lower is better when it comes to LDL-C For those with known atherosclerotic cardiovascular disease, liberal intensification of lipid lowering treatment is recommended.  Guideline Location Section 4.6.2.1, page 3276-3279, Figure 6 on page 3252, Figure 7 on page 3253 CardioNerds Decipher the Guidelines - 2021 ESC Prevention Series CardioNerds Episode Page CardioNerds Academy Cardionerds Healy Honor Roll CardioNerds Journal Club Subscribe to The Heartbeat Newsletter! Check out CardioNerds SWAG! Become a CardioNerds Patron!
12 May 2022205. Guidelines: 2021 ESC Cardiovascular Prevention – Question #11 with Dr. Eugenia Gianos00:08:13
The following question refers to Section 6.1 of the 2021 ESC CV Prevention Guidelines. The question is asked by Dr. Christian Faaborg-Andersen, answered first by UCSD cardiology fellow Dr. Harpreet Bhatia, and then by expert faculty Dr. Eugenia Gianos. Dr. Gianos specializes in preventive cardiology, lipidology, cardiovascular imaging, and women’s heart disease; she is the director of the Women’s Heart Program at Lenox Hill Hospital and director of Cardiovascular Prevention for Northwell Health. The CardioNerds Decipher The Guidelines Series for the 2021 ESC CV Prevention Guidelines represents a collaboration with the ACC Prevention of CVD Section, the National Lipid Association, and Preventive Cardiovascular Nurses Association. Question #11 A 70-year-old man with a history of hypertension, diabetes, hyperlipidemia, peptic ulcer disease with a prior upper GI bleed, as well as coronary artery disease presents to clinic. About one year ago he suffered an NSTEMI treated with percutaneous coronary intervention to the mid LAD. He is feeling well and able to walk 1 mile daily with no anginal symptoms. He is currently taking aspirin 81 mg daily, ticagrelor 90 mg BID, atorvastatin 40 mg daily, metoprolol 25 mg BID, lisinopril 5 mg daily, and lansoprazole 15mg daily. He has a preserved ejection fraction. His BP in clinic is 110/70 and HR is 65 bpm. His LDL is 50 mg/dL. What do you recommend for his further management? A. Switch ticagrelor to clopidogrel, continue indefinitelyB. Stop ticagrelor, continue aspirin indefinitelyC. Continue aspirin + ticagrelor indefinitelyD. Stop ticagrelor, start rivaroxaban 2.5 mg BIDListen to the podcast episode! Answer #11 The correct answer is B – stop ticagrelor, continue aspirin indefinitely.Twelve months of DAPT is recommended for acute coronary syndromes (Class I, LOE A). Long-term secondary prevention with dual anti-thrombotic therapy (DAPT > 12 months with a P2Y12 inhibitor and low-dose aspirin or low-dose rivaroxaban 2.5mg BID with low-dose aspirin) may be considered for patients who are at high ischemic risk without high risk of bleeding (Class IIa, LOE A). However, this patient is at increased bleeding risk (peptic ulcer disease with prior GI bleeding) and has no ischemic symptoms, and so neither would be recommended.Main TakeawayIn summary, 12 months of DAPT is recommended after ACS. Prolonged DAPT or low-dose rivaroxaban may be considered with high ischemic risk and low bleeding risk.Guideline LocationSection 6.1, Pages 3294-3295. CardioNerds Decipher the Guidelines - 2021 ESC Prevention SeriesCardioNerds Episode PageCardioNerds AcademyCardionerds Healy Honor RollCardioNerds Journal ClubSubscribe to The Heartbeat Newsletter!Check out CardioNerds SWAG!Become a CardioNerds Patron!
13 May 2022206. Guidelines: 2021 ESC Cardiovascular Prevention – Question #12 with Dr. Laurence Sperling00:09:30
The following question refers to Section 4.11 of the 2021 ESC CV Prevention Guidelines. The question is asked by Dr. Christian Faaborg-Andersen, answered first by UCSF resident Dr. Jessie Holtzman, and then by expert faculty Dr. Laurence Sperling. Dr. Laurence Sperling is the Katz Professor in Preventive Cardiology at the Emory University School of Medicine and Founder of Preventive Cardiology at the Emory Clinic. Dr. Sperling was a member of the writing group for the 2018 Cholesterol Guidelines, serves as Co-Chair for the ACC's Cardiometabolic and Diabetes working group, and is Co-Chair of the WHF Roadmap for Cardiovascular Prevention in Diabetes. The CardioNerds Decipher The Guidelines Series for the 2021 ESC CV Prevention Guidelines represents a collaboration with the ACC Prevention of CVD Section, the National Lipid Association, and Preventive Cardiovascular Nurses Association. Question #12 Medically supervised cardiac rehabilitation programs after ASCVD events and for patient with heart failure carries a Class I recommendation. However, placement of referrals, uptake and enrollment after referral, and rigor of rehabilitation all remain inconsistent. What minimum cumulative duration of cardiac rehabilitation has been chosen as a threshold of effectiveness for cardiac rehabilitation by the European Society of Cardiology? A. 100-300 minutes, 10 sessions B. 300-500 minutes, 16 sessions C. 500-700 minutes, 22 sessions D. 700-1000 minutes, 28 sessions E. >1000 minutes, 36 sessions Answer #12 The correct answer is E: >1000 minutes across 36 sessions. Cardiac rehabilitation is a comprehensive, multidisciplinary intervention not just including exercise training and physical activity counselling, but also education, risk factor modification, diet/nutritional counselling, and vocational and psychosocial support. A broad evidence base demonstrates that multidisciplinary cardiac rehabilitation and prevention programs after ASCVD events or revascularization reduce recurrent cardiovascular hospitalizations, myocardial infarction, and cardiovascular mortality. In patients with chronic HF (mainly HFrEF), exercise based cardiac rehabilitation (EBCR) may improve all-cause mortality, reduce hospital admissions, and improve exercise capacity and quality of life. Such programs include a wide array of activities including physical activity, risk factor modification, psychosocial support, nutrition counseling, and more. Despite the heterogenous design of clinical trials, cardiac rehabilitation has been shown to be a cost-effective intervention. Based upon the available review data, the European Association of Preventive Cardiology and the European Society of Cardiology proposed minimum standards for secondary prevention cardiac rehabilitation programs. Based upon a comprehensive review of the literature, ESC recommends that cardiac rehabilitation be multidisciplinary, supervised by health professionals, and start as soon as possible after a cardiovascular event. Cardiac rehabilitation should include both aerobic and muscular resistance tailored to the fitness level of the participant, should carry a duration of >1000 minutes in total, and should exceed 36 sessions total. While uptake remains limited, electronic prompts within the medical record and automatic referrals should be considered to enhance referral and participation. Future research should continue to explore the benefit of home-based cardiac rehabilitation with or without telemonitoring. Lastly, studies have shown that uptake remains lower among women, and targeted programs should be undertaken to address such disparities. Main Takeaway Current European Society of Cardiology guidelines provide a Class I (LOE A) recommendation for the participation in multidisciplinary cardiac rehabilitation programs for the secondary prevention of ASCVD events including revascularization and in individuals with heart failure (mainly HFrEF) to improve patient outcomes.
16 May 2022207. Lipids: REDUCE-IT Versus STRENGTH Trials – EPA in Clinical Practice with Dr. Peter Toth00:53:59
CardioNerds Tommy Das (Program Director of the CardioNerds Academy and cardiology fellow at Cleveland Clinic), Rick Ferraro (cardiology fellow at the Johns Hopkins Hospital), and Dr. Aliza Hussain (cardiology fellow at Baylor College Medicine) take a deep dive on the REDUCE-IT trial with Dr. Peter Toth, director of preventive cardiology at the CGH medical center in Sterling, Illinois, clinical professor in family and community medicine at the University of Illinois School of Medicine, and past president of the National Lipid Association and the American Board of Clinical Lipidology.  Special introduction to CardioNerds Clinical Trialist Dr. Jeff Wang (Emory University). Audio editing by CardioNerds academy intern, Shivani Reddy. This episode is part of the CardioNerds Lipids Series which is a comprehensive series lead by co-chairs Dr. Rick Ferraro and Dr. Tommy Das and is developed in collaboration with the American Society For Preventive Cardiology (ASPC). Relevant disclosures: None Pearls • Notes • References • Guest Profiles • Production Team CardioNerds Cardiovascular Prevention PageCardioNerds Episode PageCardioNerds AcademyCardionerds Healy Honor Roll CardioNerds Journal ClubSubscribe to The Heartbeat Newsletter!Check out CardioNerds SWAG!Become a CardioNerds Patron! Pearls - REDUCE-IT The Reduction of Cardiovascular Events with EPA-Intervention Trial (REDUCE-IT) trial was a large randomized controlled trial that showed a significant reduction in atherosclerotic cardiovascular disease (ASCVD) events with use of icosapent ethyl ester in secondary prevention patients and high risk primary prevention patients with diabetes and residual elevated triglycerides between 135 to 499 mg/dL on top of maximally tolerated statin therapy1. Despite the use of high intensity statin therapy, considerable residual risk for future atherosclerotic cardiovascular disease exists in patients with ASCVD.Elevated triglycerides (TGs) are an important marker of increased residual ASCVD risk2.There are two primary types of Omega-3 fish oils: eicosapentaenoic acid (EPA) and docosahexaenoic acid (DHA). Omege-3 fish oils have been shown to lower triglyceride levels.Low-dose combination EPA and DHA has not exhibited incremental cardiovascular benefit in either primary prevention and secondary prevention patients on top of statin therapy3-5.REDUCE-IT showed the use of high dose EPA in patients with either ASCVD or DM and one additional risk factor, and relatively well-controlled LDL-C levels on maximally tolerated statin therapy and residual hypertriglyceridemia (TG 135-499 mg/dL) results in significant reductions in cardiovascular events over a median follow-up period of 4.9 years1. Show notes - REDUCE-IT Multiple epidemiologic and Mendelian randomization studies have established elevated triglyceride (TG) levels as an important risk factor for atherosclerotic cardiovascular events6-8. However previous clinical trials using TG-lowering medication such as niacin, fibrates and low dose omega-3 fish oil have not shown to reduce cardiovascular events when added to statin therapy in patients with or without ASCVD,9,10.The JELIS trial first demonstrated a significant reduction in cardiovascular events when 1.8g daily of eicosapentaenoic acid (EPA) was added to low-intensity statin therapy in patients with ASCVD and hypercholesterolemia, However, the trial was limited due to open label design without placebo, use of low doses of background statin therapy, and geographic/demographic limitations to participants in Japan11.In a large international multicenter randomized controlled trial, the Reduction of Cardiovascular Events with Icosapent Ethyl–Intervention Trial (REDUCE-IT) randomized 8,179 patients with established atherosclerotic heart disease or diabetes and an additional risk factor, on maximally tolerated statin therapy, to 4 gm/day of icosapent ethyl (a highly purified and stable EPA ethyl ester) or miner...
20 May 2022208. Atrial Fibrillation: Epidemiology, Health Equity, & The Double Paradox with Dr. Larry Jackson00:38:36
Atrial fibrillation may reach pandemic proportions in the next 2-3 decades. Factors that drive this phenomenon have been studied in predominantly White populations, leading to a significant underrepresentation of certain racial/ethnic groups in atrial fibrillation epidemiological studies. Most atrial fibrillation epidemiology studies suggest that the non-Hispanic Black population has a lower incidence/prevalence of atrial fibrillation, despite a higher risk factor burden (“Afib paradox”). At the same time, non-Hispanic Blacks have worse outcomes compared to the White population and underrepresented populations and women are less likely than White men to receive optimal guideline-based therapies for atrial fibrillation. In this episode, CardioNerds Dr. Kelly Arps (Co-Chair Atrial Fibrillation series, Cardiology fellow at Duke University), Dr. Colin Blumenthal (Co-Chair Atrial Fibrillation series, CardioNerds Academy House Faculty Leader for House Jones, Cardiology fellow at the University of Pennsylvania), and Dr. Dinu-Valentin Balanescu (CardioNerds Academy Faculty for House Jones, rising internal medicine chief resident at Beaumont Hospital), discuss with Dr. Larry Jackson (cardiac electrophysiologist and Vice Chief of Diversity, Equity, and Inclusion in the Division of Cardiology at Duke University) about atrial fibrillation epidemiology and health equity, challenges and possible solutions to improving diversity in clinical trials, and race/ethnicity/sex/gender differences in the detection, management, and outcomes of atrial fibrillation. Audio editing by CardioNerds Academy Intern, student doctor Akiva Rosenzveig. This CardioNerds Atrial Fibrillation series is a multi-institutional collaboration made possible by contributions of stellar fellow leads and expert faculty from several programs, led by series co-chairs, Dr. Kelly Arps and Dr. Colin Blumenthal. This series is supported by an educational grant from the Bristol Myers Squibb and Pfizer Alliance. All CardioNerds content is planned, produced, and reviewed solely by CardioNerds. We have collaborated with VCU Health to provide CME. Claim free CME here! Disclosure: Larry R. Jackson II, MD, MHs, has the following relevant financial relationships:Advisor or consultant for: Biosense Webster Inc.Speaker or a member of a speakers bureau for: Biotronik Inc.; Medtronic Inc. Pearls • Notes • References • Guest Profiles • Production Team CardioNerds Atrial Fibrillation PageCardioNerds Episode PageCardioNerds AcademyCardionerds Healy Honor Roll CardioNerds Journal ClubSubscribe to The Heartbeat Newsletter!Check out CardioNerds SWAG!Become a CardioNerds Patron! Pearls and Quotes - Atrial Fibrillation: Epidemiology, Health Equity, & The Double Paradox Atrial fibrillation confers an enormous public health burden. It is estimated that it will reach pandemic proportions over the next 30 years, with potentially 100-180 million people worldwide suffering from this condition.Large epidemiological atrial fibrillation registries have very small populations of underrepresented groups. More diverse enrollment in clinical trials is essential and may be obtained by increasing diversity among research staff, principal investigators, and steering committees, and use of mobile/telehealth technologies to remove bias related to differences in presentation. The CardioNerds Clinical Trials Network specifically aims pair equitable trial enrollment with trainee personal and professional development.Most atrial fibrillation epidemiology studies suggest that the non-Hispanic Black population has lower incidence/prevalence of atrial fibrillation, despite higher risk factor burden. This “paradox” is likely due to a multifactorial process, with clinical differences, socioeconomic factors, and genetic factors contributing.Underrepresented populations are less likely than White patients to receive optimal guideline-based management of atrial fibrillation.
24 May 2022209. CardioNerds Rounds: Challenging Cases – Modern Guideline Directed Therapy in Heart Failure with Dr. Randall Starling00:45:14
It’s another session of CardioNerds Rounds! In these rounds, Co-Chairs, Dr. Karan Desai and Dr. Natalie Stokes and Dr. Tiffany Dong (FIT at Cleveland Clinic) joins Dr. Randall Starling (Professor of Medicine and Director of Heart Transplant and Mechanical Circulatory Support at Cleveland Clinic) to discuss the nuances of guideline directed medical therapy (GDMT) through real cases. As a past president of the Heart Failure Society of America (HFSA) and author on several guidelines, Dr. Starling gives us man pearls on GDMT. Come round with us today by listening to the episodes and joining future sessions of #CardsRounds! This episode is supported with unrestricted funding from Zoll LifeVest. A special thank you to Mitzy Applegate and Ivan Chevere for their production skills that help make CardioNerds Rounds such an amazing success. All CardioNerds content is planned, produced, and reviewed solely by CardioNerds. Case details are altered to protect patient health information. CardioNerds Rounds is co-chaired by Dr. Karan Desai and Dr. Natalie Stokes.  Speaker disclosures: None Cases discussed and Show Notes • References • Production Team CardioNerds Rounds PageCardioNerds Episode PageCardioNerds AcademyCardionerds Healy Honor Roll CardioNerds Journal ClubSubscribe to The Heartbeat Newsletter!Check out CardioNerds SWAG!Become a CardioNerds Patron! Show notes - CardioNerds Rounds: Challenging Cases - Modern Guideline Directed Therapy in Heart Failure with Dr. Randall Starling Case #1 Synopsis: A man in his 60s with known genetic MYPBC3 cardiomyopathy and heart failure with a reduced ejection fraction of 30% presents with worsening dyspnea on exertion over the past 6 months. His past medical history also included atrial fibrillation with prior ablation and sick sinus syndrome with pacemaker implantation. Medications are listed below. He underwent an elective right heart catheterization prior to defibrillator upgrade for primary prevention. At the time of right heart catheterization, his blood pressure was 153/99 with a heart rate of 60. His RHC demonstrated a RA pressure of 15mmHg, RV 52/16, PA 59/32 (mean 41), and PCWP 28 with Fick CO/CI of 2.8 L/min and index of 1.2 L/min/m2. His SVR  was 1900 dynes/s/cm-5. He was admitted to the cardiac ICU and started on nitroprusside that was transitioned to a regimen of Sacubitril-Valsartan and Eplerenone. His final RHC numbers were RA 7, PA 46/18/29, PCWP 16 and Fick CO/CI 6.1/2.6. His discharge medications are shown below. Takeaways from Case #1 Unless there are contraindications (cardiogenic shock or AV block), continue a patient’s home beta blocker to maintain the neurohormonal blockade benefits. A low cardiac index should be interpreted in the full context of the patient, including their symptoms, other markers of perfusion (e.g., urine output, mentation, serum lactate), and mean arterial pressure before holding or stopping beta blockade. Carvedilol, metoprolol succinate and bisoprolol are all evidence-based options for beta blockers in heart failure with reduced ejection fraction.If there is concern of lowering blood pressure too much with Sacubitril/Valsartan, one method is to trial low dose of valsartan first and then transition to Sac/Val. Note, in the PARADIGM-HF trial, the initial exclusion criteria for starting Sac/Val included no symptomatic hypotension and SBP ≥ 100. At subsequent up-titration visits, the blood pressure criteria was decreased to SBP ≥ 95.In multiple studies, protocol-driven titration of GDMT has shown to improve clinical outcomes, yet titration remains poor. The following image from Greene et al. in JACC shows that in contemporary US outpatient practices that GDMT titration is poor with few patients reaching target dosing. Case #2 Synopsis: A 43 year-old male with a past medical history of familial dilated cardiomyopathy requiring HVAD placement two years prior now comes in with low flow alarms.
27 May 2022210. Family History of Premature ASCVD with Dr. Ann Marie Navar00:44:13
CardioNerds (Amit Goyal and Daniel Ambinder), Dr. Ahmed Ghoneem (CardioNerds Academy Chief of House Taussig and medicine resident at Lahey Hospital), and Dr. Gurleen Kaur (Director of CardioNerds Internship and medicine resident at Brigham and Women’s Hospital) discuss family history of premature ASCVD with Dr. Ann Marie Navar, Preventive Cardiologist and Associate Professor in the Departments of Internal Medicine and Population and Data Sciences at UT Southwestern Medical Center. They discuss the art of soliciting a nuanced family history, refining cardiovascular risk using risk models and novel markers, counseling patients with elevated risk, and more. Show notes were drafted by Dr. Ahmed Ghoneem and reviewed by Dr. Gurleen Kaur. Audio editing was performed by CardioNerds Intern, student Dr. Adriana Mares. For related teaching, check out this Tweetorial about CAC by Dr. Gurleen Kaur, the Family History of Premature ASCVD Infographic by Dr. Ahmed Ghoneem, and the CardioNerds Cardiovascular Prevention Series. CardioNerds Cardiovascular Prevention PageCardioNerds Episode Page Show notes - Family History of Premature ASCVD with Dr. Ann Marie Navar Patient summary: Mr. B is a 51-year-old gentleman who is referred to CardioNerds Prevention Clinic by his PCP. He does not have a significant past medical history. He is a former smoker but quit 2 years ago. His BP in clinic today is 138/84; he is not on any antihypertensives. His most recent lipid profile 2 weeks prior showed a total cholesterol level of 250 mg/dL, a TG level of 230 mg/dL, an LDL cholesterol of 174 mg/dL, and an HDL cholesterol of 30 mg/dL. He tells us that his father had a “heart attack” at the age of 52, and he would like to further understand his own risk. We calculate his ASCVD risk score, and it is 9.8%. 1. What constitutes a positive family history (FHx) of premature ASCVD? What is an approach to the art of soliciting the FHx from our patients? Definition of family history of premature ASCVD: the history of an atherosclerotic event (e.g., myocardial infarction or stroke) in a male first degree relative before the age of 55 or a female first degree relative before the age of 65. Dr. Navar’s approach to soliciting a family history:Lead with a general question such as “what do you know about any medical conditions that run in your family?”.Then ask more specific questions about the parents and siblings, such as “Is your mother still alive? How long did she live? Has she ever had a heart attack or stroke?”If the answer is yes, ask about how old they were at the time of the event.A challenging aspect of the FHx can be eliciting the difference between atherosclerotic events and sudden cardiac death. While atherosclerotic diseases are a much more common cause of unexplained sudden death, it's important that we don't miss the opportunity to identify inherited cardiomyopathies, channelopathies, inherited aortopathies or other heritable SCD syndromes. 2. Is the “dose” of family history important (for example: the number of affected relatives, the closeness of those relationships, the age of onset)? While conducting studies to test this may be difficult, the few studies that have looked at the number of affected relatives have found a dose-response type relationship, where increasing number of relatives affected increases the risk of heart disease.1,2 3. How does a family history affect cardiovascular risk stratification? FHx of premature ASCVD does not improve the predictive ability of the Pooled Cohort Equations (PCE) at a population level. Therefore, it does not factor into the ASCVD risk calculation utilizing the PCE. However, it enhances the patient’s risk at an individual level. The ACC/AHA guidelines recognize FHx of premature ASCVD as a risk-enhancing factor [together with CKD, chronic inflammatory conditions such as psoriasis, primary hypercholesterolemia, high-risk ethnicity such as South Asian ancestry...
29 May 2022211. Case Report: A Zebra in Polka Dots – Coronary Intervention With Glanzmann Thrombasthenia – UCLA00:35:11
CardioNerds (Amit and Dan) join Dr. Omid Amidi (CardioNerds Academy Graduate) and Dr. Marwah Shahid from the UCLA Cardiology Fellowship program along with Dr. Evelyn Song (CardioNerds Academy House Faculty and Heart Failure Hospitalist at UCSF) to discuss a complex case focused on management of severe coronary artery disease in a patient with Glanzmann thrombasthenia. Dr. Rushi Parikh (Interventional cardiologist, UCLA) provides the ECPR for this episode. Audio editing by CardioNerds Academy Intern, student doctor Akiva Rosenzveig. Glanzmann Thrombasthenia is a bleeding disorder due to impairment of platelet aggregation secondary to a mutation in the GPIIB/IIIA receptor. This case is focused on work up of stable coronary artery disease followed by a discussion on duration of dual antiplatelet therapy post percutaneous coronary intervention in a patient with Glanzmann thrombasthenia.   Check out this published case in JACC: Case Reports Jump to: Case media - Case teaching - References CardioNerds Case Reports PageCardioNerds Episode PageCardioNerds AcademyCardionerds Healy Honor Roll CardioNerds Journal ClubSubscribe to The Heartbeat Newsletter!Check out CardioNerds SWAG!Become a CardioNerds Patron! Case Media See the published case in JACC: Case Reports Episode Schematics & Teaching Pearls 1. Patients with Glanzmann thrombocytopenia (GT) may have a higher risk of bleeding, depending on their disease phenotype. 2. It is unclear whether the mechanism of GT protects patient against stent thrombosis in the setting of PCI. Additionally, there is little data on the use of antiplatelet agents in patients with GT. 3. Short-term DAPT may be a reasonably safe option for patients with GT undergoing PCI. 4. We report a successful case of percutaneous coronary intervention in a patient with GT with no complications at a 1 year follow up. Notes 1. What is Glanzmann thrombasthenia? GT is an inherited platelet disorder that is characterized by spontaneous bleeding with phenotypic variability ranging from minimal bruising to potentially fatal hemorrhaging.  GT is caused by autosomal recessive inheritance of quantitative or qualitative deficiencies of functional αIIbβ3 integrin coded by ITGA2B or ITGB3 genes for αIIb and β3, respectively. As a result, platelets may be stimulated, but the platelet glycoprotein IIb/IIIa receptor is unable to bind fibrinogen to cross-link platelets, rending them potentially ineffective. In platelet aggregation studies, there is lack of response to collagen, epinephrine, arachidonic acid, and ADP stimulation. Thus, platelet aggregation is impaired.  2. What is known about PCI and antiplatelet therapy in the setting of Glanzmann thrombasthenia? To the best of our knowledge, this is the first case report of percutaneous coronary intervention in the setting of GT. It is unclear if the mechanism of GT alone provides sufficient antiplatelet activity and whether antiplatelet therapy leads to significantly increased bleeding risk. The use of antiplatelet therapy is not well studied in the GT population. What we do know is that the mechanism of GT prevents platelet aggregation—the final step in platelet-related thrombosis—while oral antiplatelet therapy affects platelet activation, thus, in our patient we felt that short term DAPT was reasonable. It is important to note that in the event of an active bleed requiring platelet transfusion, donor platelets possess functional glycoprotein IIb/IIIa receptors and thus exponentially increase the risk of stent thrombosis. Therefore, unlike our case, if a patient is not maintained on chronic oral antiplatelet therapy, initiation of oral or intravenous antiplatelet therapy should be considered to prevent stent thrombosis at the time of platelet transfusion. Like any other patient with a high bleeding risk, it is important to have clear indications to conduct a coronary angiogram in patient...
03 Jun 2022212. Narratives in Cardiology: Becoming & Thriving as a Fellowship Program Director with Dr. Katie Berlacher and Dr. Julie Damp – Tennessee Chapter00:55:28
CardioNerds (Amit Goyal and Daniel Ambinder), join  Dr. Gurleen Kaur (Director of CardioNerds Internship and medicine resident at Brigham and Women’s Hospital),  Dr. Victoria Thomas (Cardionerds Ambassador, Vanderbilt University Medical Center) Dr. Katie Berlacher (Cardiology program director, University of Pittsburgh Medical Center), and Dr. Julie Damp (Vanderbilt University Medical Center Cardiovascular disease fellowship program director) to discuss becoming & thriving as a fellowship program director and more in this installment of the Narratives in Cardiology Series. Special message by Tennessee ACC State Chapter Governor, Dr. John L Jefferies. Audio editing by CardioNerds Academy Intern, student doctor Akiva Rosenzveig. The PA-ACC & CardioNerds Narratives in Cardiology is a multimedia educational series jointly developed by the Pennsylvania Chapter ACC, the ACC Fellows in Training Section, and the CardioNerds Platform with the goal to promote diversity, equity, and inclusion in cardiology. In this series, we host inspiring faculty and fellows from various ACC chapters to discuss their areas of expertise and their individual narratives. Join us for these captivating conversations as we celebrate our differences and share our joy for practicing cardiovascular medicine. We thank our project mentors Dr. Katie Berlacher and Dr. Nosheen Reza. Video Version • Notes • Production Team Claim free CME just for enjoying this episode! There are no relevant disclosures for this episode. The PA-ACC & CardioNerds Narratives in Cardiology PageCardioNerds Episode PageCardioNerds AcademyCardionerds Healy Honor Roll CardioNerds Journal ClubSubscribe to The Heartbeat Newsletter!Check out CardioNerds SWAG!Become a CardioNerds Patron! Tweetorial - Becoming & Thriving as a Fellowship Program Director with Dr. Katie Berlacher and Dr. Julie Damp https://twitter.com/gurleen_kaur96/status/1542620967733805056?s=21&t=AMSKElEz4oZZTA9nVbWBCA Video version - Becoming & Thriving as a Fellowship Program Director with Dr. Katie Berlacher and Dr. Julie Damp https://youtu.be/E-C-SSV7LZg Notes - Becoming & Thriving as a Fellowship Program Director with Dr. Katie Berlacher and Dr. Julie Damp Drafted by Dr. Victoria Thomas. 1. What does it mean to be a big “E” when people say they are a clinician Educator? It can mean teaching students directly at bedside.  However, it is also a sacrifice of daily mentoring and listening to students’ challenges and difficulties.Being a clinician educator is just as much of a calling as is serving in medicine.Clinician Educators focus on medicine but also the science and best practices of teaching the art of doctoring. 2. What is physician burnout? Why is this important for to CardioNerds? Physician burnout is a syndrome of chronic workplace stress that leads to emotional exhaustion and a sense of dissatisfaction and disconnection personally and professionally. 30-45% of cardiologists have reported physician burnout. 3. What factors affect physician burnout? Emotional and physical exhaustion often lead to physician burnout. First year of training as an intern or fellow and first year of serving as an attending are particularly high-risk periods. This is largely due to learning a new system and responsibilities mixed with a sense of decreased accomplishment.The sense of decreased accomplishment can lead to physicians suffering from impostor syndrome.Grit can be defined as a perseverance for long-term goals.  The level of grit was not associated with burnout among first-year Internal Medicine residents. 4. What are some of the solutions to prevent or address physician burnout? Physicians need to feel a sense of belonging and should be supported and celebrated when they have accomplished something by their colleagues and administrators. Fellows and attendings want to feel listened to and supported.Destigmatizing this idea of “perfection in medicine”.
15 Jun 2022213. ACHD: Transitions of Care in Congenital Heart Disease with Dr. Peter Ermis and Dr. Scott Cohen00:43:50
CardioNerd (Amit Goyal),  ACHD series co-chair Dr. Agnes Koczo (UPMC), and episode FIT lead, Dr. Logan Eberly (Emory University, incoming ACHD fellow at Boston Adult Congenital Heart) join Dr. Peter Ermis (Program Director of the Adult Congenital Heart Disease Program at Texas Children's Heart Center), and Dr. Scott Cohen (Associate Professor and Director of the Adult Congenital Heart Disease Program at the Medical College of Wisconsin) for a discussion about transitions of care in congenital heart disease.  Audio editing by Dr. Gurleen Kaur (Director of the CardioNerds Internship and CardioNerds Academy Fellow). Congenital heart disease (CHD) is the most common clinically significant congenital defect, occurring in approximately 1 in 100 live births. With modern advances in pediatric cardiology and cardiac surgery, over 90% of children born in the developed world with CHD will now survive into adulthood, and there are currently more adults than children living with CHD in the United States1. As these children become adults, they will need to transition their care from pediatric to adult-centered care. Unfortunately, during this transition period, there is often delayed or inappropriate care, improper timing of the transfer of care, and undue emotional and financial stress on the patients, their families, and the healthcare system. At its worst, patients are lost to appropriate follow-up. In this episode, we review the current climate in transitions of care for CHD patients from child-centered to adult-centered care, discuss the difficulties that can occur during the transitions process. We further discuss how to mitigate them, and highlight the key elements to the successful transitions of care. The CardioNerds Adult Congenital Heart Disease (ACHD) series provides a comprehensive curriculum to dive deep into the labyrinthine world of congenital heart disease with the aim of empowering every CardioNerd to help improve the lives of people living with congenital heart disease. This series is multi-institutional collaborative project made possible by contributions of stellar fellow leads and expert faculty from several programs, led by series co-chairs, Dr. Josh Saef, Dr. Agnes Koczo, and Dr. Dan Clark. The CardioNerds Adult Congenital Heart Disease Series is developed in collaboration with the Adult Congenital Heart Association, The CHiP Network, and Heart University. See more Disclosures: None Pearls • Notes • References • Guest Profiles • Production Team CardioNerds Adult Congenital Heart Disease PageCardioNerds Episode PageCardioNerds AcademyCardionerds Healy Honor Roll CardioNerds Journal ClubSubscribe to The Heartbeat Newsletter!Check out CardioNerds SWAG!Become a CardioNerds Patron! Pearls - Transitions of Care in Congenital Heart Disease There is a clear distinction between the TRANSFER of care and TRANSITION of care. Transfer is merely moving from a pediatric to adult provider. Transition involves the continuing education of the patient with regards to their congenital heart disease, the importance of longitudinal follow up, and leading patients toward more autonomous medical care. Transition begins in the pediatric cardiology clinic prior to the transfer of care and is an ongoing process that continues well after the physical transfer of care. A critical aspect of the transition and transfer of care is cultivating trust—that is, the new adult congenital heart disease (ACHD) provider must earn the trust of the patient and family. A failure to do so will inevitably prevent an optimal transition of care. During transition, parents are transitioning along with their children. With transition to adult care, there is also a goal to transition responsibility for medical care from the parent to the child. Setting goals and expectations can help both the parents and the child effectively make this transition. Loss to follow up is one of the most concerning complications ...
19 Jun 2022214. Lipids: Review of Icosapent Ethyl with Dr. Michael Shapiro00:32:39
CardioNerds Tommy Das (Program Director of the CardioNerds Academy and cardiology fellow at Cleveland Clinic), Rick Ferraro (cardiology fellow at the Johns Hopkins Hospital), and Dr. Xiaoming Jia (Cardiology Fellow at Baylor College Medicine) take a closer look at the mechanism of icosapent ethyl in triglyceride lowering and ASCVD risk reduction with Dr. Michael Shapiro, the Fred M. Parrish professor of cardiology at Wake Forest University and Director of the Center for Preventative Cardiology at Wake Forest Baptist Health. Audio editing by CardioNerds Academy Intern, student doctor Akiva Rosenzveig. This episode is part of the CardioNerds Lipids Series which is a comprehensive series lead by co-chairs Dr. Rick Ferraro and Dr. Tommy Das and is developed in collaboration with the American Society For Preventive Cardiology (ASPC). Relevant disclosures: None Pearls • Notes • References • Guest Profiles • Production Team CardioNerds Cardiovascular Prevention PageCardioNerds Episode PageCardioNerds AcademyCardionerds Healy Honor Roll CardioNerds Journal ClubSubscribe to The Heartbeat Newsletter!Check out CardioNerds SWAG!Become a CardioNerds Patron! Pearls - Icosapent Ethyl Eicosapentaenoic acid (EPA) and docosahexaenoic acid (DHA) are two major Omega-3 fatty acids found in fish oil. While both have been shown to lower triglycerides, only purified EPA formulations have been shown to reduce ASCVD risk.Mechanisms of triglyceride (TG) lowering by icosapent ethyl are multiple and include reduction of hepatic VLDL production, stimulation of lipoprotein lipase activity, increased chylomicron clearance, reduced lipogenesis, increased beta oxidation, and reduced delivery of fatty acids to the liver.There was only modest reduction of triglycerides in REDUCE-IT and JELIS despite association with significant reduction in cardiovascular outcome events, suggesting likely mechanisms outside of triglyceride lowering that may contribute to ASCVD reduction.While there was an increased signal for peripheral edema and atrial fibrillation associated with icosapent ethyl in prior trials, overall side effect rates were very low.Icosapent ethyl is considered to be cost-effective based on cost-effective analysis. Show notes - Icosapent Ethyl EPA and DHA have differing biological properties that may explain differences in ASCVD risk reduction observed in cardiovascular outcome trials 1.The REDUCE-IT trial, which enrolled secondary prevention and high-risk primary prevention patients with elevated triglycerides who were on statin therapy, showed significant reduction of major adverse cardiovascular events in the icosapent ethyl group compared with a mineral oil placebo2. Only modest reductions of TG were seen in the REDUCE-IT and JELIS trials despite association with significant reduction in events 2,3.  Potential mechanisms contributing favorable effects of EPA on ASCVD risk reduction include inhibition of cholesterol crystal formation, stabilization of membrane structures, reversal of endothelial dysfunction, inhibition of lipoprotein and membrane lipid oxidation 4.Pleotropic effects of EPA include influence on platelet aggregation, lower thromboxane activity, increased prostaglandin level, and effects on blood pressure, insulin resistance and inflammation.Triglycerides are a surrogate for triglycerides-rich lipoproteins, which are likely causally associated with ASCVD 5.There is increased signal for bleeding, lower extremity edema, and atrial fibrillation with icosapent ethyl but overall side effect rates are very low 2.In order to ensure higher rates of medication access and adherence, clinicians must be cognizant of the cost to the patient. In practice, it is important to have a structured approach to improve insurance approval rate for medications that require prior authorizationsWith icosapent ethyl, cost effectiveness analyses have shown the medication is cost-effect for ASCVD risk reduction in secondary...
27 Jun 2022215. Atrial Fibrillation: Screening, Detection, and Diagnosis of Atrial Fibrillation with Dr. Ben Freedman00:49:35
CardioNerds (Dr. Kelly Arps, Dr. Colin Blumenthal, Dr. Dan Ambinder, and Dr. Teodora Donisan) discuss the screening, detection, and diagnosis of atrial fibrillation (AF) with Dr. Ben Freedman. AF is frequently undiagnosed and its first manifestation can be a debilitating stroke. European and American guidelines differ slightly with regards to guidelines for AF screening in asymptomatic individuals. There are multiple methods available to screen for AF; the setting and the clinical scenario can help guide the choice. Consumer-led screening has its own challenges, as it can detect AF in a younger population where we should prioritize aggressive management of risk factors and comorbidities. There is uncertainty regarding the minimum AF burden that increases thromboembolic risk, however a high CHAD2S2-VASc score remains the strongest predictor of stroke risk independent of AF burden. Perioperative AF associated with non-cardiac surgery has increased risk of future stroke and adverse cardiac outcomes and should likely be treated as a new diagnosis of chronic AF.    This CardioNerds Atrial Fibrillation series is a multi-institutional collaboration made possible by contributions of stellar fellow leads and expert faculty from several programs, led by series co-chairs, Dr. Kelly Arps and Dr. Colin Blumenthal. This series is supported by an educational grant from the Bristol Myers Squibb and Pfizer Alliance. All CardioNerds content is planned, produced, and reviewed solely by CardioNerds. We have collaborated with VCU Health to provide CME. Claim free CME here! Disclosures: Dr. Ben Freedman disclosed that he has received grant or research support from Pfizer. Pearls • Notes • References • Guest Profiles • Production Team CardioNerds Atrial Fibrillation PageCardioNerds Episode PageCardioNerds AcademyCardionerds Healy Honor Roll CardioNerds Journal ClubSubscribe to The Heartbeat Newsletter!Check out CardioNerds SWAG!Become a CardioNerds Patron! Pearls and Quotes - Screening, Detection, and Diagnosis of Atrial Fibrillation “Stroke is a poor early sign of AF.”  AF remains frequently undiagnosed and there remains uncertainty about the optimal target population and screening methodology.  “We have to tailor AF screening to the purpose we’re using it for”  If in a primary care setting, check the pulse. If the goal is to exclude high-risk AF –  handheld ECG for heart rhythm snapshots are appropriate. If the goal is to identify or exclude AF with a high level of certainty, continuous monitors are necessary for greater sensitivity. Consumer-led screening is performed by (mostly young) individuals using commercial monitors and smart watches, facilitating earlier recognition of paroxysmal AF in this population. In these cases, we should prioritize aggressive management of risk factors and comorbidities to reduce the risk of progression to persistent AF. There is no specific cutoff for AF duration which has been identified to predict elevated stroke risk; AF is likely both a risk factor and a risk marker for stroke, suggesting an underlying atrial myopathy. Non-cardiac surgeries and procedures can be considered “AF stress tests.” If AF occurs in these settings, it is usually more clinically significant and has a higher risk of stroke and death than AF associated with cardiac surgeries.  Notes - Screening, Detection, and Diagnosis of Atrial Fibrillation Notes drafted by Dr. Teodora Donisan and reviewed by Dr. Kelly Arps 1. Why is it important to screen for AF and who should be screened?  AF is frequently undiagnosed and its first manifestation can be a debilitating stroke or death. Let’s go over a few numbers:  15% of people with AF are currently undiagnosed and 75% of those individuals would be eligible for anticoagulation.1   10-38% of individuals with ischemic strokes are found to have AF as a plausible cause, and the true proportion may be even higher,
30 Jun 2022216. Guidelines: 2021 ESC Cardiovascular Prevention – Question #13 with Dr. Eugene Yang00:11:32
The following question refers to Section 3.2 of the 2021 ESC CV Prevention Guidelines. The question is asked by student Dr. Hirsh Elhence, answered first by Mayo Clinic Fellow Dr. Teodora Donisan, and then by expert faculty Dr. Eugene Yang. Dr. Yang is professor of medicine of the University of Washington where he is medical director of the Eastside Specialty Center and the co-Director of the Cardiovascular Wellness and Prevention Program. Dr. Yang is former Governor of the ACC Washington Chapter and current chair of the ACC Prevention of CVD Section. The CardioNerds Decipher The Guidelines Series for the 2021 ESC CV Prevention Guidelines represents a collaboration with the ACC Prevention of CVD Section, the National Lipid Association, and Preventive Cardiovascular Nurses Association. Question #13 You are seeing a 45-year-old woman with a past medical history of hypertension, overweight status, hyperlipidemia, and active tobacco use disorder. Her BMI is 27 kg/m2, BP is 150/75, HbA1C is 5.8%, total cholesterol is 234 mg/dL, HDL is 59 mg/dL, and LDL is 155 mg/dL. She is from Romania, a country with very high CVD risk. Which of the following statements is CORRECT? A. LDL-C needs to be decreased by at least 50%, as small absolute LDL-C reductions would not provide clinical benefit B. Hypertension is not an important CVD risk factor in our patient, as she is young. C. Prediabetes is not a significant CV risk factor for our patient, as she is not yet diabetic. D. Smoking confers a higher CVD risk for women than for men. E. Her weight does not increase her CVD risk, as she is overweight rather than obese Answer #13 The correct answer is D – Smoking confers a higher CVD risk for women than for men. Prolonged smoking increases the CVD risk more in women than in men. Our patient is 45 years old. CVD risk in smokers < 50 years-old is 5x higher than in non-smokers. Of note, smoking is responsible for 50% of all avoidable deaths in smokers and a lifetime smoker will lose 10 years of life, on average. Secondhand smoke and smokeless tobacco can also increase the CVD risk. Option A is incorrect. The SCORE2 risk chart for populations at very high CVD risk places her at a 14% (very high) 10-year risk for myocardial infarction, stroke, or cardiovascular death. She would derive benefit even from incremental reductions in LDL-C values. The absolute benefit of lowering LDL-C depends on both the absolute risk of ASCVD and the absolute reduction in LDL-C, so even a small absolute reduction in LDL-C may be beneficial in high- or very-high-risk patients. Furthermore, the reduction in CVD risk is proportional to the decrease in LDL-C, irrespective of the medications used to achieve such change. This remains true even when lowering LDL-C values to < 55 mg/dl. Option B is incorrect. Hypertension is a major cause of CVD regardless of age, and the risk of death from either CAD or stroke increases linearly from BP levels as low as 90 mmHg systolic and 75 mmHg diastolic upwards. Particularly relevant for our patient, lifetime BP evolution differs in women compared to men, potentially resulting in an increased CVD risk at lower BP thresholds. Option C is incorrect. Type 1 DM, type 2 DM, and prediabetes are all independent risk factors for ASCVD. Of note, it would be important to address this risk factor with our patient, as women who develop type 2 diabetes have a particularly high risk for stroke. Option E is incorrect. All-cause mortality is lowest at a BMI on 20-25 kg/m2 in apparently healthy patients. Even overweight patients are at increased CVD risk. There is a linear relationship between BMI and mortality in non-smokers and a J-shaped relationship in ever-smokers. In patients with heart failure, a lower mortality risk has been observed with higher BMI – the “obesity paradox.” It would be important to evaluate the waist circumference in our patient, as both BMI and waist circumference are associated with ASCVD risk.
01 Jul 2022217. Guidelines: 2021 ESC Cardiovascular Prevention – Question #14 with Dr. Allison Bailey00:11:55
The following question refers to Sections 3.3-3.4 of the 2021 ESC CV Prevention Guidelines. The question is asked by student Dr. Adriana Mares, answered first by early career preventive cardiologist Dr. Dipika Gopal, and then by expert faculty Dr. Allison Bailey. Dr. Bailey is a cardiologist at Centennial Heart. She is the editor-in-chief of the American College of Cardiology's Extended Learning (ACCEL) editorial board and was a member of the writing group for the 2018 American Lipid Guidelines.  The CardioNerds Decipher The Guidelines Series for the 2021 ESC CV Prevention Guidelines represents a collaboration with the ACC Prevention of CVD Section, the National Lipid Association, and Preventive Cardiovascular Nurses Association. Question #14 Ms. Soya M. Alone is a 70-year-old woman of Bangladeshi ethnicity with a history of anxiety and depression. She currently lives at home by herself, does not have many friends and family that live nearby, and has had a tough year emotionally after the passing of her husband. She spends most of her time in bed with low daily physical activity and has experienced more weakness and exhaustion over the past year along with loss of muscle mass. Which of the following are potential risk modifiers in this patient when considering her risk for CVD?A. Bangladeshi ethnicity B. Psychosocial factorsC. Frailty D. History of anxiety and depressionE. All of the above Answer #14 The correct answer is E – All of the above.Traditional 10-year CVD risk scores do not perform adequately in all ethnicities. Therefore, multiplication of calculated risk by relative risk for specific ethnic subgroups should be considered (Class IIa, LOE B). Individuals from South Asia have higher CVD rates. The ESC guidelines recommend using a correction factor by multiplying the predicted risk by 1.3 for Indians and Bangladeshis, and 1.7 for Pakistanis. These correction factors are derived from data from QRISK3. In the UK, the QRISK calculator algorithm has been derived and validated in 2.3 million people to estimate CVD risk in different ethnic groups and unlike other calculators, it counts South Asian origins as an additional risk factor. The reasons for such differences remain inadequately studied, as do the risks associated with other ethnic backgrounds. Barriers to developing accurate risk prediction tools include the wide heterogeneity amongst the population.The 2019 ACC/AHA guidelines also list high-risk race/ethnicities such as South Asian ancestry as a risk-enhancing factor. However, there is no separate pooled cohort equation for different ethnicities, and consideration should be given that the pooled cohort equations will underestimate ASCVD risk in South Asians.Psychosocial stress including loneliness and critical life events are associated, in a dose-response pattern, with the development and progression of ASCVD, with relative risks between 1.2 and 2.0. Conversely, indicators of mental health, such as optimism and a strong sense of purpose, are associated with lower risk. While there is not a specific way proposed by the guidelines for psychosocial factors to improve risk classification, it is important to screen patients with ASCVD for psychological stress, and clinicians should attend to somatic and emotional causes of symptoms as well. The ESC guidelines give a Class IIa (LOE B) recommendation for assessment of stress symptoms and psychosocial stressors.This patient should also be formally screened for frailty, which is not the same as aging but includes factors such as slowness, weakness, low physical activity, exhaustion and shrinking, and makes her more vulnerable to the effect of stressors and is a risk factor for both high CV and non-CV morbidity and mortality. However, the ability of frailty measures to improve CVD risk prediction has not been formally assessed, so the guidelines do not recommend integrating it into formal CVD risk assessment. Frailty may however,
05 Jul 2022218. Guidelines: 2021 ESC Cardiovascular Prevention – Question #15 with Dr. Kim Williams00:09:51
The following question refers to Section 4.3 of the 2021 ESC CV Prevention Guidelines. The question is asked by CardioNerds Academy Intern Dr. Maryam Barkhordarian, answered first by pharmacy resident Dr. Anushka Tandon and then by expert faculty Dr. Kim Williams. Dr. Williams is Chief of the Division of Cardiology and is Professor of Medicine and Cardiology at Rush University Medical Center. He has served as President of ASNC, Chairman of the Board of the Association of Black Cardiologists (ABC, 2008-2010), and President of the American College of Cardiology (ACC, 2015-2016). The CardioNerds Decipher The Guidelines Series for the 2021 ESC CV Prevention Guidelines represents a collaboration with the ACC Prevention of CVD Section, the National Lipid Association, and Preventive Cardiovascular Nurses Association. Question #15 Your patient mentions that she drinks “several” cups of coffee during the day. She also describes having a soda daily with lunch and occasionally a glass of wine with dinner. Which of the following recommendations is appropriate?  A. Coffee consumption is not harmful and may even be beneficial, regardless of the number of drinks per day. B. Drinking two glasses of wine/day is safe from a cardiovascular prevention standpoint. C. Soft drinks (and other sugar-sweetened beverages) must be discouraged. D. None of the above Listen to this podcast episode!  Answer #15 The correct answer is C.  Soft drinks (and other sugar-sweetened beverages) must be discouraged. Sugar-sweetened beverages have been associated with a higher risk of CAD and all-cause mortality. The ESC guidelines give a class I recommendation for restriction of free sugar consumption (in particular sugar-sweetened beverages) to a maximum of 10% of energy intake. This is a class IIa recommendation in the ACC/AHA guidelines. Choice A is incorrect because: the consumption of nine or more drinks a day of non-filtered coffee (such as boiled, Greek, and Turkish coffee and some espresso coffees) may be associated with an up to 25% increased risk of ASCVD mortality. Moderate coffee consumption (3-4 cups per day) is probably not harmful, and perhaps even moderately beneficial. Choice B is incorrect: It is a class I recommendation to restrict alcohol consumption to a maximum of 100 g per week. The standard drink in the US contains 14 g of alcohol, so 100 mg of alcohol translate to: o   84 ounces of beer (5% alcohol) o   Or 56 – 63 ounces of malt liquor (75% alcohol) or o   Or 35 ounces of wine (12% alcohol) or ONE 5 fl oz glass of wine/day. o   Or 31.5 ounces of distilled spirits (40% alcohol). The ACC/AHA guidelines recommended limiting alcohol consumption only for the management of hypertension to: ≤2 drinks daily for men and: ≤1 drink daily for women. Main Takeaway The main takeaway: ASCVD risk reduction can be achieved by restricting sugar-sweetened beverages to a maximum of 10% of energy intake. Guideline Location Section 4.3.2, Page 3271 CardioNerds Decipher the Guidelines - 2021 ESC Prevention Series CardioNerds Episode Page CardioNerds Academy Cardionerds Healy Honor Roll CardioNerds Journal Club Subscribe to The Heartbeat Newsletter! Check out CardioNerds SWAG! Become a CardioNerds Patron!
05 Jul 2022219. Guidelines: 2021 ESC Cardiovascular Prevention – Question #16 with Dr. Roger Blumenthal00:11:22
The following question refers to Section 4.6 and Figure 13 of the 2021 ESC CV Prevention Guidelines. The question is asked by student doctor Shivani Reddy, answered first by NP Carol Patrick, and then by expert faculty Dr. Roger Blumenthal. Dr. Roger Blumenthal is professor of medicine at Johns Hopkins where he is Director of the Ciccarone Center for the Prevention of Cardiovascular Disease. He was instrumental in developing the 2018 ACC/AHA CV Prevention Guidelines. The CardioNerds Decipher The Guidelines Series for the 2021 ESC CV Prevention Guidelines represents a collaboration with the ACC Prevention of CVD Section, the National Lipid Association, and Preventive Cardiovascular Nurses Association. Question #16 True or False: For patients with established ASCVD, secondary prevention entails adding a PCSK9 inhibitor if goal LDL is not met on maximum tolerated doses of a statin and ezetimibe. Answer #16 The correct answer is True. The ultimate on-treatment LDL-C goal of <55 mg/dL (<1.4 mmol/L) and a reduction of at least ≥50% from baseline should be considered for primary prevention of persons <70 years of age at very high risk (Class IIa) and in those with established ASCVD (Class I). It is recommended that a high-intensity statin is prescribed up to the highest tolerated dose to reach these LDL-C goals (Class I). The combination of statin with ezetimibe brings a benefit that is in line with meta-analyses showing that LDL-C reduction has benefits independent of the approach used.  The beneficial effect of ezetimibe is also supported by genetic studies. Together, these data support the position that ezetimibe should be considered as second-line therapy, either on top of statins when the therapeutic goal is not achieved (Class I), or when a statin cannot be prescribed (Class IIa). PCSK9 inhibitors (monoclonal antibodies to PCSK9) decrease LDL-C by up to 60%, either as monotherapy or in addition to the maximum tolerated dose of statin and/or other lipid-lowering therapies, such as ezetimibe. Their efficacy appears to be largely independent of background therapy. Among patients in whom statins cannot be prescribed, PCSK9 inhibition reduced LDL-C levels when administered in combination with ezetimibe. Both alirocumab and evolocumab effectively lower LDL-C levels in patients who are at high or very high CVD risk, including those with DM, with a large reduction in future ASCVD events. Therefore, for those who do not meet LDL-C goals with maximally tolerated doses of both a statin and ezetimibe, combination therapy including a PCSK9 inhibitor may be considered for primary prevention of patients at very high risk but without familial hypercholesterolemia (Class IIa) and is recommended for secondary prevention for those with established ASCVD (Class I). In addition, for very-high-risk FH patients (that is, with ASCVD or with another major risk factor) who do not achieve their goals on a maximum tolerated dose of a statin and ezetimibe, combination therapy including a PCSK9 inhibitor is recommended (Class I). Main Takeaway Statins, ezetimibe, and PCSK9 inhibitors should be used in a stepwise approach to achieve target lipid lowering goals in accordance with their risk profile. Guideline Location Page 3279, Sections 4.6.3.1.4, 4.6.3.1.5 Figure 13 page 3278; recommendation table page 3279. CardioNerds Decipher the Guidelines - 2021 ESC Prevention Series CardioNerds Episode Page CardioNerds Academy Cardionerds Healy Honor Roll CardioNerds Journal Club Subscribe to The Heartbeat Newsletter! Check out CardioNerds SWAG! Become a CardioNerds Patron!
07 Jul 2022220. Guidelines: 2021 ESC Cardiovascular Prevention – Question #17 with Dr. Melissa Tracy00:08:51
The following question refers to Section 4.9 of the 2021 ESC CV Prevention Guidelines. The question is asked by Dr. Christian Faaborg-Andersen, answered first by UCSD fellow Dr. Patrick Azcarate, and then by expert faculty Dr. Melissa Tracy. Dr. Tracy is a preventive cardiologist, former Director of the Echocardiography Lab, Director of Cardiac Rehabilitation, and solid organ transplant cardiologist at Rush University. The CardioNerds Decipher The Guidelines Series for the 2021 ESC CV Prevention Guidelines represents a collaboration with the ACC Prevention of CVD Section, the National Lipid Association, and Preventive Cardiovascular Nurses Association. Question #17 A 74-year-old man with a history of hypertension, chronic kidney disease, and gastroesophageal reflux presents with chest pain and is found to have an NSTEMI due to an obstructive lesion in the proximal LAD. One drug-eluting stent is placed, and he is started on dual antiplatelet therapy with aspirin and clopidogrel. He is concerned about the risk of bleeding from his gastrointestinal tract. What would you recommend to reduce his risk of bleeding? A. Lansoprazole, a proton pump inhibitor B. Famotidine, a histamine-2 blocker C. Calcium carbonate, an antacid D. None, proton pump inhibitors are contraindicated. Answer #17 The correct answer is A. The ESC recommends that patients at high risk for GI bleeding who are receiving antiplatelet therapy take proton pump inhibitors (Class I, LOE A). High risk for bleeding includes patients who are age ≥65, history of peptic ulcer disease, Helicobacter pylori infection, dyspepsia or GERD symptoms, chronic renal failure, diabetes mellitus, and concomitant use of other antiplatelet agents, anticoagulants, nonsteroidal anti-inflammatory drugs, or steroids. Coadministration of proton pump inhibitors that specifically inhibit CYP2C19 (omeprazole or esomeprazole) may reduce the pharmacodynamic response to clopidogrel. Although this interaction has not been shown to affect the risk of ischemic events, coadministration of omeprazole or esomeprazole with clopidogrel is not recommended. Main Takeaway In patients with high gastrointestinal bleeding risk who are receiving antiplatelet therapy, proton pump inhibitors are recommended. Omeprazole and esomeprazole may reduce the efficacy of clopidogrel and should not be used concomitantly with clopidogrel. Guideline Location Section 4.9.3, Page 3291 Figure 13 page 3278; recommendation table page 3279. CardioNerds Decipher the Guidelines - 2021 ESC Prevention Series CardioNerds Episode Page CardioNerds Academy Cardionerds Healy Honor Roll CardioNerds Journal Club Subscribe to The Heartbeat Newsletter! Check out CardioNerds SWAG! Become a CardioNerds Patron!
12 Jul 2022221. Guidelines: 2021 ESC Cardiovascular Prevention – Question #18 with Dr. Jaideep Patel00:07:15
The following question refers to Section 6.2 of the 2021 ESC CV Prevention Guidelines. The question is asked by Dr. Christian Faaborg-Andersen, answered first by Houston Methodist medicine resident Dr. Najah Khan, and then by expert faculty Dr. Jaideep Patel. Dr. Patel recently graduated from Virginia Commonwealth University cardiology fellowship and is now a preventive cardiologist at the Johns Hopkins Hospital. The CardioNerds Decipher The Guidelines Series for the 2021 ESC CV Prevention Guidelines represents a collaboration with the ACC Prevention of CVD Section, the National Lipid Association, and Preventive Cardiovascular Nurses Association. Question #18 A 60-year-old Black woman with a history of hypertension and heart failure with reduced ejection fraction (EF 40%) presents to clinic for follow-up. She is currently doing well with NYHA class II symptoms. She is taking carvedilol 25 mg BID, sacubitril/valsartan 97/103 mg BID, and spironolactone 25 mg daily, all of which have been well tolerated. In clinic, her BP is 125/80 mmHg, and her HR is 55 bpm. Routine labs are within normal limits including Cr of 1.0, K of 4.0, and HbA1c of 6.0. What is the most appropriate next step in her management? A. No change in management B. Reduce beta blocker C. Add an SGLT2 inhibitor (dapagliflozin or empagliflozin) D. Add vericiguat E. Add hydralazine/isosorbide dinitrate Answer #18 The correct answer is C – Add an SGLT2 inhibitor (dapagliflozin or empagliflozin) For patients with symptomatic HFrEF, neurohormonal antagonists (ACEi, ARB, ARNI; BB; MRA) improve survival and reduce the risk of HF hospitalization. This patient is already on these agents. The addition of an SGLT2 inhibitor on top of neurohormonal blockade reduces the risk of CV death and worsening HF in patients with symptomatic HFrEF and is the next best step for this patient (Class I, LOE A). Vericiguat may be considered in patients with symptomatic HFrEF with HF worsening despite already being on maximally tolerated neurohormonal blockade (Class IIb, LOE B), but first-line therapies should be started first. Hydralazine/Isosorbide dinitrate should be considered in self-identified Black patients or people who have EF ≤ 35% or <45% with dilated LV with class III-IV symptoms despite maximally tolerated neurohormonal blockade (Class IIa, LOE B), but is not the next best step here. She is tolerating the beta blocker without adverse effects so there is no reason to decrease the dosage. Main Takeaway In patients with symptomatic HFrEF (EF ≤ 40%), SGLT2 inhibitors are considered first line therapy in addition to ACE-I/ARB/ARNI, BB, and MRAs to reduce the risk of HF hospitalization and death. Importantly this is irrespective of presence of diabetes. Guideline Location Section 6.2, page 3295-3296 Figure 13 page 3278; recommendation table page 3279. CardioNerds Decipher the Guidelines - 2021 ESC Prevention Series CardioNerds Episode Page CardioNerds Academy Cardionerds Healy Honor Roll CardioNerds Journal Club Subscribe to The Heartbeat Newsletter! Check out CardioNerds SWAG! Become a CardioNerds Patron!
14 Jul 2022222. CardioNerds Rounds: Challenging Cases – Nuances in Pulmonary Hypertension Management with Dr. Ryan Tedford00:42:57
It’s another session of CardioNerds Rounds! In these rounds, Co-Chair, Dr. Karan Desai (previous FIT at the University of Maryland Medical Center, and now faculty at Johns Hopkins) joins Dr. Ryan Tedford (Professor of Medicine and Chief of Heart Failure and Medical Directory of Cardiac Transplantation at the Medical University of South Carolina in Charleston, SC) to discuss the nuances of managing pulmonary hypertension in the setting of left-sided heart disease. Dr. Tedford is an internationally-recognized clinical researcher, educator, clinician and mentor, with research focuses that include the hemodynamic assessment of the right ventricle and its interaction with the pulmonary circulation and left heart. This episode is supported with unrestricted funding from Zoll LifeVest. A special thank you to Mitzy Applegate and Ivan Chevere for their production skills that help make CardioNerds Rounds such an amazing success. All CardioNerds content is planned, produced, and reviewed solely by CardioNerds. Case details are altered to protect patient health information. CardioNerds Rounds is co-chaired by Dr. Karan Desai and Dr. Natalie Stokes.  Speaker disclosures: None Cases discussed and Show Notes • References • Production Team CardioNerds Rounds PageCardioNerds Episode PageCardioNerds AcademyCardionerds Healy Honor Roll CardioNerds Journal ClubSubscribe to The Heartbeat Newsletter!Check out CardioNerds SWAG!Become a CardioNerds Patron! Show notes - Challenging Cases - Nuances in Pulmonary Hypertension Management with Dr. Ryan Tedford Case #1 Synopsis: A woman in her late 30s presented to the hospital with 4 weeks of worsening dyspnea. Her history includes dilated non-ischemic cardiomyopathy diagnosed in the setting of a VT arrest around 10 years prior. Over the past 10 years she has been on guideline-directed medical therapy with symptoms that had been relatively controlled (characterized as NYHA Class II), but without objective improvement in her LV dimensions or ejection fraction (LVEF 15-20% by TTE and CMR and LVIDd at 6.8 cm). Over the past few months she had been noting decreased exercise tolerance, worsening orthopnea, and episodes of symptomatic hypotension at home. When she arrived to the hospital, she presented with BP 95/70 mmHg, increased respiratory effort, congestion and an overall profile consistent with SCAI Stage C-HF shock. In the case, we go through the hemodynamics at various points during her hospitalization and discuss options for management including medical therapy and mechanical support. The patient was eventually bridged to transplant with an Impella 5.5. Initial Hemodynamics Right Atrium (RA) Pressure Tracing: Right Ventricle (RV) Pressure Tracing: Pulmonary Artery (PA) Pressure Tracing: Pulmonary Capillary Wedge Pressure (PCWP) Tracing: Case 1 Rounding Pearls One of the first points that Dr. Tedford made was thinking about our classic frameworks of characterizing acute decompensated heart failure, specifically the “Stevenson” classification developed by Dr. Lynne Stevenson that phenotypes patients along two axes: congestion (wet or dry) and perfusion (warm or cold). Dr. Tedford cautioned that young patients may not fit into these classic boxes well, and that a normal lactate should not re-assure the clinician that perfusion is normal.In reviewing the waveforms, Dr. Tedford took a moment to note that besides just recording the absolute values of the pressures transduced in each chamber or vessel, it is critical to understand the morphology of the tracings themselves. For instance, with the RA pressure tracing above, there is no respiratory variation in the mean pressure. This is essentially a “resting Kussmaul’s sign,” which is typically indicative of significant RV dysfunction. Thus, even though our echocardiogram in this case did not necessarily show a significantly dilated RV with mildly reduced longitudinal function (T...
17 Jul 2022223. CCC: Approach to Acute Myocardial Infarction Cardiogenic Shock with Dr. Venu Menon00:47:56
With the advent and rapid evolution of contemporary percutaneous coronary intervention (PCI), the early invasive management of acute myocardial infarction (AMI) has become a mainstay in therapy with significant impact on patient outcomes. However, despite modern advances in technology and system-based practices, AMI presenting with cardiogenic shock (CS) continues to portend a high risk of morbidity and mortality. Few randomized controlled clinical trials are available to guide decision-making in this uniquely challenging patient population. Understanding the pathophysiologic mechanism by which injury occurs and propagates the shock cycle can be instrumental in selecting an appropriate strategy for revascularization and left ventricular unloading. In this episode we are joined by Dr. Venu Menon, The Mehdi Razavi Endowed Chair and Professor of Medicine at the Cleveland Clinic Lerner College of Medicine,  section head of clinical cardiology, fellowship program director, and director of the Cardiac intensive care unit at the Cleveland Clinic. Dr. Menon shares his wealth of knowledge and experience to help us review the contemporary data available for AMI CS management in a case-based discussion. We are also joined by Dr. Priya Kothapalli, star chief fellow and future interventionalist from University of Texas at Austin, series co-chair Dr. Yoav Karpenshif, and CardioNerds Co-founders Amit Goyal and Daniel Ambinder. Audio editing by CardioNerds Academy Intern, Dr. Christian Faaborg-Andersen. The CardioNerds Cardiac Critical Care Series is a multi-institutional collaboration made possible by contributions of stellar fellow leads and expert faculty from several programs, led by series co-chairs, Dr. Mark Belkin, Dr. Eunice Dugan, Dr. Karan Desai, and Dr. Yoav Karpenshif. Pearls • Notes • References • Guest Profiles • Production Team CardioNerds Cardiac Critical Care PageCardioNerds Episode PageCardioNerds AcademyCardionerds Healy Honor Roll CardioNerds Journal ClubSubscribe to The Heartbeat Newsletter!Check out CardioNerds SWAG!Become a CardioNerds Patron! Pearls and Quotes - Approach to Acute Myocardial Infarction Cardiogenic Shock with Dr. Venu Menon The H&P does matter! Age, location of infarction, heart rate, systolic blood pressure, and heart failure symptoms all carry weight in determining prognosis and risk of mortality.Define functional status, comorbid conditions, and life expectancy to help guide clinical decision-making. Do a quick bedside echocardiogram if possible to elucidate the predominant mechanism driving CS and rule out mechanical complications.Act with urgency! Get to the catheterization lab to characterize coronary anatomy and revascularize the culprit vessel as soon as possible.Minimize/avoid the use of vasopressors; if needed, wean as quickly as possible to avoid worsening myocardial ischemia. Consider mechanical circulatory support early!Despite dramatic advances in AMI management, data is limited in AMI CS management. Ask the important questions, get involved in the scientific inquiry as a trainee! Show notes - Approach to Acute Myocardial Infarction Cardiogenic Shock with Dr. Venu Menon 1. Why is it important to recognize AMI complicated by CS? AMI CS occurs in 7-10% of patients presenting with AMI and has a higher prevalence among elderly patients.The SHOCK trial (1999) showed significant survival benefit at 6 months with early revascularization with balloon angioplasty compared to medical therapy alone in AMI CS.Registry data suggests that early revascularization is beneficial in AMI CS even in elderly patients. Decision-making should be guided using a holistic view of the patient’s overall biology.Despite advances in revascularization techniques and availability of mechanical support, AMI CS portends a 40-45% risk of 30-day mortality in the modern era.Significant variation in management strategy exists between centers and data to guide decision-making is li...
05 Jan 20205. Hypertrophic Cardiomyopathy Historical Perspective with Dr. Edward Kasper00:48:34
Dr. Edward Kasper shares a unique historical perspective on how the field evolved as medicine learned more about HCM. He shares so many life lessons and clinical pearls that apply to HCM and the practice of medicine in general. Flutter moment by Dr. Sidney Schechet (ophthalmology). Show notes and images: https://www.cardionerds.com/hypertrophic-cardiomyopathy/
26 Jul 2022224. Atrial Fibrillation: Assessment of Stroke & Bleeding Risk with Dr. Elaine Hylek01:15:28
CardioNerds (Amit Goyal), Dr. Colin Blumenthal (CardioNerds Academy House Faculty Leader and FIT at the University of Pennsylvania), and Dr. Anjali Wagle (CardioNerds Ambassador and FIT at Johns Hopkins University), discuss the baseline assessment of stroke and bleeding risk in patients with atrial fibrillation (AF) with Dr. Elaine Hylek. Dr. Hylek is a professor of medicine at the Boston University School of Medicine and is the Director of the Thrombosis and Anticoagulation Service at Boston Medical Center. Stroke is a potentially devastating and preventable complication of AF. Understanding the balance between stroke and bleeding risk is crucial in determining who should be on anticoagulation. Join us to discuss this topic! In the next episode of the series, we will discuss situational risk assessment in the context of peri-cardioversion, peri-procedural status, triggered atrial fibrillation, and more. Audio editing by CardioNerds Academy Intern, Pace Wetstein. This CardioNerds Atrial Fibrillation series is a multi-institutional collaboration made possible by contributions of stellar fellow leads and expert faculty from several programs, led by series co-chairs, Dr. Kelly Arps and Dr. Colin Blumenthal. This series is supported by an educational grant from the Bristol Myers Squibb and Pfizer Alliance. All CardioNerds content is planned, produced, and reviewed solely by CardioNerds. We have collaborated with VCU Health to provide CME. Claim free CME here! Disclosures: Dr. Hylek discloses grant and research support from Medtronic and Janssen, and honoraria and/or consulting fees from Boehringer Ingelheim, and Bayer. CardioNerds Atrial Fibrillation PageCardioNerds Episode PageCardioNerds AcademyCardionerds Healy Honor Roll CardioNerds Journal ClubSubscribe to The Heartbeat Newsletter!Check out CardioNerds SWAG!Become a CardioNerds Patron! Pearls and Quotes - Atrial Fibrillation: Assessment of Stroke & Bleeding Risk The CHA2DS2-VASc should be used to determine stroke risk in all patients. It was updated from the CHADS2 score to better separate patients into high and low risk and a score of 0 has a very low risk of a stroke. Understanding a given model’s derivation is key to application for any risk model. Understanding who was and was not included when a risk score was derived helps determine how to clinically use it. For example, mechanical valves, hypertrophic cardiomyopathy, cardiac amyloidosis, and moderate to severe MS were all excluded or poorly represented and should receive AC in AF regardless of CV. The HAS-BLED score should be used to identify modifiable risk factors for bleeding and address them. It is less useful to determine when we should stop AC. Factors that go into the score are dynamic and the intention was to alert the provider of potentially modifiable factors that could be addressed to lower bleeding risk (such as better BP control). Fear the clot. Patients should be on AC unless there is a serious contraindication as embolic strokes can be devastating with a high mortality (~24% mortality at 30 days) “What am I saying by not writing the prescription... I am saying that it’s OK to have an ischemic stroke.” Survey data shows that patients are willing to experience 3.5 GI bleeds on average before 1 stroke, so favoring AC is often a patient centered approach Notes - Atrial Fibrillation: Assessment of Stroke & Bleeding Risk Notes drafted by Dr. Anjali Wagle 1. Why do strokes happen in atrial fibrillation? Why is reducing stroke risk so important? Atrial fibrillation is associated with a significantly increased risk of stroke. The mortality of strokes related to AF have been estimated to be around 25% at 30 days in early studies which included either persistent or permanent AF, though of note, these studied were biased towards larger strokes since the diagnosis was based on physical exam and not high resolution imaging.
31 Jul 2022225. ACHD: Advanced Heart Failure Therapies (MCS/HT) Among ACHD Patients with Dr. Rafael Alonso-Gonzalez01:09:40
CardioNerds (Daniel Ambinder) and ACHD series co-chair Dr. Dan Clark discuss advanced heart failure therapies including mechanical circulatory support (MCS) and heart transplantation (HT) in patients with adult congenital heart disease (ACHD) with Dr. Rafael Alonso-Gonzalez, cardiologist and director of Adult Congenital Heart Disease program at the University of Toronto and ACHD fellow Dr. Andy Pistner (University of Washington). They cover epidemiology of heart failure in ACHD, outcomes after HT, unique challenges of HT in this population, impact of allocation policies on access to transplantation, and regionalization of advanced heart failure care. They also discuss a practical approach to advanced heart failure therapy evaluation in ACHD. Audio editing by CardioNerds Academy Intern, student doctor Adriana Mares. The CardioNerds Adult Congenital Heart Disease (ACHD) series provides a comprehensive curriculum to dive deep into the labyrinthine world of congenital heart disease with the aim of empowering every CardioNerd to help improve the lives of people living with congenital heart disease. This series is multi-institutional collaborative project made possible by contributions of stellar fellow leads and expert faculty from several programs, led by series co-chairs, Dr. Josh Saef, Dr. Agnes Koczo, and Dr. Dan Clark. The CardioNerds Adult Congenital Heart Disease Series is developed in collaboration with the Adult Congenital Heart Association, The CHiP Network, and Heart University. See more Disclosures: None Pearls • Notes • References • Guest Profiles • Production Team CardioNerds Adult Congenital Heart Disease PageCardioNerds Episode PageCardioNerds AcademyCardionerds Healy Honor Roll CardioNerds Journal ClubSubscribe to The Heartbeat Newsletter!Check out CardioNerds SWAG!Become a CardioNerds Patron! Pearls - Advanced Heart Failure Therapies (MCS/HT) Among ACHD Patients Heart failure is a major comorbidity and the leading cause of death in adults with congenital heart disease.Identification of advanced heart failure in ACHD is challenging. ACHD patients do not always self-identify exercise limitations or exertional dyspnea. Cardiopulmonary exercise testing is a useful tool in evaluating these patients.Patients with ACHD awaiting heart transplantation are less likely than non-ACHD patients to receive a heart transplant, and ACHD patients have an increased risk of death or delisting while awaiting heart transplantation.Evaluation of transplant candidacy and potential need for multi-organ transplantation in complex congenital heart disease (i.e., Fontan palliation) requires a multidisciplinary approach.Regionalization of care improves outcomes for ACHD patients with advanced heart failure. High volume transplant centers have better early survival for ACHD patients after heart transplant, and the highest volume ACHD transplant centers in each UNOS region have better early survival. Show notes - Advanced Heart Failure Therapies (MCS/HT) Among ACHD Patients 1. How many ACHD patients have heart failure? Patients with ACHD are a large and heterogeneous group. The signs and symptoms of heart failure vary widely depending on the underlying congenital heart disease. Patients with D-transposition of the great arteries repaired with an arterial switch operation have low rates of heart failure (~3%)1 compared to those patients Fontan palliation for single ventricle physiology (40%)2. Heart failure is the leading cause of death in patients with ACHD3,4. 2. How many patients with ACHD end up receiving a heart transplant or mechanical circulatory support? Heart transplantation for congenital heart disease in adults has been increasing in frequency since the late 1980s. Between 2010 and 2012, this accounted for 4% of all adult heart transplants in the United States5. This represents a small fraction compared to the number of adults who die due to complications of heart failure ...
04 Aug 2022226. Case Report: Fall from Grace – A Case of Pulmonary Embolism – Massachusetts General Hospital01:06:57
CardioNerds (Amit and Dan) join join Dr. Andrew Dicks (Vascular medicine physician at Prisma Health, former fellow at Mass General Vascular) and Dr. Prateek Sharma (Vascular interventional & medicine fellow at MGH) for an ice-cold drinks at the Esplanade in Boston, MA to discuss a case about a patient who developed a pulmonary embolism and masterfully discuss the diagnosis and management of of pulmonary emboli. Dr. Ido Weinberg (Director, Vascular Medicine Fellowship at MGH) provides the ECPR for this episode. Case Abstract: A 59-year-old Spanish-speaking man with no significant past medical history presents after falling 15-20 feet from a ladder and landing on his back. He was found to have an L1 fracture and left radial fracture and underwent T12-L2 fusion with neurosurgery on hospital day 1 and ORIF of left radial fracture with orthopedic surgery on hospital day 2. On hospital day 5, he develops acute onset tachycardia with HR in the 130s bpm with new O2 requirement associated with mild shortness of breath at rest without any chest discomfort. His labs were notable for an elevated troponin and proBNP. He underwent CTPA which demonstrated acute bilateral occlusive pulmonary emboli (PE) extending in the right and left main pulmonary arteries. TTE demonstrated right ventricle dilation. The patient was started on a heparin infusion and a PE response team (PERT) meeting was held to discuss treatment options. Given recent surgery, use of thrombolytic therapy was felt to be too risky and thus he was taken for percutaneous thrombectomy in the cath lab. PA pressure prior to intervention was 51/21 mmHg. The patient underwent suction thromboembelectomy with the Flow Triever device with extraction of thrombus and improvement in PA pressure to 19/11 mmHg. He was treated with anticoagulation thereafter and discharged home two days after the procedure. Jump to: Case media - Case teaching - References CardioNerds Case Reports PageCardioNerds Episode PageCardioNerds AcademyCardionerds Healy Honor Roll CardioNerds Journal ClubSubscribe to The Heartbeat Newsletter!Check out CardioNerds SWAG!Become a CardioNerds Patron! Case Media Acute bilateral occlusive and nonocclusive pulmonary emboli extending from the right and left main pulmonary arteries to the lobar and segmental arteries of all the lobes.  Moderate right heart strain including the right atrium and the right ventricle. RV/LV ratio > 1.0. Right ventricular cavity is dilated (RV size at the base measures to 45mm). Right ventricular systolic function is moderately decreased. Right ventricular free wall is hypokinetic with sparing of the right ventricular apex consistent with acute right ventricular strain Pulmonary angiography demonstrated extensive proximal bilateral PEs Caption: Post-procedure TTE demonstrated resolution of RV strain with normalization of RV size and function. Episode Schematics & Teaching Pearls While there are markers to suggest PE, such as ECG findings or evidence of RV dilatation, a PE cannot be confirmed without imaging.Elevation of cardiac biomarkers and evidence of RV dysfunction are used to risk stratify PE, not the degree of thrombus burden or locale of thrombus.Enoxaparin is the preferred anticoagulant to initiate at time of PE diagnosis if comorbidities allow.Optimal treatment of intermediate risk PE remains uncertain as there is little data about long-term outcomes. Aggressive treatment should be used judiciously and chosen on a case-by-case basis.PE response teams (PERT) allow for multidisciplinary expert opinion in the face of scarce evidence to determine what is felt to be the best management strategy. Notes 1. What is a PERT team and why is it helpful? We have several tools and approaches for the management of PE. There are also many subspecialities involved in the care of patients with PE, including vascular medicine,
14 Aug 2022227. CCC: Positive Pressure Ventilation in the CICU with Dr. Sam Brusca, Dr. Chris Barnett, and Dr. Burton Lee01:05:40
The modern CICU has evolved to include patients with complex pulmonary mechanics requiring more non-invasive and mechanical ventilation. Series co-chairs Dr. Eunice Dugan and Dr. Karan Desai along with CardioNerds Co-founder Dr. Amit Goyal were joined by FIT lead, Dr. Sam Brusca, who has completed his NIH Critical Care and UCSF Cardiology fellow and currently faculty at USCF. We were fortunate enough to have two expert discussants: Dr. Burton Lee, Head of Medical Education and Global Critical Care within the National Institutes of Health Critical Care Medicine Department and master clinician educator with the ATS Scholar’s Critical Care for Non-Intensivists program, and Dr. Chris Barnett, ACC Critical Care Cardiology council member and Section Chair of Critical Care Cardiology at UCSF.  In this episode, these experts discuss the basics of mechanical ventilation, including the physiology/pathophysiology of negative and positive pressure breathing, a review of ventilator modes, and a framework for outlining the goals of mechanical ventilation. They proceed to apply these principles to patients in the CICU, specifically focusing on patients with RV predominant failure due to pulmonary hypertension and patients with LV predominant failure. Audio editing by CardioNerds Academy Intern, student doctor, Shivani Reddy. The CardioNerds Cardiac Critical Care Series is a multi-institutional collaboration made possible by contributions of stellar fellow leads and expert faculty from several programs, led by series co-chairs, Dr. Mark Belkin, Dr. Eunice Dugan, Dr. Karan Desai, and Dr. Yoav Karpenshif. Pearls • Notes • References • Production Team CardioNerds Cardiac Critical Care PageCardioNerds Episode PageCardioNerds AcademyCardionerds Healy Honor Roll CardioNerds Journal ClubSubscribe to The Heartbeat Newsletter!Check out CardioNerds SWAG!Become a CardioNerds Patron! Pearls and Quotes - Positive Pressure Ventilation in the CICU Respiratory distress, during spontaneous negative pressure breathing can lead to high transpulmonary pressures and potentially large tidal volumes. This will increase both RV afterload (by increasing pulmonary vascular resistance) and LV afterload (by increasing LV wall stress). An analogy for the impact of negative pleural pressure during spontaneous respiration on LV function is that of a person jumping over a hurdle. The height of the hurdle does not increase, but the ground starts to sink, so it is still harder to jump over. Intubation in patients with right ventricular failure is a tenuous situation, especially in patients with chronic RV failure and remodeling (increased RV thickness, perfusion predominantly during diastole, RV pressure near or higher than systemic pressure). The key tenant to safe intubation is avoiding hypotension, utilizing induction agents such as ketamine or etomidate, infusing pressors, and potentially even performing awake intubations. Non-invasive positive pressure ventilation in HFrEF has hemodynamic effects similar to a cocktail of IV inotropes, dilators, and diuretics. CPAP decreases pulmonary capillary wedge pressure (LV preload), decreases systemic vascular resistance (afterload), and increases cardiac output. Airway pressure during mechanical ventilation is based on the “equation of motion”: Pressure = Volume/Compliance + Flow*Resistance + PEEP. Our goals of oxygenation on mechanical ventilation include achieving acceptable PaO2/Sat with the lowest FiO2 possible (avoiding oxygen toxicity) and optimal PEEP (which increases oxygenation but can have detrimental impact on cardiac output) Our goals of ventilation on mechanical ventilation include achieving acceptable pH and PaCO2 while preventing ventilator induced lung injury and avoiding auto-PEEP. We prevent lung injury by reducing tidal volume (ideally <8cc/kg, plateau pressure < 30 cmH20, driving pressure < 15 cmH20) and auto-peep by reducing respiratory rate (and allowin...
17 Aug 2022228. Narratives in Cardiology: Radiation Safety & Women in Interventional Cardiology with Dr. Sheila Sahni – New Jersey Chapter00:54:15
In this episode, Daniel Ambinder (CardioNerds Co-Founder), Dr. Gurleen Kaur (Director of CardioNerds Internship and medicine resident at Brigham and Women’s Hospital), Dr. Eunice Dugan (Cardiology fellow at Cleveland Clinic) and Dr. Zarina Sharalaya (Interventional and Structural Cardiologist at North Texas Heart) learn from the Dr. Sheila Sahni (Interventional Cardiologist and Director of The Women’s Heart Program at The Sahni Heart Center) regarding radiation safety in the cath lab and methods of reducing radiation exposure to the operator. She also discusses radiation safety for the pregnant interventional cardiologist and how to safely manage pregnancy during the gestational period. We hear her inspirational journey as a female interventional cardiologist and her experience in starting the Women’s Heart Program at Sahni Heart Center. Special message by Dr. Jeff Lander, New Jersey ACC Chapter Governor. Audio editing by CardioNerds Academy Intern, Pace Wetstein. The PA-ACC & CardioNerds Narratives in Cardiology is a multimedia educational series jointly developed by the Pennsylvania Chapter ACC, the ACC Fellows in Training Section, and the CardioNerds Platform with the goal to promote diversity, equity, and inclusion in cardiology. In this series, we host inspiring faculty and fellows from various ACC chapters to discuss their areas of expertise and their individual narratives. Join us for these captivating conversations as we celebrate our differences and share our joy for practicing cardiovascular medicine. We thank our project mentors Dr. Katie Berlacher and Dr. Nosheen Reza. Video Version • Notes • Production Team Claim free CME just for enjoying this episode! There are no relevant disclosures for this episode. The PA-ACC & CardioNerds Narratives in Cardiology PageCardioNerds Episode PageCardioNerds AcademyCardionerds Healy Honor Roll CardioNerds Journal ClubSubscribe to The Heartbeat Newsletter!Check out CardioNerds SWAG!Become a CardioNerds Patron! Video version - Radiation Safety & Women in Interventional Cardiology with Dr. Sheila Sahni https://youtu.be/iIwnsu6qJ4k Tweetorial - Radiation Safety & Women in Interventional Cardiology with Dr. Sheila Sahni https://twitter.com/gurleen_kaur96/status/1563608232211296256?s=21&t=iay5zosSBDjPBLWJ4kWIAw Quoatables - Radiation Safety & Women in Interventional Cardiology with Dr. Sheila Sahni “Having anyone who can believe in you when you are really passionate about something is really all you need… the passion is what’s going to carry you through. It’s not about being male or female or pregnant or not pregnant, it’s about what you love to do and how can you master it.” “Our careers can wait, but family planning cannot. If you are fortunate enough to have the opportunity to start a family even if it’s during your training, you should”. Notes - Radiation Safety & Women in Interventional Cardiology with Dr. Sheila Sahni What are procedural techniques to utilize during a heart catheterization to reduce radiation exposure to the operator? Decrease number and length of cine acquisitions and fluoroscopy timeDecrease the frame rate - halving the frame rate reduces radiation dose by 50%Decrease the distance between the image intensifier and the patientLimit steep LAO angulationsApply collimation as much as possible which reduces overall patient dose and scatter radiationLimit digital magnification which can increase skin dose exposure by 50% What are the important dose limits to consider for a pregnant female and her fetus in the cath lab? The US Nuclear Regulatory Commission (NRC) regulatory equivalent dose limit is 5mSv during the entire pregnancy of the declared pregnant woman.The annual natural background radiation dose in the US is 3mSv.The average under-lead dose to a working pregnant interventionalist over the entire gestation is ~0.3mSv.The fetus of a working pregnant interventionalist is estimated to ...
23 Aug 2022229. CardioNerds Rounds: Challenging Cases – Atrial Fibrillation with Dr. Hugh Calkins00:42:23
It’s another session of CardioNerds Rounds! In these rounds, Dr. Stephanie Fuentes (EP FIT at Houston Methodist) joins Dr. Hugh Calkins (Professor of Medicine and Director of the Electrophysiology Laboratory and Arrhythmia Service at Johns Hopkins Hospital) to discuss the nuances of atrial fibrillation (AF) management through challenging cases. As an author of several guideline and expert consensus statements in the management of AF and renowned clinician, educator, and researcher, Dr. Calkins gives us many pearls on the management of AF, so don’t miss these #CardsRounds!  This episode is supported with unrestricted funding from Zoll LifeVest. A special thank you to Mitzy Applegate and Ivan Chevere for their production skills that help make CardioNerds Rounds such an amazing success. All CardioNerds content is planned, produced, and reviewed solely by CardioNerds. Case details are altered to protect patient health information. CardioNerds Rounds is co-chaired by Dr. Karan Desai and Dr. Natalie Stokes.  Speaker disclosures: None Challenging Cases - Atrial Fibrillation with Dr. Hugh Calkins CardioNerds Rounds PageCardioNerds Episode PageCardioNerds AcademyCardionerds Healy Honor Roll CardioNerds Journal ClubSubscribe to The Heartbeat Newsletter!Check out CardioNerds SWAG!Become a CardioNerds Patron! Show notes - Challenging Cases - Atrial Fibrillation with Dr. Hugh Calkins Case #1 Synopsis: A woman in her mid-60s presents with symptomatic paroxysmal atrial fibrillation (AF). An echocardiogram has demonstrated that she has a structurally normal heart. Her primary care doctor had started Metoprolol 50 mg twice a day but she has remained symptomatic. In office, an EKG confirms AF, but she converts to sinus while there. She is seeking advice to prevent further episodes and in general wants to avoid additional medications Case #2 Takeaways We discussed several potential options for treatment. Amongst the first things we discussed was amiodarone. In a patient of this nature without structural heart disease and under the age of 70, Dr. Calkins discussed that he would probably consider amiodarone as a 2nd line option. While amiodarone may be effective in maintaining sinus rhythm in comparison to other antiarrhythmic medications like sotalol, flecainide, and propafenone, it does have significant toxicity.If antiarrhythmic drugs (AAD) were to be considered, we also discussed the options of dofetilide versus sotalol. Dofetilide typically requires inpatient initiation due to the risk of QT prolongation and Torsades. Since women tend to have longer corrected QT (QTc) intervals, high dose dofetilide may be more proarrhythmogenic in women. Though, Dr. Calkins noted that many patients don’t tolerate sotalol due to fatigue and generally dofetilide is well tolerated.When it comes to the “pill in the pocket” approach, Dr. Calkins noted that its utility is more so in patients with persistent AF that is known to not stop on its own. For instance, an individual who has AF a few times a year that is persistent may benefit from flecainide or propafenone (“in the pocket”) instead of being brought in for an electrical cardioversion. In this scenario, the first time one of these agents is used, the patient ought to be closely monitored. For our patient, her episodes were too frequent and self-terminating for a “pill in the pocket” approach to be effective.Current guideline recommendations for catheter ablation include a Class IA recommendation for patients with paroxysmal AF refractory to AADs, and a Class IIA recommendation as first-line therapy for patients with paroxysmal AF.In the 2020 ESC Atrial Fibrillation Guidelines, catheter ablation is given a Class IA recommendation to improve symptoms of AF recurrences in patients who have failed or are intolerant of one Class I or III AADs. For patients who have failed or have been intolerant of beta blocker alone for rhythm control,
31 Aug 2022230. Case Report: A Tight Spot On The Right – Cleveland Clinic00:33:48
CardioNerds (Amit Goyal and Dan Ambinder) join Dr. Radi Zinoviev, Dr. Josh Cohen, and Dr. Tiffany Dong (CardioNerds Ambassador) from the Cleveland Clinic for a day on Edgewater beach. They discuss the following case of the evaluation and management of prosthetic tricuspid valve stenosis in a patient with a history of Ebstein Anomaly. The expert commentary and review (ECPR) is provided by Dr. Jay Ramchand, staff cardiologist with expertise in multimodality cardiovascular imaging at the Cleveland Clinic. Jump to: Case media - Case teaching - References CardioNerds Case Reports PageCardioNerds Episode PageCardioNerds AcademyCardionerds Healy Honor Roll CardioNerds Journal ClubSubscribe to The Heartbeat Newsletter!Check out CardioNerds SWAG!Become a CardioNerds Patron! Case Media CXR ECG TTE RHC Final TTE TTE 1 TTE 2 TTE 3 Follow up TTE 1 Follow up TTE 2 Episode Schematics & Teaching Pearls - Tricuspid Valve Stenosis Tricuspid stenosis is uncommon (<1% of the US population) and thus we have a lack of evidence as well as guideline recommendations.While there are no official diagnostic criteria for severe tricuspid stenosis, some echocardiographic features include flow acceleration across the valve, a mean pressure gradient of ≥ 5mmHg and an inflow VTI of > 60cm.Structural findings that support the presence of severe tricuspid stenosis include a moderately dilated RA and a dilated IVC, though these are not specific.Right heart catheterization hemodynamics that support tricuspid stenosis include a high right atrial pressure and gradual “y” descent.Bioprosthetic tricuspid valves are generally favored over mechanical valves due to risk of thrombosis and longevity of these valves in the tricuspid position. Notes - Tricuspid Valve Stenosis What are causes of tricuspid stenosis? Causes of tricuspid stenosis can be divided into congenital and acquired causes. Congenital causes include tricuspid atresia or stenosis. Acquired causes include rheumatic heart disease, carcinoid syndrome, endocarditis, prior radiation, or fibrosis from endomyocardial procedures or placement of electrical leads. Rheumatic heart disease is the most common cause of tricuspid stenosis and is usually associated with mitral valvulopathy. What are the symptoms and physical exam findings of tricuspid stenosis? Findings revolve around right sided congestion or heart failure symptoms such as peripheral edema, abdominal distension with ascites, hepatomegaly, and jugular venous distension. When examining the jugular vein, you may see prominent a-waves and an almost absent or slow y descent reflective of delayed emptying of the right atrium (in the absence of tricuspid regurgitation). The murmur of tricuspid stenosis includes an opening snap and low diastolic murmur at the left lower sternal border with inspiratory accentuation. Patients may also report fatigue due to decreased cardiac output from obstruction. On echocardiography, what are the features supportive of severe tricuspid stenosis? Qualitatively, the leaflets may be thickened with reduced mobility and there may be diastolic dooming of the valve. Doppler may show high gradients of ≥ 5 mmHg, which may be elevated if there is coexisting tricuspid regurgitation and lower with decreased cardiac output. Associated structural changes include dilated right atrium and inferior vena cava. What is expected on right heart catheterization for tricuspid stenosis? Assuming the patient remains in sinus rhythm, patients with tricuspid stenosis would display high right atrial pressures and a gradual “y” descent. A diastolic gradient may be measured with dual catheters in the right atrium and the right ventricle. What are the treatment options for tricuspid stenosis? Medical management of tricuspid stenosis includes diuretics and addressing the underlying cause.
12 Jan 20206. Hypertrophic Cardiomyopathy Surgical & Transcatheter Septal Reduction Therapies with Drs. Amar Krishnaswamy and Per Wierup00:35:24
Amit joins Dr. Laura Young to take a pulse check with experts, and learn more about the structural management of obstructive HCM. They discuss the interventional perspective with interventional cardiologist Dr. Amar Krishnaswamy and surgical perspective from cardiothoracic surgeon Dr. Per Wierup. Flutter moment by Crystal Silbak, RN. Show notes and images: https://www.cardionerds.com/hypertrophic-cardiomyopathy/
12 Sep 2022231. ACHD: Congenital Heart Disease and Psychosocial Wellbeing with Dr. Adrienne Kovacs and Dr. Lauren Lastinger01:12:41
CardioNerds Dr. Josh Saef, Dan Ambinder, join Dr. Jim Kimber and interview experts Dr. Adrienne Kovacs, and Dr. Lauren Lastinger and discuss behavioral health needs and psychosocial wellbeing in the congenital heart disease population. In this episode, our experts tackle issues surrounding mental and behavioral health including anxiety/depression, ADHD, neurodevelopmental disabilities, psychosocial challenges, stressors unique to patients with ACHD and their families, and how the healthcare system can better optimize mental health care for the CHD patient population. Audio editing by CardioNerds Academy Intern, Pace Wetstein. The CardioNerds Adult Congenital Heart Disease (ACHD) series provides a comprehensive curriculum to dive deep into the labyrinthine world of congenital heart disease with the aim of empowering every CardioNerd to help improve the lives of people living with congenital heart disease. This series is multi-institutional collaborative project made possible by contributions of stellar fellow leads and expert faculty from several programs, led by series co-chairs, Dr. Josh Saef, Dr. Agnes Koczo, and Dr. Dan Clark. The CardioNerds Adult Congenital Heart Disease Series is developed in collaboration with the Adult Congenital Heart Association, The CHiP Network, and Heart University. See more Disclosures: None Pearls • Notes • References • Guest Profiles • Production Team CardioNerds Adult Congenital Heart Disease PageCardioNerds Episode PageCardioNerds AcademyCardionerds Healy Honor Roll CardioNerds Journal ClubSubscribe to The Heartbeat Newsletter!Check out CardioNerds SWAG!Become a CardioNerds Patron! Pearls - Congenital Heart Disease and Psychosocial Wellbeing Among patients with congenital heart disease, symptoms of anxiety are more common than symptoms of depression. “Heart-focused anxiety” relates to symptoms attributable to a heart condition including fear of appointments, surgery, or health-uncertainty. It is important to differentiate this from generalized anxiety.Predictors of depression and anxiety include patient-reported physical health status. Defect severity (mild, moderate, great complexity) and physician-diagnosed NYHA class were NOT associated with rates of depression/anxiety [2].Despite CHD, patient self-reported Quality of Life (QoL) is relatively high. Predictors of decreased QoL include older age, lack of employment, never having married, and worse self-reported NYHA functional classImportant treatment strategies include: education for patients and caregivers, early identification and referral to mental health providers, incorporation of providers into CHD teams, and encouraging physical activity and peer-interaction. Show notes - Congenital Heart Disease and Psychosocial Wellbeing Notes (developed by Dr. Jim Kimber) Mental Health Terminology: Adults with CHD face the same mental health challenges as people who don’t have a heart condition. Symptoms of depression and anxiety are the most common: Approximately 1/4 - 1/3 of CHD patients will struggle with clinically significant depression or anxiety at any one point. Up to ½ will meet lifetime diagnostic criteria for these conditions Mood and anxiety disorders differ in that they have separate diagnostic criteria. Importantly, research often uses self-reported symptoms, rather than patients who have formally met diagnostic criteria. Historically, the focus has been on depression.  However, elevated symptoms of anxiety are much more common than elevated symptoms of depression. It is important to make the distinction between “Generalized Anxiety,” and “Heart-Focused Anxiety.”Heart-Focused Anxiety: symptoms of anxiety directly related to having a heart condition, such as fear of appointments / worry about a decline in health status, getting an ICD, preparing for surgery, transplants, or having a shortened life expectancy, etc. This may also include a significant component of he...
16 Sep 2022232. Case Report: A String of Pearls Not Worth Keeping – Emory University00:37:41
CardioNerds (Daniel Ambinder and Amit Goyal) join Dr. Arielle Schwartz (Emory University cardiology fellow), Dr. Joshua Zuniga (former Emory vascular medicine fellow and now USC cardiology fellow), and Dr. Patrick Zakka (UCLA cardiology fellow) from the Emory University School of Medicine. They discuss a case of a young woman with new onset hypertension refractory to 3 antihypertensive agents who is ultimately diagnosed renovascular hypertension due to fibromuscular dysplasia complicated by saccular aneurysm. Dr. Bryan Wells (Director of Vascular Medicine at Emory University) provides the ECPR for this episode. Audio editing by CardioNerds Academy intern, Dr. Christian Faaborg-Andersen. CardioNerds Case Reports PageCardioNerds Episode PageCardioNerds AcademyCardionerds Healy Honor Roll CardioNerds Journal ClubSubscribe to The Heartbeat Newsletter!Check out CardioNerds SWAG!Become a CardioNerds Patron! Case Media References Gornik HL, Persu A, Adlam D, Aparicio LS, Azizi M, Boulanger M, Bruno RM, de Leeuw P, Fendrikova-Mahlay N, Froehlich J, Ganesh SK, Gray BH, Jamison C, Januszewicz A, Jeunemaitre X, Kadian-Dodov D, Kim ES, Kovacic JC, Mace P, Morganti A, Sharma A, Southerland AM, Touzé E, van der Niepen P, Wang J, Weinberg I, Wilson S, Olin JW, Plouin PF. First International Consensus on the diagnosis and management of fibromuscular dysplasia. Vasc Med. 2019 Apr;24(2):164-189. doi: 10.1177/1358863X18821816. Epub 2019 Jan 16. Erratum in: Vasc Med. 2019 Oct;24(5):475. Erratum in: Vasc Med. 2021 Aug;26(4):NP1. PMID: 30648921. Olin, Circulation. 2014;129:1048-1078. Fibromuscular Dysplasia: State of the Science and Critical Unanswered Questions A Scientific Statement From the American Heart Association S.H.KimMD, MPH†Jeffrey W.OlinDO‡James B.FroehlichMD, MPH§XiaokuiGuMA§J. MichaelBacharachMD‖Bruce H.GrayDO¶Michael R.JaffDO#Barry T.KatzenMD∗∗EvaKline-RogersMS, RN, NP§Pamela D.MaceRN††Alan H.MatsumotoMD‡‡Robert D.McBaneMD§§Christopher J.WhiteMD‖‖Heather L.GornikMD, MHS†. Clinical Manifestations of Fibromuscular Dysplasia Vary by Patient Sex: A Report of the United States Registry for Fibromuscular Dysplasia. JACC. Volume 62, Issue 21, 19–26 November 2013, Pages 2026-2028
19 Sep 2022233. Cardio-Oncology: The Need for Cardio-Oncology with Dr. Bonnie Ky00:42:15
CardioNerds (Amit Goyal and Dan Ambinder), Series Co-Chair Dr. Dinu Balanescu (Academy House Faculty and Chief Resident at Beaumont Hospital), and Episode Lead Dr. Manu Mysore (Former CardioNerds Ambassador and Cardiologist at the University of Maryland) discuss The Need for Cardio-Oncology with Expert Faculty Dr. Bonnie Ky, Director of Penn Cardio-Oncology Translation Center of Excellence and Editor-in-Chief of JACC CardioOncology. Audio editing by CardioNerds Academy Intern, student doctor Yousif Arif. This episode is supported by a grant from Pfizer Inc. Cardio-Oncology is a burgeoning field. There is a need for cardiologists and oncologists to work together in a multidisciplinary fashion using multi-modality imaging and personalized medicine. Cardiologists in particular need to understand basic oncology, anti-cancer therapies, and address risk factors which play an important role in oncologic progression and/or adverse cardiovascular events. The field can only be furthered by research with a focus on specificity of endpoints and multidisciplinary collaboration. The future of the field is in the hands of investigators and clinicians alike. This CardioNerds Cardio-Oncology series is a multi-institutional collaboration made possible by contributions of stellar fellow leads and expert faculty from several programs, led by series co-chairs, Dr. Giselle Suero Abreu, Dr. Dinu Balanescu, and Dr. Teodora Donisan.  Pearls • Notes • References • Production Team CardioNerds Cardio-Oncology PageCardioNerds Episode PageCardioNerds AcademyCardionerds Healy Honor Roll CardioNerds Journal ClubSubscribe to The Heartbeat Newsletter!Check out CardioNerds SWAG!Become a CardioNerds Patron! Pearls and Quotes - The Need for Cardio-Oncology with Dr. Bonnie Ky Over 20 million new cancer cases are expected to be added annually to the global burden as novel therapies have improved cancer survivorship. These therapies may be directly associated with cardiotoxicity or may prolong life to allow time for cardiovascular disease to develop in cancer survivors. Hypertension, hyperlipidemia, and obesity are modifiable risk factors that portend a poor prognosis from both an oncologic and cardiovascular perspective. Multi-modality imaging is useful in risk assessment within oncology, with echocardiography (including strain imaging) having a class I indication prior to treatment with many chemotherapeutics. Diverse trial enrollment is essential for furthering the science within Cardio-Oncology to translate clinically into personalized management. There is a need to strengthen a pipeline of young physicians and scientists to further the field of Cardio-Oncology. Show notes - The Need for Cardio-Oncology with Dr. Bonnie Ky Why should cardiologists have familiarity with cancer therapies? By 2030, 23.6 million new cancer cases are expected to be added annually to the global burden.1 Novel therapies and/or combination therapies have improved cancer survivorship but are associated with cardiovascular complications, especially in the elderly and those with pre-existing cardiovascular comorbidities.2 Cardiologists currently lack an understanding of oncologic treatments, with poor knowledge of dosing protocols and cardiotoxicities. This can lead to less aggressive protocols administered, as well as early discontinuation of important treatments for both oncologic and cardiovascular conditions.3 A multidisciplinary collaboration between pharmacists, cardiologists, oncologists, and nurse navigators is needed to improve treatment decision-making for the benefit of cancer patients. Cardiologists should have basic knowledge and understanding of some of the commonly used chemotherapeutic drugs and any adverse events during treatment courses based on clinical trials, FDA reporting, and epidemiological data. JACC Cardio-Oncology seeks to disseminate knowledge through live courses such as Advancin...
23 Sep 2022234. Narratives in Cardiology: Structural Heart Disease and LatinX Representation in Cardiology with Dr. Mayra Guerrero – Minnesota Chapter00:39:39
In this episode, Daniel Ambinder and Amit Goyal (CardioNerds co-founders), Dr. Gurleen Kaur (medicine resident at Brigham and Women’s Hospital and Director of CardioNerds Internship), student doctor Adriana Mares (medical student at the University of Texas El Paso/Texas Tech University Health Sciences Center El Paso, CardioNerds Academy Intern), and Dr. Teodora Donisan (general cardiology fellow at the Mayo Clinic and CardioNerds Academy Chief) discuss with Dr. Mayra Guerrero (Interventional Cardiologist and Professor of Medicine at the Mayo Clinic) about challenges with diagnosing and treating valve disease in women, as well as ideas on how to increase recruitment for women in cardiology including interventional and structural cardiology. Dr. Guerrero shares her inspiring personal journey and advice for how to navigate becoming a structural cardiologist as an international medical graduate, woman, and mother. Audio editing by CardioNerds Academy Intern, student doctor Adriana Mares. The PA-ACC & CardioNerds Narratives in Cardiology is a multimedia educational series jointly developed by the Pennsylvania Chapter ACC, the ACC Fellows in Training Section, and the CardioNerds Platform with the goal to promote diversity, equity, and inclusion in cardiology. In this series, we host inspiring faculty and fellows from various ACC chapters to discuss their areas of expertise and their individual narratives. Join us for these captivating conversations as we celebrate our differences and share our joy for practicing cardiovascular medicine. We thank our project mentors Dr. Katie Berlacher and Dr. Nosheen Reza. Video Version • Notes • Production Team The PA-ACC & CardioNerds Narratives in Cardiology PageCardioNerds Episode PageCardioNerds AcademyCardionerds Healy Honor Roll CardioNerds Journal ClubSubscribe to The Heartbeat Newsletter!Check out CardioNerds SWAG!Become a CardioNerds Patron! Video version - Structural Heart Disease and LatinX Representation in Cardiology with Dr. Mayra Guerrero https://youtu.be/KvKADqUwUHQ Quoatables - Structural Heart Disease and LatinX Representation in Cardiology with Dr. Mayra Guerrero “Work hard, give it your best, and your work will speak for itself. Don’t be afraid to work hard and you’ll be able to achieve anything you want.”“I’m very fortunate to have had the opportunities that I’ve had, but now it’s my responsibility and the responsibility of many to make sure that we create those opportunities and that we provide mentorship for others who may want to follow the same steps into this field.”“I get angry, it’s normal to have emotions, but what I’ve learned is to transform my anger into something good – think of a project, find a paper, do something good for your career…channel that energy to do something good.”“It’s important that even at young ages you start thinking about how to pay it forward.”“Don’t wait too long to have kids. There’s never a perfect time to be a parent. Once you decide to have a family don’t put a pause on your personal life for your career.” Notes - Structural Heart Disease and LatinX Representation in Cardiology with Dr. Mayra Guerrero Notes (by Dr. Teodora Donisan) Structural valve disease in women and valve care in the global settingHeart disease is the leading cause of death for women. However, the awareness regarding this major public health concern has been declining over the past decade. Valve disease awareness is one of the lowest, at less than 3%.Women have higher mortality than men when they undergo surgical aortic or mitral interventions, mainly because of a higher risk profile. For example, women with severe aortic stenosis usually present at older ages and have many associated comorbidities, however the outcomes are good when they are treated with transcatheter aortic valve replacement (TAVR). Despite this, women are less likely to be referred for aortic valve replacement (AVR) than men.
30 Sep 2022235. CCC: Post-cardiotomy Shock with Dr. Gavin Hickey and Dr. David Kaczorowski00:28:09
In this episode, Dr. Carly Fabrizio (Advanced Heart Failure and Transplant Cardiology Physician at Christiana Care Hospital), CardioNerds Critical Care Series Co-Chair Dr. Mark Belkin (Advanced Heart Failure and Transplant Fellow at University of Chicago) and CardioNerds Co-Founder Dr. Amit Goyal (Cleveland Clinic) join Dr. Gavin Hickey (Director of the AHFTC Fellowship and medical director of the left ventricular assist device program at UPMC) and Dr. David Kaczorowski (Surgical Director for the Advanced Heart Failure center, Department of Cardiothoracic Surgery at UPMC) for a discussion on post-cardiotomy shock. Audio editing by CardioNerds Academy Intern, student doctor, Shivani Reddy. Post-cardiotomy shock is characterized by heart failure that results in the inability to wean from cardiopulmonary bypass or develops post cardiac surgery. Patients who develop post-cardiotomy shock typically require inotropic support and may ultimately require temporary mechanical circulatory support. Post-cardiotomy shock carries a high mortality rate. However, early recognition and prevention strategies can help mitigate the risk for developing post-cardiotomy shock. The CardioNerds Cardiac Critical Care Series is a multi-institutional collaboration made possible by contributions of stellar fellow leads and expert faculty from several programs, led by series co-chairs, Dr. Mark Belkin, Dr. Eunice Dugan, Dr. Karan Desai, and Dr. Yoav Karpenshif. Pearls • Notes • References • Production Team CardioNerds Cardiac Critical Care PageCardioNerds Episode PageCardioNerds AcademyCardionerds Healy Honor Roll CardioNerds Journal ClubSubscribe to The Heartbeat Newsletter!Check out CardioNerds SWAG!Become a CardioNerds Patron! Pearls and Quotes - Post-cardiotomy Shock Weaning from cardiopulmonary bypass is an intricate process that includes: rewarming the patient, de-airing the cardiac chambers, ensuring a perfusing heart rhythm, confirming adequate ventilation and oxygenation, removing the intracardiac catheters and cannulas and slowly reducing the blood diverted to the cardiopulmonary circuit and returning it small aliquots to the patient. Much to monitor during the process! Assessing the risk for post-cardiotomy shock prior to going to the OR is important. Consider left ventricular, right ventricular, and valvular function, and don’t forget about the value of hemodynamic assessments (pulmonary artery catheter evaluations) to ensure patients are adequately compensated. Close peri-operative monitoring of hemodynamics, hemo-metabolic derangements, and acid/base status can help identify patients who are failing therapy and may require upgrade to temporary MCS. RV assessment is challenging. Utilizing both imaging and hemodynamic evaluations can help understand which RV’s will require more support. Multi-disciplinary discussions with a heart team approach prior to cardiac surgery are valuable in identifying high risk patients for post cardiotomy shock and discussing contingency plans if issues arise. Show notes - Post-cardiotomy Shock (drafted by Dr. Carly Fabrizio) How can we diagnose post cardiotomy shock?We can diagnose post cardiotomy shock as patients who are undergoing cardiac surgery that develop hypotension and or tachycardia with hypoperfusion and end organ dysfunction. How can assess the risk of developing postcardiotomy shock prior to going to the OR?LV systolic function is not the only evaluation of cardiac functionDon’t ignore the RV!Valvular function must be evaluated in conjunction with LV/RV functionHemodynamics can be helpful prior to going to the ORFilling pressures and CO/CI evaluation --> the more normal range - the less risk of post cardiotomy shockIf going in more deranged --> more complications are likely to occurThink about what options are available post operatively if issues ariseInclude a multi-disciplinary discussions and planning prior to going to the OR ...
06 Oct 2022236. CardioNerds Rounds: Challenging Cases – Mitral Regurgitation with Dr. Rick Nishimura00:45:51
It’s another session of CardioNerds Rounds! In these rounds, Dr. Natalie Stokes (Formerly FIT at University of Pittsburgh and now General Cardiology Faculty at University of Pittsburgh) and Dr. Karan Desai (formerly FIT at University of Maryland and now General Cardiology faculty at Johns Hopkins) join Dr. Rick Nishimura (Professor of Medicine at Mayo Clinic) to discuss the nuances of managing mitral regurgitation through real cases. Dr. Nishimura has been an author or Chair of the ACC/AHA valve guidelines going back 20 years and has been recognized internationally as one of the world’s best educators, so you don’t want to miss the #NishFactor on these #CardsRounds! Audio editing by CardioNerds academy intern, Pace Wetstein. This episode is supported with unrestricted funding from Zoll LifeVest. A special thank you to Mitzy Applegate and Ivan Chevere for their production skills that help make CardioNerds Rounds such an amazing success. All CardioNerds content is planned, produced, and reviewed solely by CardioNerds. Case details are altered to protect patient health information. CardioNerds Rounds is co-chaired by Dr. Karan Desai and Dr. Natalie Stokes.  Speaker disclosures: None Challenging Cases - Atrial Fibrillation with Dr. Hugh Calkins CardioNerds Rounds PageCardioNerds Episode PageCardioNerds AcademyCardionerds Healy Honor Roll CardioNerds Journal ClubSubscribe to The Heartbeat Newsletter!Check out CardioNerds SWAG!Become a CardioNerds Patron! Show notes - Mitral Regurgitation with Dr. Rick Nishimura Case #1 Synopsis: A man in his 70s with a history of non-ischemic cardiomyopathy (last known LVEF 15-20%) and atrial fibrillation, presented with decompensated heart failure in the setting of moderate to severe mitral regurgitation. He was diuresed, transitioned to GDMT, and referred to cardiac rehabilitation. Over the next 6 months, he continued to have debilitating dyspnea (NHYA Class IIIa) and his outpatient physicians were limited on titrating GDMT further due to hypotension. A TEE was done which demonstrated EF 15%, severe MR by color and quantitation (EROA of 0.5 cm2; Regurgitant Volume of 65 mL), systolic flow reversal in the pulmonary vein and severe tricuspid regurgitation. We were asked how we would approach this case Case #1Takeaways In attempting to keep the evaluation of chronic mitral regurgitation relatively simple, we should ask ourselves three primary questions: (1) What is causing the MR; (2) How much MR is there; and (3) What is the hemodynamic consequence of the MR.To the first question of what is the etiology of the MR – a simple framework is to think of the etiology as an issue of the valve (primary) or an issue of the ventricle/atria (secondary). There is further classification that can be made based on the Carpentier Classification which speaks to the valve leaflet movement and position (normal leaflet motion, excessive leaflet motion [e.g., prolapse], or restricted in systole and/or diastole [e.g., rheumatic heart disease]).During rounds, Dr. Nishimura provided some historical context in that the original valve guidelines had recommendations for intervention on primary mitral regurgitation and not secondary – given that it is considered a disease of the ventricle. Trials like the COAPT trial have greatly shifted our practice in treating secondary mitral regurgitation. Though, we have to be familiar with which patients with secondary MR would truly derive benefit from mitral valve interventionIn regards to the COAPT trial, patients with moderate to severe (3+) or severe (4+) mitral regurgitation who remained symptomatic despite maximally tolerated guideline-directed medical therapy (GDMT) were included. Dr. Nishimura makes the point that about one-third of patients intended to be enrolled in the trial were not included because they improved so much on GDMT. And thus, when evaluating patients for consideration of mitral valve intervention in secondary MR – a...
08 Oct 2022237. CCC: Approach to LV Predominant Cardiogenic Shock with Dr. Shashank Sinha00:43:15
Cardiogenic shock (CS) remains a complex, multifactorial syndrome associated with significant morbidity and mortality. The CardioNerds Critical Care Cardiology Series tackles this important syndrome in a series of several episodes including: LV-predominant Shock, RV-predominant Shock, and Bi-ventricular Shock. In this episode, we review the definitions, pathophysiology, evaluation, and contemporary management, including use of inotropes and mechanical circulatory support, of left ventricular (LV) predominant CS. Series co-chairs Dr. Eunice Dugan and Dr. Karan Desai along with CardioNerds Co-founders Dr. Amit Goyal and Dr. Daniel Ambinder were joined by FIT lead, Dr. Vanessa Blumer, the recipient of the AHA 2021 Laennec Fellow in Training Clinician Award and currently pursuing Advanced Heart Failure and Transplant fellowship at the Cleveland Clinic. Our episode expert is Dr. Shashank Sinha, an Advanced Heart Failure, Mechanical Circulatory Support, and Cardiac Transplant cardiologist, Medical Director of the Cardiac Intensive Care Unit, and Director of the Cardiovascular Critical Care Research Program at INOVA Fairfax Hospital. His illustrious career accomplishments include being a Steering Committee member and site Principal Investigator for the multicenter Cardiogenic Shock Working Group and Critical Care Cardiology Trials Network. Audio editing by CardioNerds academy intern, Anusha Gandhi. The CardioNerds Cardiac Critical Care Series is a multi-institutional collaboration made possible by contributions of stellar fellow leads and expert faculty from several programs, led by series co-chairs, Dr. Mark Belkin, Dr. Eunice Dugan, Dr. Karan Desai, and Dr. Yoav Karpenshif. Pearls • Notes • References • Production Team CardioNerds Cardiac Critical Care PageCardioNerds Episode PageCardioNerds AcademyCardionerds Healy Honor Roll CardioNerds Journal ClubSubscribe to The Heartbeat Newsletter!Check out CardioNerds SWAG!Become a CardioNerds Patron! Pearls and Quotes - LV Predominant Cardiogenic Shock LV-CS is complex! It is important to recognize that the pathophysiology of heart failure-related cardiogenic shock (HF-CS) is distinct from that of acute myocardial infarction (AMI-CS), and also crucial to differentiate between LV-dominant, right ventricular (RV)-dominant and biventricular (BiV)-shock.The SCAI SHOCK Stage Classification provides a unified and standardized vocabulary when assessing severity of CS, and facilitates communication about the diagnosis, presentation, and evolving nature of CS.Norepinephrine is considered the initial vasopressor of choice in most CS patients; the initial inotrope choice is a bit more nuanced!When considering mechanical circulatory support (MCS) for LV shock, high-quality data to guide therapy is lacking but one must always consider “the right patient, for the right device, at the right time” and remember that “pumps pump blood, decisions save lives”.Multidisciplinary, team-based care is paramount to improving survival of the critically ill patient with CS. Show notes - LV Predominant Cardiogenic Shock Notes drafted by Dr. Vanessa Blumer. 1. What tools do you use to define LV CS? CS is a hemodynamically complex and multifactorial syndrome, one of the most common indications for admission to a cardiac intensive care unit, with short-term mortality ranging from 35-50%.It is defined by systemic hypoperfusion and tissue hypoxia due to a primary cardiac insult or dysfunction.Clinical criteria used to define CS typically include evidence of hypotension (classically defined as SBP < 90 mmHg for 30 minutes and/or use of vasopressors, inotropes, or MCS to maintain systolic blood pressure > 90 mmHg) AND evidence of end-organ hypoperfusion (for example, serum lactic acid > 2 mmol/L, acute kidney injury, acute liver injury, altered mental status) in the setting of acute coronary syndrome or acute decompensated heart failure.Laboratory markers,
13 Oct 2022238. Cardio-Oncology: Radiation-Associated Cardiovascular Disease with Dr. Eric Yang01:02:34
CardioNerds (Dr. Patrick Azcarate, Dr. Teodora Donisan, and Amit Goyal) discuss Radiation-Associated Cardiovascular Disease (RACD) with Dr. Eric Yang, cardio-oncologist, assistant professor of medicine, and associate fellowship program director at UCLA. RACD is a consequence of radiation treatment for various mediastinal tumors (breast, lung, lymphoma). It is the second most common cause of morbidity and mortality in patients treated with mediastinal radiation for cancer. While novel techniques decrease radiation exposure during cancer treatment, the incidence is expected to increase because of historical practices and delayed onset of symptoms. The prevalence of RACD is difficult to estimate given under-recognition. Additionally, most of the data comes from patients treated with radiation techniques from decades ago. In this discussion we review every nook and cranny of RACD to help guide you the next time you see a patient with a history of chest radiation. Review this CardioNerds Case Report of radiation associated cardiovascular disease for more: Episode #169. Chest pain in a Young Man – “A Gray (Gy) Area” – UC San Diego. Audio editing by CardioNerds Academy Intern, student doctor Yousif Arif. This episode is supported by a grant from Pfizer Inc. This CardioNerds Cardio-Oncology series is a multi-institutional collaboration made possible by contributions of stellar fellow leads and expert faculty from several programs, led by series co-chairs, Dr. Giselle Suero Abreu, Dr. Dinu Balanescu, and Dr. Teodora Donisan.  Pearls • Notes • References • Production Team CardioNerds Cardio-Oncology PageCardioNerds Episode PageCardioNerds AcademyCardionerds Healy Honor Roll CardioNerds Journal ClubSubscribe to The Heartbeat Newsletter!Check out CardioNerds SWAG!Become a CardioNerds Patron! Pearls and Quotes - Radiation-Associated cardiovascular disease Due to the legacy effect, the incidence of RACD will continue to increase in the next few years. When treating patients with a history of mediastinal radiation, we should remember to ask: How much radiation was given? Could the heart have been exposed? Radiation can affect every part of the heart by causing coronary artery disease (CAD), valvulopathy, myocardial disease, conduction disease, and pericardial disease. Exposure to ~25-30 Gy or more significantly increases the risk but RACD can occur at lower doses. Try to delay surgery as much as possible and do all you can in one operation to avoid re-operation in the future. For revascularization, percutaneous coronary intervention (PCI) is typically preferred over coronary artery bypass grafting (CABG) but the choice should be individualized in consultation with a multidisciplinary heart team experienced in the management of RACD. In general, for aortic valve disease, transcatheter replacement is recommended over surgical aortic valve replacement. For mitral valve disease, surgical replacement is recommended over repair. Every decision should be made with a heart team approach and made unique to that specific patient. Show notes - Radiation-Associated cardiovascular disease Notes were drafted by Dr. Patrick Azkarate. 1. Understand the pathophysiology of RACD Ionizing radiation has the potential to damage DNA. Both normal cells and cancer cells get damaged, but cancer has less effective DNA repair mechanisms and therefore malignant cells are more vulnerable to radiation therapy. After radiation causes acute damage, this sets off an inflammatory cascade leading to myofibroblast activation, fibrosis and collagen deposition, and subsequent stiffening of the myocardium and vessels. 2. What may increase one’s risk of developing RACD? Young age (<50 years-old) at the time of radiation High cumulative dose (>30 Gy) or high dose of radiation fractions (>2 Gy/day) Anterior or left chest radiation (breast cancer, lung cancer, lymphoma)
23 Oct 2022239. CCC: Approach to RV Predominant Cardiogenic Shock with Dr. Ryan Tedford00:38:10
CardioNerd (Daniel Ambinder) and series co-chairs Mark Belkin (AHFT Fellow, University of Chicago) and Karan Desai (Cardiologist, Johns Hopkins), join fellow lead, Dr. Pablo Sanchez (FIT, Stanford) for a discussion with Dr. Ryan Tedford (Professor of Medicine at the Medical University of South Carolina) about Right Ventricular (RV) predominant cardiogenic shock. In this episode we explore risk factors, pathophysiology, hemodynamics, and treatment strategies in this common and complex problem. We dissect three cases that epitomize the range of diagnostic dilemmas and management decisions in RV predominant shock, as Dr. Tedford expertly weaves us through the pathophysiology and decision-making involved in managing the “people’s ventricle.” Audio editing by Dr. Gurleen Kaur (Director of the CardioNerds internship program, CardioNerds academy fellow, and IM resident at Brigham and Women’s Hospital). The CardioNerds Cardiac Critical Care Series is a multi-institutional collaboration made possible by contributions of stellar fellow leads and expert faculty from several programs, led by series co-chairs, Dr. Mark Belkin, Dr. Eunice Dugan, Dr. Karan Desai, and Dr. Yoav Karpenshif. Pearls • Notes • References • Production Team CardioNerds Cardiac Critical Care PageCardioNerds Episode PageCardioNerds AcademyCardionerds Healy Honor Roll CardioNerds Journal ClubSubscribe to The Heartbeat Newsletter!Check out CardioNerds SWAG!Become a CardioNerds Patron! Pearls and Quotes - RV Predominant Cardiogenic Shock The degree of RV dysfunction and failure are modulated by stretching its capacity to tolerate insults from deranged afterload, preload, and contractility.Afterload insults are MUCH LESS tolerated than other insults and broadly comprise the most common pathophysiologic cause of both acute and chronic RV failure.RV and left ventricular (LV) function are anatomically and physiologically connected.  Progressive derangements in RV function can lead to the deadly “RV spiral,” in which poor RV function causes lower LV preload, leading to hypotension, and thus worsening RV perfusion and function.In RV failure/shock, some basic tenets including treating reversible causes, optimizing preload and afterload, and using inotropes and/or temporary MCS for as limited time as possible.Many acute RV failure patients can recover, but multiorgan injury plays an important role. Therefore, thoughtful and expeditious use of mechanical circulatory support is important. Show notes - RV Predominant Cardiogenic Shock Notes drafted by Dr. Pablo Sanchez. What is the basic difference between RV dysfunction and failure?Dysfunction: Abnormalities in systolic/diastolic function of the RV, but not necessarily to the point of leading to end-organ perfusion defects. RV dysfunction leads to poor outcomes regardless of mechanism.1Failure: Clinical syndrome of inability of RV to maintain adequate output despite adequate preload. 1 How is the RV different from the LV and what impact does it have on pathophysiology and hemodynamics?The LV and RV originate from different embryologic “heart fields.”1,2The RV wall is thinner and more compliant and has only two layers (instead of 3 like the LV).3 Furthermore, unlike the LV which has a significant proportion of endocardial and epicardial transverse myocardial fibers, the RV myocardial fibers are aligned in a longitudinal plane for the most part. Thus, a more significant proportion of RV systolic contraction is longitudinal – base of the ventricle moving towards the apex.The RV is crescent-shaped and has a large surface-to-volume ratio meaning smaller inward motion ejects the same stroke volume. 1Hemodynamically, the RV takes blood from a low-pressure venous system and gives it to a distensible system with low impedance (the normal pulmonary circuit at baseline typically has a resistance one-tenth of the systemic resistance). Therefore, volume loads (preload) are much better handled...
03 Nov 2022240. CardioNerds Rounds: Challenging Cases – Antithrombotic Management with Dr. Deepak Bhatt00:46:54
It’s another session of CardioNerds Rounds! In these rounds, Dr. Priya Kothapalli (Interventional FIT at University of Texas at Auston, Dell Medical School) joins Dr. Deepak Bhatt (Dr. Valentin Fuster Professor of Medicine and Director of Mount Sinai Heart) to discuss the nuances of antithrombotic therapy. As one of the most prolific cardiovascular researchers, clinicians, and educators, CardioNerds is honored to have Dr. Bhatt on Rounds, especially given that Dr. Bhatt has led numerous breakthroughs in antithrombotic therapy. Come round with us today by listening to the episodes of #CardsRounds! Audio editing by CardioNerds Academy Intern, Dr. Christian Faaborg-Andersen. This episode is supported with unrestricted funding from Zoll LifeVest. A special thank you to Mitzy Applegate and Ivan Chevere for their production skills that help make CardioNerds Rounds such an amazing success. All CardioNerds content is planned, produced, and reviewed solely by CardioNerds. Case details are altered to protect patient health information. CardioNerds Rounds is co-chaired by Dr. Karan Desai and Dr. Natalie Stokes.  Speaker disclosures: None Challenging Cases - Atrial Fibrillation with Dr. Hugh Calkins CardioNerds Rounds PageCardioNerds Episode PageCardioNerds AcademyCardionerds Healy Honor Roll CardioNerds Journal ClubSubscribe to The Heartbeat Newsletter!Check out CardioNerds SWAG!Become a CardioNerds Patron! Show notes - Antithrombotic Management with Dr. Deepak Bhatt Case #1 Synopsis: A woman in her early 70s with a history of hypertension, hyperlipidemia, and paroxysmal atrial fibrillation presented with sudden-onset chest pressure and diaphoresis while at rest and was found to have an acute thrombotic 99% mid-LAD occlusion. The patient received OCT-guided PCI with a single drug-eluting stent. We discussed what the appropriate antithrombotic strategy would be for a patient with recent acute coronary syndrome and atrial fibrillation. Case #1Takeaways According to the recent 2021 revascularization guidelines, in patients with atrial fibrillation undergoing PCI and taking oral anticoagulant therapy, it is recommended to discontinue aspirin after 1 to 4 weeks while maintaining P2Y12 inhibitors in addition to a non-vitamin K oral anticoagulant or warfarin.There are two recent trials – AUGUSTUS and the ENTRUST-AF PCI trial – that evaluated regimens of apixaban and edoxaban, respectively, that support earlier findings reporting lower bleeding rates in patients maintained on oral anticoagulant plus a P2Y12 inhibitor compared to triple therapy.Of note, none of these trials were specifically powered for ischemic endpoints, but when pooling data from these trials, rates of death, MI and stent thrombosis with dual therapy were similar to those seen in patients on triple therapy.Additionally, all of these patients enrolled in these trials were briefly treated with triple therapy after PCI before the aspirin was discontinued. In the 2021 guidelines, it is noted that analyses of stent thrombosis suggest that 80% of events occur within 30 days of PCI. Thus, it is reasonable to consider extending triply therapy to 1 month after PCI in high risk patients to reduce risk of stent thromboses.In AUGUSTUS, 90% of patients received clopidogrel as their P2Y12 inhibitor Case #2 Synopsis: A man in his mid-50s with a history of peripheral vascular disease with prior SFA stent for chronic limb ischemia, hyperlipidemia, tobacco use, diabetes, and chronic kidney disease presented with a two day history of “reflux” that was worse with exertion and that improved with rest and associated with diaphoresis. He was diagnosed with an NSTEMI. His LHC revealed 99% mid-RCA thrombotic occlusion with moderate disease in the LAD. He underwent thrombectomy and PCI with a single drug-eluting stent to the RCA. We discussed his short-term and long-term antithrombotic therapy Case #2 Takeaways
11 Nov 2022241. Case Report: A Massive Surprise – UCLA00:41:48
CardioNerds Cofounder Dr. Amit Goyal is joined by an esteemed group of UCLA cardiology fellows – Dr. Patrick Zakka (CardioNerds Academy Chief), Dr. Negeen Shehandeh (Chief Fellow), and Dr. Adrian Castillo – to discuss a case of primary cardiac angiosarcoma. An expert commentary is provided by Dr. Eric Yang, beloved educator, associate clinical professor of medicine, assistant fellowship program director, and founder of the Cardio-Oncology program at UCLA.   Case synopsis: A female in her 40s presents to the ED for fatigue that had been ongoing for approximately 1 month. She also developed night sweats and diffuse joint pains, for which she has been taking NSAIDs. She was seen by her PCP and after bloodwork was done, was told she had iron deficiency so was on iron replacement therapy. Vital signs were within normal limits. She was in no acute distress. Her pulmonary and cardiac exams were unremarkable. Her lab studies showed a Hb of 6.6 (MCV 59) and platelet count of 686k. CXR was without significant abnormality, and EKG showed normal sinus rhythm. She was admitted to medicine and received IV iron (had not consented to receiving RBC transfusion). GI was consulted for anemia work-up. Meanwhile, she developed a new-onset atrial fibrillation with rapid ventricular response seen on telemetry, for which Cardiology was consulted. A TTE was ordered in part of her evaluation, and surprisingly noted a moderate pericardial effusion circumferential to the heart. Within the pericardial space, posterior to the heart and abutting the RA/RV was a large mass measuring approximately 5.5x5.9 cm. After further imaging work-up with CMR and PET-CT, the mass was surgically resected, and patient established care with outpatient oncology for chemotherapy.  CardioNerds Case Reports PageCardioNerds Episode PageCardioNerds AcademyCardionerds Healy Honor Roll CardioNerds Journal ClubSubscribe to The Heartbeat Newsletter!Check out CardioNerds SWAG!Become a CardioNerds Patron! Case Media - primary cardiac angiosarcoma Episode Schematics & Teaching Pearls – primary cardiac angiosarcoma The pericardium is composed of an outer fibrous sac, and an inner serous sac with visceral and parietal layers.   Pericardial masses can be primary (benign or malignant) or metastatic. There are other miscellaneous pericardial masses.  Imaging modalities for the pericardium include echocardiography, cardiac CT and cardiac MRI. There is also role for PET-CT in pericardial imaging for further characterization of pericardial masses.   Cardiac angiosarcomas are extremely rare but are the most common cardiac primary malignant tumors.  Evidence-based management if lacking because of paucity of clinical data given the rarity of cardiac angiosarcomas. Surgery is the mainstay of therapy. Radiotherapy and chemotherapy are often used as well.  Notes – primary cardiac angiosarcoma Pericardial Anatomy  The pericardium is a fibroelastic sac composed of two layers.   Outer layer: fibrous pericardium (<2 mm thick)  Inner layer: serous pericardium, two-layered sac.  Visceral pericardium: adherent to underlying myocardium Parietal pericardium: lines fibrous sac.  Between the serous layers, there is the pericardial cavity which normally contains up to 50 cc pericardial fluid.  Pericardial Masses  Benign  Lipoma: slow-growing, collection of adipose cells, thought to arise in AV groove Teratoma: benign germ cell tumors, often right sided. Can cause compressive symptoms of RA, SVC, PA, aortic root.  Fibroma: solid mass of connective tissue Hemangioma: vascular mass, often arising from visceral pericardium  Malignant  Sarcoma: various types including angiosarcoma and liposarcoma.  Lymphoma: usually non-Hodgkin B-cell lymphoma, often in immunocompromised patients Mesothelioma: no apparent association with asbestos. Pericardial effusions with nodules/plaques are seen. 
15 Nov 2022242. ACHD: Atrial Septal Defects with Dr. Richard Krasuski01:15:10
CardioNerds (Dan Ambinder), episode lead Dr. Sarah Fahnhorst (ACHD Cardiologist at Spectrum Health in Grand Rapids, Michigan), and series co-chair Dr. Agnes Koczo (fellow at UPMC) learn about ASD from Dr. Richard Krasuski (ACHD Cardiologist and Director of ACHD at Duke University). Audio editing by CardioNerds Academy Intern, student doctor Adriana Mares An atrial septal defect (ASD) is a common congenital heart disease most often diagnosed in childhood, but initial presentation can be in adulthood. ASDs are abnormal communications between the left and the right atrium.  There are four types of ASDs with different embryologic origins. If the defects are large, they will require percutaneous or surgical closure. Unrepaired defects can lead to symptoms of shortness of breath, exercise intolerance, recurrent chest infections, or pulmonary hypertension. This episode of CardioNerds will review the natural history, embryologic origin, diagnostic modalities/findings, indication for closure and long term complications of repaired and unrepaired atrial septal defects.  The CardioNerds Adult Congenital Heart Disease (ACHD) series provides a comprehensive curriculum to dive deep into the labyrinthine world of congenital heart disease with the aim of empowering every CardioNerd to help improve the lives of people living with congenital heart disease. This series is multi-institutional collaborative project made possible by contributions of stellar fellow leads and expert faculty from several programs, led by series co-chairs, Dr. Josh Saef, Dr. Agnes Koczo, and Dr. Dan Clark. The CardioNerds Adult Congenital Heart Disease Series is developed in collaboration with the Adult Congenital Heart Association, The CHiP Network, and Heart University. See more Disclosures: None Pearls • Notes • References • Guest Profiles • Production Team CardioNerds Adult Congenital Heart Disease PageCardioNerds Episode PageCardioNerds AcademyCardionerds Healy Honor Roll CardioNerds Journal ClubSubscribe to The Heartbeat Newsletter!Check out CardioNerds SWAG!Become a CardioNerds Patron! Pearls - Atrial Septal Defects It’s a CLASSIC! – On physical exam a wide fixed split S2 along with a systolic ejection murmur due to increased blood flow across the pulmonary valve and potentially a diastolic rumble across the tricuspid valve are CLASSIC findings with atrial septal defects. Atrial septal defects are not all the same. There are four types of atrial septal defects: secundum ASD, primum ASD, sinus venosus and coronary sinus defects (NOTE – the latter are atrial level defects which actually do not involve the interatrial septum). The different types warrant a different approach to closure. Use your tools and if your suspicion is high for an atrial septal defect, keep looking. Sinus venosus defects can easily be missed since the superior vena cava is difficult to image in adults. Diagnostic tools include: history and physical exam (USE the stethoscope), ECG, echocardiogram, cardiac MRI, cardiac CT, and cardiac catheterization.Not all defects NEED to be closed immediately. Moderate-large defects with a shunt greater than 1.5:1 should be closed due to increased risk of pulmonary hypertension and arrhythmias, barring contraindications. Surgery was previously the gold standard for closure of ASDs, but many defects especially secundum atrial septal defects are closed in the cath lab.    Show notes - Atrial Septal Defects Notes (developed by Dr. Sarah Fahnhorst What are the four different types of atrial level defects?Secundum atrial septal defectMost common type of atrial septal defect (75%)Located in the center of the atrial septum (fossa ovalis)Hole in the primum septum due to deficiency of the septum secundumPrimum atrial septal defectAccounts for 15-20% of ASDLocated at the inferior portion of the atrial septumIn the spectrum of atrioventricular septal defects/endocardial cushion defectsDefect in the developme...
20 Nov 2022243. Case Report: A Rare Case of Isolated Non-Compaction Cardiomyopathy – Mount Sinai Medical Center in Miami00:46:02
CardioNerds Cofounder Dr. Amit Goyal is joined by Dr. Douglas Salguero (Internal medicine resident), Dr. Francisco Ujueta (Cardiology fellow), and Dr. Priscilla Wessly (Chief cardiology fellow) from the Columbia University Division of Cardiology at Mount Sinai Medical Center in Miami to discuss a rare case of isolated non-compaction cardiomyopathy. Expert commentary is provided by Dr. Christos Mihos (Director, Echocardiography Laboratory, Columbia University Division of Cardiology, Mount Sinai Medical Center). Audio editing by CardioNerds Academy Intern, Shivani Reddy.   CardioNerds Case Reports PageCardioNerds Episode PageCardioNerds AcademyCardionerds Healy Honor Roll CardioNerds Journal ClubSubscribe to The Heartbeat Newsletter!Check out CardioNerds SWAG!Become a CardioNerds Patron! Case Media - Non-Compaction Cardiomyopathy Episode Schematics & Teaching The etiology has been a constant debate since 1980. It has been debated among researchers and clinicians whether LVNC is a physiologic or a pathologic manifestation. Waning et al., classified 327 unrelated patients into 3 categories: 1) genetic, 2) probably genetic, or 3) sporadic, identifying the most common mutations: MYH7, MYBPC3 and TTN in the genetic LVNC patients, which mostly encode for sarcomere, Z-disc and nuclear-envelope proteins. This supports the hypothesis that the inherited phenotype can arise from a gene mutation possibly during embryogenesis, disrupting the physiologic compaction of normally developing myocardium, which progresses from the base to the apex of the cardiac tissue. It is estimated that genetic LVNC accounts approximately 18-44% of cases, with autosomal dominant transmission being the most common form of inheritance. Physiologic remodeling with prominent trabeculations may be noted in athletes and pregnant women, in comparison to pathologic remodeling which may be encountered in patients with cardiomyopathy (e.g. pressure or volume load).  (1) There is no pathognomonic signs or symptoms in LVNC. LVNC patients may encounter various potential clinical characteristics. Presentations are myriad and include heart failure symptoms (HFrEF or HFpEF), ventricular tachycardia (VT/VF), atrial fibrillation, thromboembolism including cerebrovascular accident (CVA), and syncope. In a cohort of 95 probands with LVNC investigated in Europe, as many as 32.3% had an ICD/CRT-D implantation, with 11.8% experiencing a cardiovascular death and 18.2% having an appropriate ICD shock. (2) Imaging plays a key role in diagnosis for LVNC. The identification and diagnosis of LVNC is evaluated using 2D echocardiography. The initial proposed method by Chin et al., evaluated the size of the trabeculation in the center. (3) The most commonly used criteria, Jenni et al. (4), entail the following four finding: Two-layer structure, with a thin compacted layer and a thick non-compacted layer measure at end-systole at the parasternal short-axis view. LVNC is defined by a ratio of N/C > 2 Absence of co-existing cardiac structural abnormalities Prominent, excessive trabeculations and deep intra-trabecular recesses Recesses supplied by intraventricular blood on color doppler Cardiac MRI has increased the diagnostic accuracy in the diagnosis of LVNC. It has been suggested that a NC/C ratio of > 2.3 in diastole distinguished pathological non-compaction, with sensitivity of 86% and a specificity of 99%, respectively. Although studies have shown an increase specificity with cardiac MRI, caution is needed as it may overestimate the presence of LVNC. Late gadolinium enhancement which suggests myocardial fibrosis or scar has been shown to have some prognostic value in LVNC patients. (5) Management for LVNC is multifaceted. As above,LVNC has a variety of presentations and prevailing manifestations will differ among patients. Therefore, the diagnostic and management approach much be personalized for a given patient...
24 Nov 2022244. Cardiovascular Genomics: Intro to Cardiovascular Genetics in Electrophysiology with Dr. James Daubert00:53:55
The field of Cardiovascular Genomics has advanced tremendously over the past two decades, having a significant clinical impact and changing the perception of the role and scope of genetic testing in several cardiovascular domains.  To kickstart the Cardiovascular Genomics series, CardioNerds Dr. Sara Coles (FIT at Duke University), Dr. Colin Blumenthal (CardioNerds Academy faculty and FIT at UPenn), and Dr. Karla Asturias (CardioNerds Academy fellow and medicine resident at Pennsylvania Hospital) have a great discussion with Dr. James Daubert, a clinical electrophysiologist at Duke University, with a particular interest in inherited arrhythmia syndromes and sports cardiology. In this episode, we review basic concepts of cardiovascular genomics and genetics in electrophysiology while discussing when to (and when not to!) test our patients and their families and how to approach those results. Audio editing by CardioNerds academy intern, Pace Wetstein. This episode was developed in collaboration with the American Society of Preventive Cardiology and is supported with unrestricted educational funds from Illumina, Inc. All CardioNerds content is planned, produced, and reviewed solely by CardioNerds. This CardioNerds Cardiovascular Genomics series is a multi-institutional collaboration made possible by contributions of stellar fellow leads and expert faculty from several programs. Pearls • Notes • References CardioNerds Cardiovascular Genomics PageCardioNerds Episode PageCardioNerds AcademyCardionerds Healy Honor Roll CardioNerds Journal ClubSubscribe to The Heartbeat Newsletter!Check out CardioNerds SWAG!Become a CardioNerds Patron! Pearls and Quotes - Genetics in Electrophysiology The first step is identifying the right phenotype! Getting the right phenotype is crucial, as genetic testing done in a patient without a clear phenotype (or an incorrect one) would lead to significant anxiety, unnecessary tests and interventions, and potentially misleading and dangerous conclusions for patients and their families. Genetic testing typically should be reserved for patients with a confirmed or suspected diagnosis of an inherited disease or for individuals with a previously diagnosed pathogenic variant in a first-degree relative.1 Discuss with your patient! Genetic counseling is essential and recommended for all patients before and after genetic testing. It should include a thorough discussion of risks, benefits, and possible outcomes, including variants of uncertain significance.2 Cardiovascular genetics is a dynamic and rapidly evolving field. New information can cause a variant of uncertain significance to be reclassified as a pathogenic or likely pathogenic variant or to be downgraded to benign or likely benign as variant databases expand. Another possibility is that new research might identify novel genes for a particular disease, which could warrant retesting, particularly for phenotype-positive and genotype-negative patients.1 Brugada syndrome is an inherited arrhythmogenic disorder characterized by ST-segment elevation in the right precordial leads and malignant ventricular arrhythmias, with occasional conduction disease and atrial arrhythmias. It is diagnosed in patients with ST-segment elevation ≥ 2 mm in ≥ 1 lead among the right precordial leads, with a type I morphology (J-point elevation with slowly descending or concave ST segment elevation merging into a negative T wave), shown in the image below. This pattern can be observed spontaneously or after provocative drug testing (e.g., procainamide). Pathogenic genetic variants in SCN5A that result in loss of function of the cardiac sodium channel are identified in approximately 20% of cases.3,4 Image adapted from Batchvarov VN. The Brugada Syndrome – Diagnosis, Clinical Implications and Risk Stratification. Eur Cardiol Rev. 2014;9(2):82. doi:10.15420/ECR.2014.9.2.82 Measure the QT interval yourself!
28 Nov 2022245. ACHD: Ventricular Septal Defects with Dr. Keri Shafer01:03:50
Congenital heart disease is the most common birth defect, affecting 1 in 100 babies. Amongst these ventricular septal defects are very common with the majority of patients living into adulthood. In this episode we will be reviewing key features of VSDs including embryologic origin, anatomy, physiology, hemodynamic consequences, clinical presentation and management of VSDs. Dr. Tommy Das (CardioNerds Academy Program Director and FIT at Cleveland Clinic), Dr. Agnes Koczo (CardioNerds ACHD Series Co-Chair and FIT at UPMC), and Dr. Anu Dodeja (Associate Director for ACHD at Connecticut Children’s) discuss VSDs with expert faculty Dr. Keri Shafer. Dr. Shafer is an adult congenital heart disease specialist at Boston Children’s Hospital, and an assistant professor of pediatrics within Harvard Medical School. She is a medical educator and was an invited speaker for the inaugural CardioNerds Sanjay V Desai Lecture, on the topic of growth mindset. Script and notes were developed by Dr. Anu Dodeja. Audio editing by CardioNerds Academy Intern, Shivani Reddy. The CardioNerds Adult Congenital Heart Disease (ACHD) series provides a comprehensive curriculum to dive deep into the labyrinthine world of congenital heart disease with the aim of empowering every CardioNerd to help improve the lives of people living with congenital heart disease. This series is multi-institutional collaborative project made possible by contributions of stellar fellow leads and expert faculty from several programs, led by series co-chairs, Dr. Josh Saef, Dr. Agnes Koczo, and Dr. Dan Clark. The CardioNerds Adult Congenital Heart Disease Series is developed in collaboration with the Adult Congenital Heart Association, The CHiP Network, and Heart University. See more Disclosures: None Pearls • Notes • References • Guest Profiles • Production Team CardioNerds Adult Congenital Heart Disease PageCardioNerds Episode PageCardioNerds AcademyCardionerds Healy Honor Roll CardioNerds Journal ClubSubscribe to The Heartbeat Newsletter!Check out CardioNerds SWAG!Become a CardioNerds Patron! Pearls - Ventricular Septal Defects Most common VSDs: Perimembranous VSD The shunt volume in a VSD is determined largely by the size of the defect and the pulmonary vascular resistance. VSDs cause left to right shunt. The long-term effects are left sided chamber dilation, as is the case with PDAs (post-tricuspid shunts) VSDs can be associated with acquired RVOTO, double chamber right ventricle, LVOTO/sub aortic membrane formation, and aortic regurgitation from aortic valve prolapse. Eisenmenger syndrome results from long-term left-to-right shunt, usually at higher shunt volumes. The resulting elevated pulmonary artery pressure is irreversible and leads to a reversal in the ventricular level shunt, desaturation, cyanosis, and secondary erythrocytosis. Endocarditis prophylaxis is not indicated for simple VSD. It is required for 6 months post VSD closure, in patients post VSD closure with a residual shunt and in Eisenmenger patients with R—>L shunt and cyanosis. Show notes - Ventricular Septal Defects Notes (developed by Dr. Anu Dodeja): What are types OF VSD? (Please note that there are several nomenclatures) Perimembranous VSDMost common type of VSD - 80% of VSDsOccurs in the membranous septum and can be associated with inlet or outlet extensionLocated near the tricuspid and aortic valves, often time can be closed off by tissue from the septal leaflet of the tricuspid valve and associated with abnormalities in the septal leaflet of the tricuspid valve secondary to damage from the left to right shuntCan be associated with acquired RVOTO, double chamber right ventricle, LVOTO/sub aortic membrane formation On TTE, the parasternal short axis view at the base demonstrates this type of VSD at the 10-12 o’clock position. Muscular VSDSecond most common VSD - 15-20% of VSDsCompletely surrounded by muscle,
30 Nov 2022246. Cardiovascular Genomics: Genetic Counseling & Family Screening in Arrhythmogenic Cardiomyopathies with Dr. Allison Hays and Dr. Cindy James00:45:25
The CardioNerds Cardiovascular Genomics Series continues! In this episode Dr. Dan Ambinder (CardioNerds Cofounder and Interventional Cardiologist), Dr. Anjali Wagle (FIT Ambassador at Johns Hopkins) and Dr. James Sampognaro (medicine resident at Johns Hopkins Osler Medicine Residency) learn from Dr. Allison Hays (Associate Professor of Medicine, Division of Cardiology, Johns Hopkins CMR researcher and Medical Director of Echocardiography) and Dr. Cindy James (Associate Professor of Medicine and certified genetic counselor at Johns Hopkins with research focusing on cardiovascular genetic counseling and arrhythmogenic cardiomyopathies). They discuss arrhythmogenic RV cardiomyopathy as the context to learn about genetic counseling and family screening.  Episode script and notes were developed by Dr. Anjali Wagle. Audio editing by CardioNerds Academy Intern, student doctor Chelsea Amo Tweneboah. This episode was developed in collaboration with the American Society of Preventive Cardiology and is supported with unrestricted educational funds from Illumina, Inc. All CardioNerds content is planned, produced, and reviewed solely by CardioNerds. This CardioNerds Cardiovascular Genomics series is a multi-institutional collaboration made possible by contributions of stellar fellow leads and expert faculty from several programs. Check out this REVIEW describing the “Multimodality Imaging in Arrhythmogenic Right Ventricular Cardiomyopathy” by Nitin Malik, Allison Hays, and colleagues.   For related episodes, please enjoy these case-based discussions:  Ep 56. Case Report: Arrhythmogenic Desmoplakin Cardiomyopathy – Northwestern University  Ep 74. Case Report: Arrhythmogenic Right Ventricular Cardiomyopathy (ARVC) – Summa Health  Pearls • Notes • References CardioNerds Cardiovascular Genomics PageCardioNerds Episode PageCardioNerds AcademyCardionerds Healy Honor Roll CardioNerds Journal ClubSubscribe to The Heartbeat Newsletter!Check out CardioNerds SWAG!Become a CardioNerds Patron! Show notes - Genetic Counseling & Family Screening in Arrhythmogenic Cardiomyopathies Notes (developed by Dr. Anjali Wagle)   What is the underlying pathophysiology of arrhythmogenic RV cardiomyopathy (ARVC)?  Fibrofatty replacement cardiac myocytes  Associated with genetically mediated disruption of desmosomal proteins   This leads to thinning and weakness of the heart that can lead to aneurysms and progressive dilatation and failure of the right ventricle (RV)  How is ARVC diagnosed?  2010 taskforce criteria (Marcus et al, 2010):    RV structural abnormalities including findings seen on echocardiogram, MRI, and RV angiography  Pathological criteria  Repolarization abnormalities   Depolarization/conduction abnormalities   Ventricular arrhythmias   Genetics and/or family history   How does ARVC present?   Young, healthy individual will have symptoms of arrhythmias (syncope, pre-syncope, SCD) or heart failure  Family screening   What are the inheritance and genetic factors of ARVC?  Autosomal dominant pattern  Low penetrance and variable expressivity   Half of patients who are index cases will be found to have a mutation in the desmosomal gene.   What are the most common mutations associated with ARVC?  Most commonly the genes involved are plakophilin-2 (PKP-2) and desmoplakin.   For PKP-2 the most common mutations are truncating mutations.   In patients who have inherited two truncating mutations, this will result in neonatal lethality.   Is there a difference in the genetic factors of left and right arrhythmogenic cardiomyopathy?   ACM is disproportionally a right dominated cardiomyopathy. Left dominated cardiomyopathy has a different genetic profile.   Pathogenic variants in desmoplakin disproportionally cause biventricular forms of ACM or left dominated forms.  
05 Dec 2022247. CCC: Biventricular Failure and the Use of VA-ECMO with Dr. Ann Gage00:54:56
In this episode, we discuss the utility of veno-arterial extra-corporeal membrane oxygenation (VA-ECMO) for the temporary management of biventricular failure and cardiogenic shock requiring full cardiopulmonary support. Here, we define the types of ECMO and describe the unique physiology of this mechanical circulatory support platform, as well as review the potential complications and management strategies. Most notably, we highlight indications for and contraindications to the use of VA-ECMO and review the importance of patient selection.  Lastly, we discuss de-escalation and de-cannulation strategies for patients on VA-ECMO as a bridge to recovery. Join Dr. Amit Goyal (CardioNerds Cofounder and FIT at Cleveland Clinic), Dr. Yoav Karpenshif (Series Co-chair and FIT at University of Pennsylvania), and Dr. Megan Burke (Episode FIT Lead and FIT at University of Pennsylvania) as they learn about how to care for some of our sickest patients from Dr. Ann Gage, interventional and critical care cardiologist at Centennial Heart. At the beginning of the episode, enjoy a message from the very first CardioNerds Scholar, Dr. Katie Vaughan (Chief Resident and soon Cardiology Fellow at BIDMC). Episode notes were developed by Dr. Megan Burke. Audio editing by CardioNerds Academy Intern, Hirsh Elhence. The CardioNerds Cardiac Critical Care Series is a multi-institutional collaboration made possible by contributions of stellar fellow leads and expert faculty from several programs, led by series co-chairs, Dr. Mark Belkin, Dr. Eunice Dugan, Dr. Karan Desai, and Dr. Yoav Karpenshif. Pearls • Notes • References • Production Team CardioNerds Cardiac Critical Care PageCardioNerds Episode PageCardioNerds AcademyCardionerds Healy Honor Roll CardioNerds Journal ClubSubscribe to The Heartbeat Newsletter!Check out CardioNerds SWAG!Become a CardioNerds Patron! Pearls and Quotes - Biventricular Failure and the Use of VA-ECMO Veno-arterial extracorporeal membrane oxygenation (VA-ECMO) is a form of temporary mechanical circulatory support that can do the work of both the heart and lungs. The ECMO circuit is a narcissist, i.e. cannulas are named in reference to the circuit and not the patient (“inflow” vs “outflow”). The decision to utilize ECMO should be made by a multidisciplinary shock team and patient selection is KEY! ECMO physiology rule #1: VA-ECMO increases LV afterload Patients on VA-ECMO should be monitored with a PA catheter and an arterial line in the right arm Show notes - Biventricular Failure and the Use of VA-ECMO Notes drafted by Dr. Megan Burke. 1. What is ECMO and what are the different types? Extracorporeal membrane oxygenation (ECMO) is a temporary form of mechanical life support that comes in two flavors: veno-arterial, or “VA” and veno-venous, or “VV.”  VV-ECMO supports extracorporeal gas exchange in the setting of acute respiratory failure VA-ECMO provides full circulatory support in addition to gas exchange, doing the work of both the heart and lungs.  2. What are the components and “anatomy” of the VA-ECMO circuit? The circuit is made up of the following major components: Venous (inflow) cannula Centrifugal Pump Oxygenator (also responsible for CO2 removal) Arterial (outflow) cannula The cannulas are named in reference to the ECMO circuit, not the patient. Dr. Gage suggests that we think of the ECMO circuit (and mechanical circulatory support in general) as narcissistic, i.e. flow is always in reference to the device. Gas exchange happens in the oxygenator. In the oxygenator blood flows through thin filaments that allow for diffusion of oxygen and carbon dioxide. Gas flows in the opposite direction of blood flow to maximize diffusion through the countercurrent effect. Oxygenation is determined by rate of blood flow through the oxygenator and FiO2 delivered. Carbon dioxide removal is determined by rate of countercurrent gas flow,
08 Dec 2022248. Cardiovascular Genomics: Frontiers in Clinical Genetics in Cardiovascular Prevention with Dr. Pradeep Natarajan00:59:50
As the burden of cardiovascular disease increases in the United States, the importance of enhanced screening tools, early risk prediction, and prevention strategies grows. Novel risk scoring methods, including polygenic risk scores (PRS), may help identify patients that benefit from early intervention and risk modification. In this episode, we discuss how a PRS is calculated, how to incorporate a PRS into clinical practice, and current barriers to the equitable implementation of risk scores. In terms of frontiers in clinical genetics we also discuss the burgeoning field of pharmacogenetics and how pharmacogenetics may be used to identify responders and non-responders to certain therapies. Join CardioNerds Dr. Jessie Holtzman (CardioNerds Academy Chief and Chief Resident and soon FIT at UCSF), Dr. Alaa Diab (CardioNerds Academy Fellow and Medicine Resident at GBMC), and student doctor Hirsh Elhence (CardioNerds Academy Intern and medical student at USC Keck School of Medicine) as they discuss frontiers in clinical genetics with Dr. Pradeep Natarajan (Director of Preventive Cardiology, Massachusetts General Hospital). Audio editing by CardioNerds Academy Intern, student doctor Akiva Rosenzveig. This episode was developed in collaboration with the American Society of Preventive Cardiology and is supported with unrestricted educational funds from Illumina, Inc. All CardioNerds content is planned, produced, and reviewed solely by CardioNerds. This CardioNerds Cardiovascular Genomics series is a multi-institutional collaboration made possible by contributions of stellar fellow leads and expert faculty from several programs. Pearls • Notes • References CardioNerds Cardiovascular Genomics PageCardioNerds Episode PageCardioNerds AcademyCardionerds Healy Honor Roll CardioNerds Journal ClubSubscribe to The Heartbeat Newsletter!Check out CardioNerds SWAG!Become a CardioNerds Patron! Pearls - Frontiers in Clinical Genetics in Cardiovascular Prevention For common diseases like coronary artery disease, rare mutations may confer a several-fold increased risk of disease – for instance, in familial hypercholesterolemia, a single rare mutation may confer as much as a three-fold increase in risk of coronary artery disease. However, for most common diseases, the overall cumulative impact of several common genetic variants may be greater than that of a monogenetic trait. Family history is a particularly coarse predictor of CV risk, highlighting the need for polygenic risk scores. In particular, younger patients with borderline cardiovascular risk may benefit from the use of a polygenic risk score in the determination of their overall cardiovascular risk profile. A polygenic risk score (PRS) is a weighted sum of several risk-conferring alleles. The weight assigned to an allele is determined by the strength of the association between the allele and CV disease, as determined by genome-wide association studies (GWAS). The data used for genome-wide associated studies in cardiovascular disease have historically included populations primarily of European ancestry. However, more data is being collected from diverse patient cohorts to increase the external validity and broader applicability of such studies. Pharmacogenetic polygenic risk scores may be used to predict drug efficacy and toxicity, as well as to identify biologically plausible drug targets for clinical trial design. Show notes - Frontiers in Clinical Genetics in Cardiovascular Prevention What is a polygenic risk score (PRS)? Monogenic conditions are those in which a variant in a single gene causes a pathological phenotype. For example, familial hypercholesterolemia is often the result of a mutated allele in the LDL receptor gene. In contrast, polygenic risk suggests that there are variants in multiple genes that all confer risk independently, each with a small individual effect size. By aggregating many variants,
18 Dec 2022249. CardioNerds Rounds: Challenging Cases – HFpEF Diagnosis and Management with Dr. Jane Wilcox00:39:22
It’s another session of CardioNerds Rounds! In these rounds, Dr. Loie Farina (Advanced Heart Failure and Transplant Fellow at Northwestern University) joins Dr. Jane Wilcox (Chief of the Section of Heart Failure Treatment and Recovery at Northwestern University) to discuss the nuances of HFpEF diagnosis and management. Dr. Wilcox is also the Associate Director of the T1 Center for Cardiovascular Therapeutics in the Bluhm Cardiovascular Institute and Director of the Myocardial Recovery Clinic at Northwestern University. Dr. Wilcox is a prolific researcher, clinician, and thought leader in Heart Failure and we are honored to have her on CardioNerds Rounds! Notes were drafted by Dr. Karan Desai. Audio editing by CardioNerds Academy Intern, student doctor Akiva Rosenzveig. This episode is supported with unrestricted funding from Zoll LifeVest. A special thank you to Mitzy Applegate and Ivan Chevere for their production skills that help make CardioNerds Rounds such an amazing success. All CardioNerds content is planned, produced, and reviewed solely by CardioNerds. Case details are altered to protect patient health information. CardioNerds Rounds is co-chaired by Dr. Karan Desai and Dr. Natalie Stokes.  Speaker disclosures: None Challenging Cases - Atrial Fibrillation with Dr. Hugh Calkins CardioNerds Rounds PageCardioNerds Episode PageCardioNerds AcademyCardionerds Healy Honor Roll CardioNerds Journal ClubSubscribe to The Heartbeat Newsletter!Check out CardioNerds SWAG!Become a CardioNerds Patron! Show notes - HFpEF Diagnosis and Management Case #1 Synopsis: A woman in her 80s with a history of HFpEF presented with worsening dyspnea on exertion over the course of a year but significantly worsening over the past two months. Her other history includes prior breast cancer with chemotherapy and radiation therapy, permanent atrial fibrillation with AV node ablation and CRT-P, and CKD Stage III. She presented for an outpatient RHC with exercise to further characterize her HFpEF. Her echo showed normal LV size, no LVH, LVEF of 50%, decreased RV systolic function, severe left atrial enlargement, significantly elevated E/e’ and mild MR. Right heart catheterization showed moderately elevated bi-ventricular filling pressures at rest but with passive leg raise and Stage 1 exercise the wedge pressure rose significantly. We were asked to comment on management. Case #1 Takeaways Amongst the things that were discussed were the role of specific therapies in symptomatic patients with HFpEF. In patients with HFpEF and documented congestion, they will require diuretic therapy for symptomatic relief. But in addition to diuretic therapy, we discussed starting HFpEF-specific therapies. Amongst, those specific therapies mineralocorticoid receptor antagonist (MRA) and sodium-glucose co-transporter 2 (SGLT2) inhibitor. In multiple trials that have included patients with HFPEF, SGLT2i have reduced the risk of hospitalization. This includes the EMPEROR-PRESERVED Trial (see the CardioNerds Journal Club discussion on the trial) in which nearly 6000 patients with NYHA Class II-IV symptoms, EF > 40% and elevated NT-proBNP with a prior HF hospitalization within the past 12 months were randomized to Empagliflozin or placebo. The primary outcome – death from CV causes or hospitalization for Heart Failure – was significantly lower in the SGLT2i arm (13.8% vs 17.1%, 95% CI 0.69-0.90, P <0.001). In regards to MRA, an important trial was the TOPCAT trial which randomized patients with symptomatic HF and LVEF > 45% to receive either spironolactone or placebo. The primary endpoint (death from CV cause, aborted cardiac arrest, or hospitalization for HF) was not statistically different between treatment arms. Of note, however, there were concerns for regional differences which is outlined well in this NEJM Evidence piece. Case #2 Synopsis: A woman in her 70s with history of hypertension, obesity,
26 Dec 2022250. ACHD: Partial Anomalous Pulmonary Venous Return (PAPVR) with Dr. Ian Harris00:43:28
Partial anomalous pulmonary venous return refers to anomalies in which one or more (but not all) of the pulmonary veins connects to a location other than the left atrium. This causes left to right shunting which may have hemodynamic and therefore clinical significance, warranting repair in some patients. Join CardioNerds to learn about partial anomalous pulmonary venous return! Dr. Dan Ambinder (CardioNerds co-founder), Dr. Josh Saef (ACHD FIT at the University of Pennsylvania and ACHD Series co-chair), and Dr. Tripti Gupta (ACHD FIT at Vanderbilt University and episode lead) learn from Dr. Ian Harris (Director of the Adult Congenital Heart Disease program at University of California, San Francisco). Audio editing by CardioNerds Academy Intern, student doctor Shivani Reddy. The CardioNerds Adult Congenital Heart Disease (ACHD) series provides a comprehensive curriculum to dive deep into the labyrinthine world of congenital heart disease with the aim of empowering every CardioNerd to help improve the lives of people living with congenital heart disease. This series is multi-institutional collaborative project made possible by contributions of stellar fellow leads and expert faculty from several programs, led by series co-chairs, Dr. Josh Saef, Dr. Agnes Koczo, and Dr. Dan Clark. The CardioNerds Adult Congenital Heart Disease Series is developed in collaboration with the Adult Congenital Heart Association, The CHiP Network, and Heart University. See more Disclosures: None Pearls • Notes • References • Guest Profiles • Production Team CardioNerds Adult Congenital Heart Disease PageCardioNerds Episode PageCardioNerds AcademyCardionerds Healy Honor Roll CardioNerds Journal ClubSubscribe to The Heartbeat Newsletter!Check out CardioNerds SWAG!Become a CardioNerds Patron! Pearls - Partial Anomalous Pulmonary Venous Return (PAPVR) What is partial anomalous pulmonary venous return (PAPVR)? PAPVR refers to anomalies in which one or more (but not all) of the pulmonary veins connects to a location other than the left atrium. Often, this means one or more pulmonary veins empty into the right atrium or a systemic vein such as the superior vena cava or inferior vena cava. Physiologically, this produces a left-to-right shunt, allowing for already-oxygenated blood to recirculate into the lungs and result in excessive pulmonary blood flow.  What are the clinical features of PAPVR?Diagnosis is usually incidental on a cross sectional imaging such as CTA or CMR.The most common associated lesion is an atrial-level defect.It is unusual for a single anomalous pulmonary venous connection of only 1 pulmonary lobe to result in significant shunting. Patients with a significant degree of left to right shunting may have right heart dilatation or symptoms of dyspnea on exertion. When are some strategies for managing patients with PAPVR?A surgical correction is recommended for patients with PAPVR when functional capacity is impaired and RV enlargement is present, there is a net left-to-right shunt sufficiently large to cause physiological sequelae (aka: ratio of pulmonary flow (Qp) to systemic flow (Qs) is > 1.5:1), PA systolic pressure is less than 50% systemic pressure and pulmonary venous resistance is less than one third of systemic venous resistance.Surgical repair involves intracaval baffling of the left atrium (Warden procedure) or direct reimplantation of the anomalous pulmonary vein into the left atrium.Pregnancy is well tolerated in patients with repaired PAPVR. In patients with unrepaired lesion who may have right sided heart dilatation and/or pulmonary hypertension, preconception evaluation and counseling should address how pregnancy may affect mother’s and fetus’s health. Antibiotic prophylaxis for infective endocarditis is typically not needed unless patients are less than 6 months from recent surgery, have residual defect at the patch margin or prior history of infective endocarditi...
01 Jan 2023251. CCC: Palliative Care and Shared Decision-Making in the CICU with Dr. Larry Allen00:58:50
This episode is focused on Palliative Care and Shared Decision-Making in the CICU. In this episode, we learn about how the principles of palliative care and shared decision-making apply to our patients across the spectrum of cardiovascular care, especially in the cardiac intensive care unit. We discuss pivotal trials of specialty palliative care and decision aids in cardiology and how they might inform our practice to enhance patient quality of life and improve goal-concordant care. Finally, we discuss practical tips and communication strategies for how to engage patients about end-of-life decisions and topics that can be utilized from outpatient to inpatient to critical care settings. “We need to help patients hope for the best and plan for the worst as time goes on.” Dr. Larry Allen Series co-chairs Dr. Eunice Dugan and Dr. Karan Desai, along with CardioNerds Co-founder Amit Goyal are joined by FIT lead, Dr. Sarah Chuzi. Dr. Chuzi is a Chicagoan and completed her internal medicine residency, cardiology fellowship, AHFTC fellowship and is now Assistant Professor at Northwestern University. Our episode expert is a true national leader in shared decision-making and palliative care in heart failure – Dr. Larry Allen, Medical Director of Advanced Heart Failure and the Co-Director of the Colorado Program for Patient-Centered Decisions at the University of Colorado School of Medicine. Audio editing by CardioNerds Academy Intern, Dr. Christian Faaborg-Andersen. The CardioNerds Cardiac Critical Care Series is a multi-institutional collaboration made possible by contributions of stellar fellow leads and expert faculty from several programs, led by series co-chairs, Dr. Mark Belkin, Dr. Eunice Dugan, Dr. Karan Desai, and Dr. Yoav Karpenshif. Pearls • Notes • References • Production Team CardioNerds Cardiac Critical Care PageCardioNerds Episode PageCardioNerds AcademyCardionerds Healy Honor Roll CardioNerds Journal ClubSubscribe to The Heartbeat Newsletter!Check out CardioNerds SWAG!Become a CardioNerds Patron! Pearls and Quotes - Palliative Care and Shared Decision-Making in the CICU 1. “Much of what we do in cardiology is thinking about how to make people feel better (not just improving cardiac function or length of life). So, on a day-to-day basis we are really providing primary palliative care.” – Dr. Larry Allen 2. “Risk models in cardiology can only be so accurate… While risk models can give us some grounding, we also need to embrace the concept of uncertainty, and help patients understand that there are a variety of things that might happen to them, suggest some things they might plan for, and continue to iteratively come back to the patient and reevaluate what their options are.” – Dr. Larry Allen 3. “Our goal is to help people live happy, healthy, full lives. But, everyone dies. So understanding that death is a part of life and understanding how to help them make those transitions is critical” – Dr. Larry Allen 4. “Having good deaths is a part of good healthcare. We can’t ignore that. We can’t fight against it. We should embrace it. And we have the opportunity to do that.” – Dr. Larry Allen 5. We should still keep in mind the concept of medical futility and determining what options are reasonable for patients. Part of shared decision-making includes discussing what interventions would not be feasible or helpful with patients and families Show notes - Palliative Care and Shared Decision-Making in the CICU Notes drafted by Dr. Sarah Chuzi. 1. How are the basic principles of palliative care relevant to cardiology, and can you define the key concepts of shared decision-making, primary palliative care, specialty (or secondary) palliative care, and hospice care? Throughout medicine, we confront the concepts of symptom control, difficult medical decision-making, and end-of-life. These are the principles of palliative care and they apply very easily across the spec...
06 Jan 2023252. Cardio-Oncology: Cardiac Amyloidosis with Dr. Omar Siddiqi00:56:23
The importance of recognition and diagnosis of cardiac amyloidosis is at an all-time high due to its high prevalence and improved therapeutic strategies. Here we discuss what CardioNerds need to know about the manifestations, diagnosis, and management of transthyretin (ATTR) and light chain (AL) cardiac amyloidosis. Join Dr. Dan Ambinder (CardioNerds Cofounder), Dr. Dinu-Valentin Balanescu (Series Cochair, Chief Resident at Beaumont Health, and soon FIT at Mayo Clinic), and Dr. Dan Davies (Episode FIT Lead and FIT at Mayo Clinic) as they discuss cardiac amyloidosis with Dr. Omar Siddiqi, cardiologist at the Boston University Amyloidosis Center and program director for the general cardiovascular fellowship program at Boston University, a CardioNerds Healy Honor Roll Program. Episode notes were drafted by Dr. Dan Davies. Audio editing by CardioNerds Academy Intern, student doctor Chelsea Amo Tweneboah. Access the CardioNerds Cardiac Amyloidosis Series for a deep dive into this important topic. This episode is supported by a grant from Pfizer Inc. This CardioNerds Cardio-Oncology series is a multi-institutional collaboration made possible by contributions of stellar fellow leads and expert faculty from several programs, led by series co-chairs, Dr. Giselle Suero Abreu, Dr. Dinu Balanescu, and Dr. Teodora Donisan.  Pearls • Notes • References • Production Team CardioNerds Cardio-Oncology PageCardioNerds Episode PageCardioNerds AcademyCardionerds Healy Honor Roll CardioNerds Journal ClubSubscribe to The Heartbeat Newsletter!Check out CardioNerds SWAG!Become a CardioNerds Patron! Pearls and Quotes Cardiac amyloidosis is no longer considered a rare disease, especially transthyretin amyloidosis in older male patients with HFpEF and aortic stenosis. Echocardiogram is the “gate keeper” of cardiac imaging and provides initial evidence of amyloid infiltration, while cardiac MRI can help refine the presence of an infiltrative cardiomyopathy versus other causes of increased wall thickness. The most clinically important types of amyloid heart disease are transthyretin (ATTR) and light chain (AL) amyloidosis. The workup to differentiate these disorders includes a gammopathy panel to screen for the presence of potentially amyloidogenic light chains (serum and urine electrophoresis WITH immunofixation and serum free light chains), and cardiac scintigraphy with Technetium-99m-labeled bone-seeking tracers (PYP, DPD, etc.) to identify cardiac aTTR infiltration if the gammopathy panel is unrevealing. There is still a role for endomyocardial biopsy in the diagnosis of cardiac amyloidosis! All patients in whom there is concern for cardiac amyloidosis and gammopathy panel indicates the presence of monoclonal light chains should have a biopsy to obtain a tissue diagnosis of likely AL amyloidosis. Alternatively, an endocardial biopsy may prove valuable in patients who have confusing phenotypic features between amyloid types, such as a patient with abnormal monoclonal protein and positive PYP imaging. Be suspicious of heart failure patients that do not tolerate typical medications that lower heart rate. In the restrictive cardiomyopathy of cardiac amyloidosis, patients are reliant on higher heart rates to compensate for the inability to augment stroke volume. Be suspicious of amyloidosis in patients with recurrent left atrial thrombi despite anticoagulation. Show notes CardioNerds Cardiac Amyloid, updated 1.20.21 1. What is cardiac amyloidosis and how common is it? Cardiac amyloidosis is adisorder caused by misfolding of proteins into insoluble forms which are deposited into extracellular spaces of the heart, commonly causing a stiff and thick heart with progressive diastolic dysfunction with restrictive hemodynamics and ensuing heart failure. The two most common types of amyloid protein that affect the heart are transthyretin (ATTR) and light chain (AL).
08 Jan 2023253. The 2022 AHA/ACC/HFSA Guideline for the Management of Heart Failure – Perspectives from Writing Committee Chair Dr. Paul Heidenreich00:38:47
Join CardioNerds (Dr. Mark Belkin and Dr. Natalie Tapaskar) as they discuss the 2022 AHA/ACC/HFSA Guideline for the Management of Heart Failure with Writing Committee Chair Dr. Paul Heidenreich. They discuss how one gets involved with a guideline writing committee, the nuts and bolts of the guideline writing process, pitfalls and utility of the term “GDMT,” background behind inclusion of “Value Statements,” potential omissions from the document, clinical uptake of recommendations, and anticipated changes for the next iteration. Audio editing by CardioNerds academy intern, Pace Wetstein. This discussion is a prelude to the CardioNerds Decipher The Guidelines Series designed to enhance understanding and uptake of the 2022 AHA/ACC/HFSA Guideline for the Management of Heart Failure. We will be using high-impact, board-style, clinical vignette-based questions to highlight core concepts relevant to your practice. We will do so by releasing several short bite-sized Pods with one question per episode. Note that the cases used are hypothetical and created solely to illustrate core concepts. This series was developed by the CardioNerds and created in collaboration with the American Heart Association and the Heart Failure Society of America. It was created by 30 trainees spanning college through advanced fellowship under the leadership of CardioNerds Cofounders Dr. Amit Goyal and Dr. Dan Ambinder, with mentorship from Dr. Anu Lala, Dr. Robert Mentz, and Dr. Nancy Sweitzer. We thank Dr. Judy Bezanson and Dr. Elliott Antman for tremendous guidance. Decipher the Guidelines: 2022 Heart Failure Guidelines PageCardioNerds Episode PageCardioNerds AcademyCardionerds Healy Honor Roll CardioNerds Journal ClubSubscribe to The Heartbeat Newsletter!Check out CardioNerds SWAG!Become a CardioNerds Patron!
09 Jan 2023254. Guidelines: 2022 AHA/ACC/HFSA Guideline for the Management of Heart Failure – Question #1 with Dr. Biykem Bozkurt00:12:44
The following question refers to Section 2.1 of the 2022 AHA/ACC/HFSA Guideline for the Management of Heart Failure. The question is asked by Keck School of Medicine USC medical student & CardioNerds Intern Hirsh Elhence, answered first by Mount Sinai Hospital cardiology fellow and CardioNerds FIT Trialist Dr. Jason Feinman, and then by expert faculty Dr. Biykem Bozkurt. Dr. Bozkurt is the Mary and Gordon Cain Chair, Professor of Medicine, Director of the Winters Center for Heart Failure Research, and an advanced heart failure and transplant cardiologist at Baylor College of Medicine in Houston, TX. She is former President of HFSA, former senior associate editor for Circulation, current Editor-In-Chief of JACC Heart Failure. Dr. Bozkurt was the Vice Chair of the writing committee for the 2022 Heart Failure Guidelines. The Decipher the Guidelines: 2022 AHA / ACC / HFSA Guideline for The Management of Heart Failure series was developed by the CardioNerds and created in collaboration with the American Heart Association and the Heart Failure Society of America. It was created by 30 trainees spanning college through advanced fellowship under the leadership of CardioNerds Cofounders Dr. Amit Goyal and Dr. Dan Ambinder, with mentorship from Dr. Anu Lala, Dr. Robert Mentz, and Dr. Nancy Sweitzer. We thank Dr. Judy Bezanson and Dr. Elliott Antman for tremendous guidance. Enjoy this Circulation 2022 Paths to Discovery article to learn about the CardioNerds story, mission, and values. Question #1 A 23-year-old man presents to his primary care physician for an annual visit. His father was diagnosed with idiopathic cardiomyopathy at 40 years of age. His blood pressure in clinic is 146/90 mmHg. He is a personal trainer and exercises daily, including both weightlifting and cardio. He denies any anabolic steroid use. He is an active tobacco smoker, approximately ½ pack per day. Review of systems is negative for symptoms. What stage of heart failure most appropriately describes his current status? A Stage A B Stage B C Stage C D Stage D E None of the above Answer #1 The correct answer is A – Stage A of heart failure. Overall, the ACC/AHA stages of HF were designed to emphasize the development and progression of disease. More advanced stages and progression are associated with reduced survival. Stage A HF is where patients are “at risk for HF”, but without current or previous symptoms or signs of HF, and without structural/functional heart disease or abnormal biomarkers. At-risk patients include those with hypertension, cardiovascular disease, diabetes, obesity, exposure to cardiotoxic agents, genetic variant for cardiomyopathy, or family history of cardiomyopathy. Stage B HF is the “pre-heart failure” stage where patients are without current or previous symptoms or signs of HF but do have at least one of the following: Structural heart disease (i.e., reduced left or right ventricular systolic function, ventricular hypertrophy, chamber enlargement, wall motion abnormalities, and valvular heart disease) Evidence of increased filling pressures Risk factors and increased natriuretic peptide levels or persistently elevated cardiac troponin in the absence of an alternate diagnosis Stage C HF indicates symptomatic heart failure where patients have current or previous symptoms or signs of HF. Stage D HF indicates advanced heart failure with marked HF symptoms that interfere with daily life and with recurrent hospitalizations despite attempts to optimize guideline-directed medical therapy. Therapeutic interventions in each stage aim to modify risk factors (Stage A), treat risk and structural heart disease to prevent HF (stage B), and reduce symptoms, morbidity, and mortality (stages C and D). Given this patient’s family and social histories, along with the clinical finding of elevated blood pressure, he is best classified as having Stage A, or at risk for HF.
10 Jan 2023255. Guidelines: 2022 AHA/ACC/HFSA Guideline for the Management of Heart Failure – Question #2 with Dr. Mark Drazner00:08:08
The following question refers to Section 6.1 of the 2022 AHA/ACC/HFSA Guideline for the Management of Heart Failure. The question is asked by Keck School of Medicine USC medical student & CardioNerds Intern Hirsh Elhence, answered first by Mount Sinai Hospital cardiology fellow and CardioNerds FIT Trialist Dr. Jason Feinman, and then by expert faculty Dr. Mark Drazner. Dr. Drazner is an advanced heart failure and transplant cardiologist, Professor of Medicine, and Clinical Chief of Cardiology at UT Southwestern. He is the President of the Heart Failure Society of America. The Decipher the Guidelines: 2022 AHA / ACC / HFSA Guideline for The Management of Heart Failure series was developed by the CardioNerds and created in collaboration with the American Heart Association and the Heart Failure Society of America. It was created by 30 trainees spanning college through advanced fellowship under the leadership of CardioNerds Cofounders Dr. Amit Goyal and Dr. Dan Ambinder, with mentorship from Dr. Anu Lala, Dr. Robert Mentz, and Dr. Nancy Sweitzer. We thank Dr. Judy Bezanson and Dr. Elliott Antman for tremendous guidance. Enjoy this Circulation 2022 Paths to Discovery article to learn about the CardioNerds story, mission, and values. Question #2 A 67-year-old man with a past medical history of type 2 diabetes mellitus, hypertension, and active tobacco smoking presents to the emergency room with substernal chest pain for the past 5 hours. An electrocardiogram reveals ST segment elevations in the anterior precordial leads and he is transferred emergently to the catheterization laboratory. Coronary angiography reveals 100% occlusion of the proximal left anterior descending artery, and he is successfully treated with a drug eluting stent resulting in TIMI 3 coronary flow. Following his procedure, a transthoracic echocardiogram is performed which reveals a left ventricular ejection fraction of 35% with a hypokinetic anterior wall. Which of the following medications would be the best choice to prevent the incidence of heart failure and reduce mortality? A Lisinopril B Diltiazem C Carvedilol D Sacubitril-valsartan E Both A and C Answer #2 The correct answer is E – both lisinopril and carvedilol are appropriate to reduce the incidence of heart failure and mortality. Evidence-based beta-blockers and ACE inhibitors both have Class 1 recommendations in patients with a recent myocardial infarction and left ventricular ejection fraction ≤ 40% to reduce the incidence of heart failure and to reduce mortality. Multiple randomized controlled trials have investigated both medications in the post myocardial infarction setting and demonstrated improved ventricular remodeling as well as benefits for mortality and development of incident heart failure. At this time, there is not sufficient evidence to recommend ARNi over ACEi for patients with reduced LVEF following acute MI. The PARADISE-MI trial randomized a total of 5,661 patients with myocardial infarction complicated by a reduced LVEF, pulmonary congestion, or both to receive either sacubitril-valsartan (97-103mg twice daily) or ramipril (5mg twice daily). After a median follow up time of 22 months, there was no statistically significant difference in the primary outcome of cardiovascular death or incident heart failure. At this time, ARNi have not been included in the guidelines for this specific population. Diltiazem is a non-dihydropyridine calcium channel blocker, a family of drugs with negative inotropic effects and which may be harmful in patients with depressed LVEF (Class 3: Harm, LOE C-LD). Main Takeaway:  For patients with recent myocardial infarction and reduced left ventricular function both beta blockers and ACEi have Class 1 recommendations to reduce the incidence of heart failure and decrease mortality. Guideline Location: Section 6.1
13 Jan 2023256. Guidelines: 2022 AHA/ACC/HFSA Guideline for the Management of Heart Failure – Question #3 with Dr. Shelley Zieroth00:08:11
The following question refers to Section 3.1 of the 2022 AHA/ACC/HFSA Guideline for the Management of Heart Failure. The question is asked by Texas Tech University medical student and CardioNerds Academy Intern Dr. Adriana Mares, answered first by Rochester General Hospital cardiology fellow and Director of CardioNerds Journal Club Dr. Devesh Rai, and then by expert faculty Dr. Shelley Zieroth. Dr. Zieroth is an advanced heart failure and transplant cardiologist, Head of the Medical Heart Failure Program, the Winnipeg Regional Health Authority Cardiac Sciences Program, and an Associate Professor in the Section of Cardiology at the University of Manitoba. Dr. Zieroth is a past president of the Canadian Heart Failure Society. She is a steering committee member for PARAGLIE-HF and a PI Mentor for the CardioNerds Clinical Trials Program. The Decipher the Guidelines: 2022 AHA / ACC / HFSA Guideline for The Management of Heart Failure series was developed by the CardioNerds and created in collaboration with the American Heart Association and the Heart Failure Society of America. It was created by 30 trainees spanning college through advanced fellowship under the leadership of CardioNerds Cofounders Dr. Amit Goyal and Dr. Dan Ambinder, with mentorship from Dr. Anu Lala, Dr. Robert Mentz, and Dr. Nancy Sweitzer. We thank Dr. Judy Bezanson and Dr. Elliott Antman for tremendous guidance. Enjoy this Circulation 2022 Paths to Discovery article to learn about the CardioNerds story, mission, and values. Question #3 Which of the following is/are true about heart failure epidemiology? A Although the absolute number of patients with HF has partly grown, the incidence of HF has decreased B Non-Hispanic Black patients have the highest death rate per capita resulting from HF C In patients with established HF, non-Hispanic Black patients have a higher HF hospitalization rate compared with non-Hispanic White patients D In patients with established HF, non-Hispanic Black patients have a lower death rate compared with non-Hispanic White patients E All of the above Answer #3 Explanation    The correct answer is “E – all of the above.”   Although the absolute number of patients with HF has partly grown as a result of the increasing number of older adults, the incidence of HF has decreased. There is decreasing incidence of HFrEF and increasing incidence of HFpEF. The health and socioeconomic burden of HF is growing. Beginning in 2012, the age-adjusted death-rate per capita for HF increased for the first time in the US. HF hospitalizations have also been increasing in the US. In 2017, there were 1.2 million HF hospitalizations in the US among 924,000 patients with HF, a 26% increase compared with 2013.   Non-Hispanic Black patients have the highest death rate per capita. A report examining the US population found the age-adjusted mortality rate for HF to be 92 per 100,000 individuals for non-Hispanic Black patients, 87 per 100,000 for non-Hispanic White patients, and 53 per 100,000 for Hispanic patients.   Among patients with established HF, non-Hispanic Black patients experienced a higher rate of HF hospitalization and a lower rate of death than non-Hispanic White patients with HF.Hispanic patients with HF have been found to have similar or higher HF hospitalization rates and similar or lower mortality rates compared with non-Hispanic White patients. Asian/Pacific Islander patients with HF have had a similar rate of hospitalization as non-Hispanic White patients but a lower death rate.   These racial and ethnic disparities warrant studies and health policy changes to address health inequity. Main Takeaway Racial and ethnic disparities in death resulting from HF persist, with non-Hispanic Black patients having the highest death rate per capita, and a higher rate of HF hospitalization. Further clinical studies and health policy changes are needed to address ...
15 Jan 2023257. Guidelines: 2022 AHA/ACC/HFSA Guideline for the Management of Heart Failure – Question #4 with Dr. Eldrin Lewis00:23:42
The following question refers to Section 4.1 of the 2022 AHA/ACC/HFSA Guideline for the Management of Heart Failure. The question is asked by Texas Tech University medical student and CardioNerds Academy Intern Dr. Adriana Mares, answered first by Baylor University cardiology fellow and CardioNerds FIT Trialist Dr. Shiva Patlolla, and then by expert faculty Dr. Eldrin Lewis. Dr. Lewis is an Advanced Heart Failure and Transplant Cardiologist, Professor of Medicine and Chief of the Division of Cardiovascular Medicine at Stanford University. The Decipher the Guidelines: 2022 AHA / ACC / HFSA Guideline for The Management of Heart Failure series was developed by the CardioNerds and created in collaboration with the American Heart Association and the Heart Failure Society of America. It was created by 30 trainees spanning college through advanced fellowship under the leadership of CardioNerds Cofounders Dr. Amit Goyal and Dr. Dan Ambinder, with mentorship from Dr. Anu Lala, Dr. Robert Mentz, and Dr. Nancy Sweitzer. We thank Dr. Judy Bezanson and Dr. Elliott Antman for tremendous guidance. Enjoy this Circulation 2022 Paths to Discovery article to learn about the CardioNerds story, mission, and values. Question #4 Mr. Stevens is a 55-year-old man who presents with progressively worsening dyspnea on exertion for the past 2 weeks. He has associated paroxysmal nocturnal dyspnea, intermittent exertional chest pressure, and bilateral lower extremity edema. Otherwise, Mr. Stevens does not have any medical history and does not take any medications.  Which of the following will be helpful for diagnosis at this time? A  Detailed history and physical examination B  Chest x-ray C  Blood workup including CBC, CMP, NT proBNP D  12-lead ECG E All of the above   Answer #4   The correct answer is E – All of the above.  Mr. Stevens presents with signs and symptoms of volume overload concerning for new onset heart failure. The history and physical exam remain the cornerstone in the assessment of patients with HF. Not only is the H&P valuable for identifying the presence of heart failure but also may provide hints about the degree of congestion, underlying etiology, and alternative diagnoses. As such H&P earns a Class 1 indication for a variety of reasons in patients with heart failure: 1.       Vital signs and evidence of clinical congestion should be assessed at each encounter to guide overall management, including adjustment of diuretics and other medications (Class 1, LOE B-NR) 2.       Clinical factors indicating the presence of advanced HF should be sought via the history and physical examination (Class 1, LOE B-NR) 3.       A 3-generation family history should be obtained or updated when assessing the cause of the cardiomyopathy to identify possible inherited disease (Class 1, LOE B-NR) 4.       A thorough history and physical examination should direct diagnostic strategies to uncover specific causes that may warrant disease-specific management (Class 1, LOE B-NR) 5.       A thorough history and physical examination should be obtained and performed to identify cardiac and noncardiac disorders, lifestyle and behavioral factors, and social determinants of health that might cause or accelerate the development or progression of HF (Class 1, LOE C-EO) Building on the H&P, laboratory evaluation provides important information about comorbidities, suitability for and adverse effects of treatments, potential causes or confounders of HF, severity and prognosis of HF, and more. As such, for patients who are diagnosed with HF, laboratory evaluation should include complete blood count, urinalysis, serum electrolytes, blood urea nitrogen, serum creatinine, glucose, lipid profile, liver function tests, iron studies, and thyroid-stimulating hormone to optimize management (Class 1, LOE C-EO). In addition, the specific cause of HF should be explored using additional laboratory testi...
19 Jan 20207. Cardiac Amyloid part 1: case discussion01:03:10
A new case of congestive heart failure due to amyloid cardiomyopathy is presented by Yuxuan Wang and discussed by Carine, Heather, Dan and Amit. Guest oncology star: Jackie Zimmerman. Flutter moment by Mark Heslin. Show notes and images: https://www.cardionerds.com/cardiac-amyloid/
20 Jan 2023258. Guidelines: 2022 AHA/ACC/HFSA Guideline for the Management of Heart Failure – Question #5 with Dr. Clyde Yancy00:12:02
The following question refers to Section 7.1 of the 2022 AHA/ACC/HFSA Guideline for the Management of Heart Failure. The question is asked by New York Medical College medical student and CardioNerds Intern Akiva Rosenzveig, answered first by Cornell cardiology fellow and CardioNerds Ambassador Dr. Jaya Kanduri, and then by expert faculty Dr. Clyde Yancy.Dr. Yancy is Professor of Medicine and Medical Social Sciences, Chief of Cardiology, and Vice Dean for Diversity and Inclusion at Northwestern University, and a member of the AHA/ACC/HFSA Heart Failure Guideline Writing Committee.The Decipher the Guidelines: 2022 AHA / ACC / HFSA Guideline for The Management of Heart Failure series was developed by the CardioNerds and created in collaboration with the American Heart Association and the Heart Failure Society of America. It was created by 30 trainees spanning college through advanced fellowship under the leadership of CardioNerds Cofounders Dr. Amit Goyal and Dr. Dan Ambinder, with mentorship from Dr. Anu Lala, Dr. Robert Mentz, and Dr. Nancy Sweitzer. We thank Dr. Judy Bezanson and Dr. Elliott Antman for tremendous guidance.Enjoy this Circulation 2022 Paths to Discovery article to learn about the CardioNerds story, mission, and values. Question #5 Ms. L is a 65-year-old woman with nonischemic cardiomyopathy with a left ventricular ejection fraction (LVEF) of 35%, hypertension, and type 2 diabetes mellitus. She has been admitted to the hospital with decompensated heart failure (HF) twice in the last six months and admits that she struggles to understand how to take her medications and adjust her sodium intake to prevent this.  Which of the following interventions has the potential to decrease the risk of rehospitalization and/or improve mortality? A Access to a multidisciplinary team (physicians, nurses, pharmacists, social workers, care managers, etc) to assist with management of her HF   B Engaging in a mobile app aimed at improving HF self-care   C Vaccination against respiratory illnesses   D A & C   Answer #5   The correct answer is D – both A (access to a multidisciplinary team) and C (vaccination against respiratory illness).   Choice A is correct. Multidisciplinary teams involving physicians, nurses, pharmacists, social workers, care managers, dieticians, and others, have been shown in multiple RCTs, metanalyses, and Cochrane reviews to both reduce hospital admissions and all-cause mortality. As such, it is a class I recommendation (LOE A) that patients with HF should receive care from multidisciplinary teams to facilitate the implementation of GDMT, address potential barriers to self-care, reduce the risk of subsequent rehospitalization for HF, and improve survival. Choice B is incorrect.  Self-care in HF comprises treatment adherence and health maintenance behaviors. Patients with HF should learn to take medications as prescribed, restrict sodium intake, stay physically active, and get vaccinations. They also should understand how to monitor for signs and symptoms of worsening HF, and what to do in response to symptoms when they occur. Interventions focused on improving the self-care of HF patients significantly reduce hospitalizations and all-cause mortality as well as improve quality of life. Therefore, patients with HF should receive specific education and support to facilitate HF self-care in a multidisciplinary manner (Class I, LOE B-R). However, the method of delivery and education matters. Reinforcement with structured telephone support has been shown to be effective. In contrast the efficacy of mobile health-delivered educational interventions in improve self-care in patients with HF remains uncertain. Choice C is correct. In patients with HF, vaccinating against respiratory illnesses is reasonable to reduce mortality (Class 2a, LOE B-NR). For example, administration of the influenza vaccine in HF patients has been shown to reduce...
20 Jan 2023259. Guidelines: 2022 AHA/ACC/HFSA Guideline for the Management of Heart Failure – Question #6 with Dr. Randall Starling00:09:31
The following question refers to Section 7.4 of the 2022 AHA/ACC/HFSA Guideline for the Management of Heart Failure. The question is asked by New York Medical College medical student and CardioNerds Intern Akiva Rosenzveig, answered first by Cornell cardiology fellow and CardioNerds Ambassador Dr. Jaya Kanduri, and then by expert faculty Dr. Randall Starling.Dr. Starling is Professor of Medicine and an advanced heart failure and transplant cardiologist at the Cleveland Clinic where he was formerly the Section Head of Heart Failure, Vice Chairman of Cardiovascular Medicine, and member of the Cleveland Clinic Board of Governors. Dr. Starling is also Past President of the Heart Failure Society of America in 2018-2019. Dr. Staring was among the earliest CardioNerds faculty guests and has since been a valuable source of mentorship and inspiration. Dr. Starling’s sponsorship and support was instrumental in the origins of the CardioNerds Clinical Trials Program.The Decipher the Guidelines: 2022 AHA / ACC / HFSA Guideline for The Management of Heart Failure series was developed by the CardioNerds and created in collaboration with the American Heart Association and the Heart Failure Society of America. It was created by 30 trainees spanning college through advanced fellowship under the leadership of CardioNerds Cofounders Dr. Amit Goyal and Dr. Dan Ambinder, with mentorship from Dr. Anu Lala, Dr. Robert Mentz, and Dr. Nancy Sweitzer. We thank Dr. Judy Bezanson and Dr. Elliott Antman for tremendous guidance.Enjoy this Circulation 2022 Paths to Discovery article to learn about the CardioNerds story, mission, and values. Question #6 Mr. D is a 50-year-old man who presented two months ago with palpations and new onset bilateral lower extremity swelling. Review of systems was negative for prior syncope. On transthoracic echocardiogram, he had an LVEF of 40% with moderate RV dilation and dysfunction. EKG showed inverted T-waves and low-amplitude signals just after the QRS in leads V1-V3. Ambulatory monitor revealed several episodes non-sustained ventricular tachycardia with a LBBB morphology. He was initiated on GDMT and underwent genetic testing that revealed 2 desmosomal gene variants associated with arrhythmogenic right ventricular cardiomyopathy (ARVC). Is the following statement true or false? “ICD implantation is inappropriate at this time because his LVEF is >35%” True   False   Answer #6 Explanation This statement is False. ICD implantation is reasonable to decrease sudden death in patients with genetic arrhythmogenic cardiomyopathy with high-risk features of sudden death who have an LVEF ≤45% (Class 2a, LOE B-NR). While the HF guidelines do not define high-risk features of sudden death, the 2019 HRS expert consensus statement on evaluation, risk stratification, and management of arrhythmogenic cardiomyopathy identify major and minor risk factors for ventricular arrhythmias as follows: Major criteria: NSVT, inducibility of VT during EPS, LVEF ≤ 49%. Minor criteria: male sex, >1000 premature ventricular contractions (PVCs)/24 hours, RV dysfunction, proband status, 2 or more desmosomal variants. According to the HRS statement, high risk is defined as having either three major, two major and two minor, or one major and four minor risk factors for a class 2a recommendation for primary prevention ICD in this population (LOE B-NR). Based on these criteria, our patient has 2 major risk factors (NSVT & LVEF ≤ 49%), and 3 minor risk factors (male sex, RV dysfunction, and 2 desmosomal variants) for ventricular arrhythmias. Therefore, ICD implantation for primary prevention of sudden cardiac death is reasonable. Decisions around ICD implantation for primary prevention remain challenging and depend on estimated risk for SCD, co-morbidities, and patient preferences, and so should be guided by shared decision making weighing the possible benefits against the risks,
22 Jan 2023260. Case Report: Cardioembolic Stroke from an Unusual Valve Pathology from The University of Alabama at Birmingham00:35:42
CardioNerds Cofounder Dr. Amit Goyal join Dr. Usman Hasnie and Dr. Will Morgan from University of Alabama at Birmingham for a hike up Red Mountain. They discuss the following case: A 75-year-old woman with prior mitral valve ring annuloplasty presented with subacute, intermittent, self-limiting neurologic deficits. Brain MRI revealed multiple subacute embolic events consistent with cardioembolic phenomena. Transesophageal echochardiogram discovered a mobile mass on the mitral valve as the likely cause for cardioembolic stroke. She was taken for surgical repair of the mitral valve. Tissue biopsy confirmed that the mass was an IgG4-related pseudotumor. Expert commentary is provided by Dr. Neal Miller (Assistant Professor of Cardiology, University of Alabama at Birmingham). Audio editing by CardioNerds Academy Intern, student doctor Adriana Mares Check out this published case report here: IgG4-Related Disease Masquerading as Culture-Negative Endocarditis! CardioNerds Case Reports PageCardioNerds Episode PageCardioNerds AcademyCardionerds Healy Honor Roll CardioNerds Journal ClubSubscribe to The Heartbeat Newsletter!Check out CardioNerds SWAG!Become a CardioNerds Patron! Pearls - Cardioembolic Stroke due to an IgG4-related pseudotumor Surgical indications for endocarditis include severe heart failure, valvular dysfunction with severe hemodynamic compromise, prosthetic valve infection, invasion beyond the valve leaflets, recurrent systemic embolization, large mobile vegetations, or persistent sepsis (in infective endocarditis) despite adequate antibiotic therapy. IgG4 related disease is rare, and likely underrecognized due to the lack of reliable biomarkers. Biopsy and histologic confirmation are imperative to clinch the diagnosis. Cardiac manifestations of IgG4-related disease are rare but are often related to aortopathies. Valvular disease is extremely rare as a manifestation of the disease. Treatment of IgG4 related disease includes steroids as the first line treatment. IgG4 related disease requires a multi-disciplinary approach to both diagnose and treat. Show Notes - Cardioembolic Stroke due to an IgG4-related pseudotumor Notes were drafted by Dr. Hasnie and Dr. Morgan IgG4-related disease has a very diverse presentation including mimicry of infection, malignancy and other autoimmune conditions. It is a fibroinflammatory condition that results in deposition of IgG4 positive plasma cells. It has been described in multiple organ systems including the pancreas, kidneys, lungs and salivary glands.  Cardiac manifestations are extremely rare and valvular disease even more so. There are thirteen cases of IgG4 related valvular disease, and of these only two had mitral valve involvement such as this case. The most commonly reported cardiovascular manifestations are related to aortopathies.  This disease remains poorly understood at this point. There are no true biomarkers that can be used to risk stratify the diagnosis for clinicians. Biopsy is imperative to the diagnosis. Even serum IgG4 levels are normal in 30% of cases despite meeting histologic criteria on biopsy making the diagnosis incredibly difficult to make.  While guidelines have not been developed to guide treatment of IgG4-related disease, steroids are considered the first line treatment option for patients. Often times dosing is 2-4 weeks with a prolonged taper. When looking for glucocorticoid sparing agents, azathioprine, mycophenolate mofetil, and methotrexate are considered alternatives.  References - Cardioembolic Stroke due to an IgG4-related pseudotumor 1. Kamisawa T, Funata N, Hayashi Y, et al. A new clinicopathological entity of IgG4- related autoimmune disease. J Gastroenterol 2003;38:982-4.  2. Deshpande V, Zen Y, Chan JK, et al. Consensus statement on the pathology of IgG4-related disease. Mod Pathol. 2012;25(9):1181-1192. doi:10.1038/modpathol.2012.72  3. Dahlgren M,
31 Jan 2023261. Cardio-Oncology: Cancer Therapy-Related Cardiac Dysfunction (CTRCD) – The Oncologist Perspective with Dr. Susan Dent00:45:40
Dr. Filip Ionescu (hematology-oncology fellow at Moffitt Cancer Center in Tampa, FL), Dr. Teodora Donisan (cardiology fellow at the Mayo Clinic in Rochester, MN and CardioNerds House Thomas chief), Dr. Sarah Waliany (internal medicine chief resident at Stanford University in Palo Alto, CA), Dr. Dinu Balanescu (internal medicine chief resident at Beaumont Hospital in Royal Oak, MI) and Dr. Amit Goyal (structural interventional cardiology fellow at the Cleveland Clinic, in Cleveland, OH and CardioNerds Co-Founder), discuss the cardiotoxicities of common cancer treatments with Dr. Susan Dent, a medical oncologist and one of the founders of the field of Cardio-Oncology. Using the recently published ESC Guidelines on cardio-oncology, they cover cardiovascular risk stratification in oncology patients, pretreatment testing, as well as prevention and management of established cardiotoxicity resulting from anthracyclines, trastuzumab, and fluoropyrimidines. They touch on the unique aspects of cardio-oncology encountered in patients with breast cancer, rectal cancer, and lung cancer, who are frequently the recipients of multiple cardiotoxic treatments. Audio editing by CardioNerds Academy Intern, student doctor Chelsea Amo Tweneboah. Access the CardioNerds Cardiac Amyloidosis Series for a deep dive into this important topic. This episode is supported by a grant from Pfizer Inc. This CardioNerds Cardio-Oncology series is a multi-institutional collaboration made possible by contributions of stellar fellow leads and expert faculty from several programs, led by series co-chairs, Dr. Giselle Suero Abreu, Dr. Dinu Balanescu, and Dr. Teodora Donisan.  Pearls • Notes • References • Production Team CardioNerds Cardio-Oncology PageCardioNerds Episode PageCardioNerds AcademyCardionerds Healy Honor Roll CardioNerds Journal ClubSubscribe to The Heartbeat Newsletter!Check out CardioNerds SWAG!Become a CardioNerds Patron! Pearls and Quotes - Cancer Therapy-Related Cardiac Dysfunction (CTRCD) – The Oncologist Perspective with Dr. Susan Dent Formal cardiovascular risk stratification must be performed prior to initiating a potentially cardiotoxic anticancer treatment regimen. Considering both drug toxicity and patient-related factors (e.g., age, smoking, hypertension etc) is important.  Anthracyclines affect the cardiomyocyte in complex ways which lead to a largely irreversible cardiomyopathy. All patients should have a pretreatment echocardiogram and ECG.  Trastuzumab cardiotoxicity, by contrast, is more like stunning the myocardium, which manifests as a reversible decrease in left ventricular ejection fraction which generally normalizes upon discontinuation of the drug.  The treatment of chemotherapy-induced cardiomyopathy should involve interdisciplinary discussions and shared decision making with the patient. Beyond guideline-directed medical therapy of heart failure with reduced ejection fraction, management can include temporarily holding or permanently discontinuing the offending agent.  Fluoropyrimidine-associated cardiotoxicity manifests as cardiac ischemia from coronary vasospasm. A 5FU infusion is essentially a stress test as it tends to unmask clinically silent atherosclerosis.  Show notes What is the basic pretreatment assessment of any oncology patient who is to receive a potentially cardiotoxic regimen?  Awareness and management of the cardiovascular toxicity of oncology treatments are of paramount importance to be able to deliver treatment safely and to achieve maximal efficacy guided by an expert multidisciplinary team. Thanks to Dr. Dent and her colleagues’ work, this year we have seen the publication of the first Cardio-Oncology guideline (1). Perhaps the most important recommendation is that cancer patients about to start a cardiotoxic regimen should undergo formal cardiovascular risk stratification by considering both the adverse profile of the planned treatment...
06 Feb 2023262. CCC: Management of Cardiorenal Syndrome in the CICU with Dr. Nayan Arora and Dr. Elliott Miller00:41:18
The Cardiorenal Syndrome is commonly encountered, and frequently misunderstood. Join the CardioNerds team as we discuss the complex interplay between the heart and kidneys with Dr. Elliott Miller (Assistant Professor of Medicine at Yale University School of Medicine and Associate Medical Director of the Cardiac Intensive Care Unit of Yale New Haven Hospital), and Dr. Nayan Arora (Clinical Assistant Professor of Medicine and Nephrologist at the University of Washington Medical Center). We are hosted by FIT lead Dr. Matthew Delfiner (Cardiology Fellow at Temple University), Cardiac Critical Care Series Co-Chairs Dr. Mark Belkin (AHFTC faculty at University of Chicago) and Dr. Karan Desai (Cardiologist at Johns Hopkins Hospital), and CardioNerds Co-Found Dr. Dan Ambinder. In this episode we discuss the definition and pathophysiology of the cardiorenal syndrome, explore strategies for initial diuresis and diuretic resistance, and management of the common heart failure medications in this setting. Show notes were developed by Dr. Matthew Delfiner. Audio editing by CardioNerds Academy Intern, student doctor Akiva Rosenzveig. The CardioNerds Cardiac Critical Care Series is a multi-institutional collaboration made possible by contributions of stellar fellow leads and expert faculty from several programs, led by series co-chairs, Dr. Mark Belkin, Dr. Eunice Dugan, Dr. Karan Desai, and Dr. Yoav Karpenshif. Pearls • Notes • References • Production Team CardioNerds Cardiac Critical Care PageCardioNerds Episode PageCardioNerds AcademyCardionerds Healy Honor Roll CardioNerds Journal ClubSubscribe to The Heartbeat Newsletter!Check out CardioNerds SWAG!Become a CardioNerds Patron! Pearls and Quotes - Management of Cardiorenal Syndrome in the CICU Cardiorenal syndrome (CRS) represents a range of clinical entities in which there is both heart and kidney dysfunction, and can be driven by one, or both, of the organs. CRS is caused by reduced renal perfusion, elevated renal congestion, or a combination of the two. Treatment therefore focuses on increasing perfusion, by optimizing cardiac output and mean arterial pressure, and reducing congestion through diuresis. Patients should be monitored for an adequate response to the initial diuretic dose within 2 hours of administration. If the response is inadequate, the loop diuretic dose should be doubled. Diuretic resistance can be managed via sequential nephron blockade, most commonly with thiazide diuretics, but also with amiloride, high-dose spironolactone, or acetazolamide, as these target different regions of the nephron. In cases of refractory diuretic resistance, hypertonic saline can be considered with the help of an experienced clinician. Continuation or cessation of renin-angiotensin-aldosterone system (RAAS) inhibitors in the setting of CRS should be made on a case-by-case basis. Show notes - Management of Cardiorenal Syndrome in the CICU 1. Cardiorenal syndrome (CRS) is a collection of signs/symptoms that indicate injury to both the heart and kidneys. Organ dysfunction in one can drive dysfunction in the other. Cardiorenal syndrome can be categorized as: Type 1 - Acute heart failure causing acute kidney injury Type 2 - Chronic heart failure causing chronic kidney injury Type 3 - Acute kidney injury causing acute heart failure Type 4 - Chronic kidney injury causing chronic heart failure Type 5 - Co-development of heart and kidney injury by another systemic process. These categories can be helpful for education, discussion, and research purposes, but they do not usually enter clinical practice on a regular basis since different categories of cardiorenal syndrome are not necessarily treated differently. 2. CRS is caused by either reduced renal perfusion, elevated renal congestion, or a combination of the two. When dealing with CRS, note that: CRS can be caused by poor kidney perfusion,
10 Feb 2023263. ACHD: Patent Ductus Arteriosus & Eisenmenger Syndrome with Dr. Candice Silversides00:27:24
Join CardioNerds to learn about patent ducts arteriosus and Eisenmenger syndrome! Dr. Dan Ambinder (CardioNerds co-founder), ACHD series co-chair Dr. Dan Clark,  Dr. Tony Pastor (ACHD fellow, Harvard Medical School), and Dr. Kate Wilcox, Medicine/Pediatrics Resident, Medical College of Wisconsin join Dr. Candice Silversides (Editor-in-chief #JACCAdvances) for this terrific discussion. Notes were drafted by Dr. Kate Wilcox. .Audio editing by CardioNerds Academy Intern, Dr. Maryam Barkhordarian. The CardioNerds Adult Congenital Heart Disease (ACHD) series provides a comprehensive curriculum to dive deep into the labyrinthine world of congenital heart disease with the aim of empowering every CardioNerd to help improve the lives of people living with congenital heart disease. This series is multi-institutional collaborative project made possible by contributions of stellar fellow leads and expert faculty from several programs, led by series co-chairs, Dr. Josh Saef, Dr. Agnes Koczo, and Dr. Dan Clark. The CardioNerds Adult Congenital Heart Disease Series is developed in collaboration with the Adult Congenital Heart Association, The CHiP Network, and Heart University. See more Disclosures: None CardioNerds Adult Congenital Heart Disease PageCardioNerds Episode PageCardioNerds AcademyCardionerds Healy Honor Roll CardioNerds Journal ClubSubscribe to The Heartbeat Newsletter!Check out CardioNerds SWAG!Become a CardioNerds Patron! Pearls - Patent Ductus Arteriosus & Eisenmenger Syndrome The ductus arteriosus, which is formed from the distal portion of the left sixth arch, is key to fetal circulation because it allows blood to bypass the high resistance pulmonary circuit present in utero. After birth there is a significant drop in pulmonary vascular resistance (PVR) which generally leads to functional ductal closure within 48 hours (permanent seal takes 2-3 weeks to form). Risk factors for having a PDA include birth before 37 weeks of gestation, trisomy 21, and congenital rubella. A PDA results in a left to right shunt (qP:qS >1) which over time overloads the left side of the heart and causes pulmonary vascular remodeling. The extra workload on the left side of the heart causes left atrial (can cause atrial arrhythmias) and left ventricular dilation. If left untreated you can eventually have shunt reversal due to very high PVR (Eisenmenger physiology). There are some treatment options at this point (pulmonary vasodilators, etc) but it’s definitely better to close the PDA before this point. One interesting physical exam finding that can stem from shunt reversal in a hemodynamically significant PDA is differential cyanosis (upper body or pre-ductal saturations will be higher than lower body/post-ductal saturations). You can also see clubbing in the toes but not the hands for the same reason. Meet Our Collaborators! Adult Congenital Heart AssociationFounded in 1998, the Adult Congenital Heart Association is an organization begun by and dedicated to supporting individuals and families living with congenital heart disease and advancing the care and treatment available to our community. Our mission is to empower the congenital heart disease community by advancing access to resources and specialized care that improve patient-centered outcomes. Visit their website (https://www.achaheart.org/) for information on their patient advocacy efforts, educational material, and membership for patients and providers CHiP Network The CHiP network is a non-profit organization aiming to connect congenital heart professionals around the world. Visit their website (thechipnetwork.org) and become a member to access free high-quality educational material, upcoming news and events, and the fantastic monthly Journal Watch, keeping you up to date with congenital scientific releases. Visit their website (https://thechipnetwork.org/) for more information.
12 Feb 2023264. CCC:  Approach to Renal Replacement Therapy in the CICU with Dr. Joel Topf00:47:42
Renal replacement therapy (RRT) is routinely utilized in the CICU. Series co-chairs Dr. Eunice Dugan and Dr Karan Desai along with CardioNerds Co-founder Dr. Daniel Ambinder were joined by FIT lead and CardioNerds Ambassador from University of Washington, Dr. Tomio Tran. Our episode expert is world-renowned nephrologist Dr. Joel Topf. Dr. Topf is Medical Director of Research at St. Clair Nephrology, and editor of the Handbook of Critical Care Nephrology. In this episode, we describe a case of cardiogenic shock due to acute myocardial infarction resulting in renal failure, ultimately requiring continuous RRT (CRRT). We discuss the most common causes of AKI within the cardiac ICU, indications for initiating RRT, evidence on the timing of RRT, different modes of RRT, basic management of the RRT circuit, and how to transition patients off of RRT during renal recovery. Episode notes were drafted by Dr. Tomio Tran. Audio editing by CardioNerds Academy Intern, Dr. Maryam Barkhordarian. The CardioNerds Cardiac Critical Care Series is a multi-institutional collaboration made possible by contributions of stellar fellow leads and expert faculty from several programs, led by series co-chairs, Dr. Mark Belkin, Dr. Eunice Dugan, Dr. Karan Desai, and Dr. Yoav Karpenshif. Pearls • Notes • References • Production Team CardioNerds Cardiac Critical Care PageCardioNerds Episode PageCardioNerds AcademyCardionerds Healy Honor Roll CardioNerds Journal ClubSubscribe to The Heartbeat Newsletter!Check out CardioNerds SWAG!Become a CardioNerds Patron! Pearls and Quotes - Approach to Renal Replacement Therapy in the CICU Do not commit “Renalism” - withholding lifesaving treatments from patients with renal impairment due to fear of causing renal injury. Shared decision making is key. In the ICU, most of the time, AKI is caused by ATN due to adverse hemodynamics. Nephrologists can help determine the cause if the patient has an atypical presentation. Late dialysis initiation is non-inferior to early dialysis initiation. Early initiation may lead to higher rates of prolonged time on dialysis. Slow low efficiency daily diafiltration (SLEDD) vs CRRT are equivalent in terms of outcomes and are the preferred methods among patients with hypotension. Intermittent Hemodialysis (iHD) can be used once patients are hemodynamically stable. A “Furosemide Stress Test” can be used to test intact renal function or renal recovery by challenging the nephron to make urine. Show notes - Approach to Renal Replacement Therapy in the CICU What are the risk factors and differential for AKI in the CICU? Start by using the pre-renal vs intrinsic renal vs post-renal framework. Additional considerations in cardiac patients include contrast induced nephropathy, pigment nephropathy, cardiorenal syndrome. Enjoy Episode 262. Management of Cardiorenal Syndrome in the CICU. In the ICU setting, intrinsic renal injury due to ATN is among the most common etiology of AKI. Many risk factors for AKI are not modifiable in the ICU. Optimize renal function by avoiding nephrotoxins, minimizing contrast usage, and keeping the MAP >65-75 mmHg. Contrast nephropathy as an etiology is questionable and may be a marker of a sicker patient population. Avoid “Renalism” - providing substandard care to patients with renal disease due to fear of worsening renal function. Most etiologies are treated with supportive care. What is the approach to timing of renal replacement therapy initiation? Definitions for early vs late vs very late initiation of RRT:Early – Worsening AKI without indications for RRTLate – Worsening AKI with relative indications for RRT Very late – Worsening AKI with strict indications for RRT Late initiation is noninferior in terms of mortality; early initiation is associated with higher rates of prolonged/permanent RRT.1,2,3 Very late initiation associated with worse outcomes.4 In general,
15 Feb 2023265. Case Report: An Unusual Case of Non-ischemic Cardiomyopathy – Cleveland Clinic00:53:33
CardioNerds co-founder Daniel Ambinder joins Cleveland Clinic cardiology fellows, Dr. Essa Hariri, Dr. Anna Scandinaro, and Dr. Beka Bekhdatze, Clinical pharmacist at Cleveland Clinic, Dr. Ashley Kasper, and Dr. Craig Parris from Ohio State University Medical Center for a walk at Edgewater Park in Cleveland, Ohio. Dr. Andrew Higgins (Crtitical Care Cardiology and Advanced HF / Transplant Cardiology at Cleveland Clinic) provides the ECPR for this episode. They discuss the following case involving a rare cause of non-ischemic cardiomyopathy. A young African American male was admitted for cardiogenic shock following an admission a month earlier for treatment resistant psychosis. He was diagnosed with medication-induced non-ischemic cardiomyopathy, which resolved with a remarkable recovery of his systolic function after discontinuation of the culprit medication, Clozapine. Episode notes were drafted by Dr. Essa Hariri. Audio editing by CardioNerds Academy Intern, student doctor Shivani Reddy. Enjoy this case report co-published in US Cardiology Review: Clozapine-induced Cardiomyopathy: A Case Report CardioNerds is collaborating with Radcliffe Cardiology and US Cardiology Review journal (USC) for a ‘call for cases’, with the intention to co-publish high impact cardiovascular case reports, subject to double-blind peer review. Case Reports that are accepted in USC journal and published as the version of record (VOR), will also be indexed in Scopus and the Directory of Open Access Journals (DOAJ). CardioNerds Case Reports PageCardioNerds Episode PageCardioNerds AcademyCardionerds Healy Honor Roll CardioNerds Journal ClubSubscribe to The Heartbeat Newsletter!Check out CardioNerds SWAG!Become a CardioNerds Patron! Pearls - An Unusual Case of Non-ischemic Cardiomyopathy The diagnosis of drug-induced non-ischemic cardiomyopathy is usually one of exclusion. High clinical suspicion is needed to diagnose drug-induced cardiomyopathy. Missing the culprit medication causing drug-induced cardiomyopathy could be detrimental as there is a high probability of reversing a systolic dysfunction after stopping the offending medication. Clozapine is an effective medication for the treatment-resistant schizophrenia and is associated with reduced suicide risk. Clozapine is reported to cause drug-induced cardiomyopathy and is more common with rapid drug titration. Clozapine is more commonly associated with myocarditis. Close monitoring and vigilance are critical to preventing cardiac complications associated with initiating clozapine. The management of clozapine-associated cardiomyopathy includes clozapine cessation and heart failure guideline-directed medical therapy. Show Notes - An Unusual Case of Non-ischemic Cardiomyopathy We treated a case of clozapine-associated cardiomyopathy presenting in cardiogenic shock. Drug-induced cardiomyopathy is a common yet under-recognized etiology of non-ischemic cardiomyopathy. Clozapine is an FDA-approved atypical antipsychotic medication frequently prescribed for treatment-resistant schizophrenia and the only antipsychotic agent that has been proven to significantly reduce suicide among this patient population. However, Clozapine is reported to be associated with several forms of cardiotoxicity, including myocarditis (most common), subclinical clozapine associated cardiotoxicity, and least commonly, drug-induced cardiomyopathy. Clozapine-associated cardiomyopathy should be considered as a differential diagnosis in schizophrenic patients presenting with signs of acute heart failure.  Rapid titration of clozapine is a risk factor for clozapine-associated cardiomyopathy and clozapine-associated myocarditis. To date, there is no evidence or consensus supporting preemptive screening. According to the American Psychiatric Association, whenever clozapine-induced myocarditis or cardiomyopathy is suspected, a cardiology consult is warranted.
15 Feb 2023266. Guidelines: 2022 AHA/ACC/HFSA Guideline for the Management of Heart Failure – Question #7 with Dr. Robert Mentz00:12:20
The following question refers to Section 7.3.1 of the 2022 AHA/ACC/HFSA Guideline for the Management of Heart Failure.  The question is asked by Palisades Medical Center medicine resident & CardioNerds Intern Dr. Maryam Barkhordarian, answered first by MedStar Washington Hospital Center cardiology hospitalist & CardioNerds Academy Graduate Dr. Luis Calderon, and then by expert faculty Dr. Robert Mentz.  Dr. Mentz is associate professor of medicine and section chief for Heart Failure at Duke University, a clinical researcher at the Duke Clinical Research Institute, and editor-in-chief of the Journal of Cardiac Failure. Dr. Mentz is a mentor for the CardioNerds Clinical Trials Network as lead principal investigator for PARAGLIDE-HF and is a series mentor for this very 2022 heart failure Decipher the Guidelines Series. For these reasons and many more, he was awarded the Master CardioNerd Award during ACC22. Welcome Dr. Mentz!  The Decipher the Guidelines: 2022 AHA / ACC / HFSA Guideline for The Management of Heart Failure series was developed by the CardioNerds and created in collaboration with the American Heart Association and the Heart Failure Society of America. It was created by 30 trainees spanning college through advanced fellowship under the leadership of CardioNerds Cofounders Dr. Amit Goyal and Dr. Dan Ambinder, with mentorship from Dr. Anu Lala, Dr. Robert Mentz, and Dr. Nancy Sweitzer. We thank Dr. Judy Bezanson and Dr. Elliott Antman for tremendous guidance. Enjoy this Circulation 2022 Paths to Discovery article to learn about the CardioNerds story, mission, and values. Question #7 Ms. Valarie Sartan is a 55-year-old woman with a history of HFrEF (EF 35%) and well controlled, non-insulin dependent diabetes mellitus who presents to heart failure clinic for routine follow up. She is currently being treated with metoprolol succinate 200mg daily, lisinopril 10mg daily, empagliflozin 10mg daily, and spironolactone 50mg daily. She notes stable dyspnea with moderate exertion, making it difficult to do her yardwork. On exam she is well appearing, and blood pressure is 115/70 mmHg with normal jugular venous pulsations and trace bilateral lower extremity edema. On labs, her potassium is 4.0 mmol/L and creatinine is 0.7 mg/dL with an eGFR > 60 mL/min/1.73m2. Which of the following options would be the most appropriate next step in heart failure therapy?  A  Increase lisinopril to 40mg daily  B  Increase spironolactone to 100mg daily  C  Add sacubitril-valsartan to her regimen  D  Discontinue lisinopril and start sacubitril-valsartan in 36 hours  E  No change  Answer #7 Explanation   The correct answer is D – transitioning from an ACEi to an ARNi is the most appropriate next step in management.   The renin-angiotensin aldosterone system (RAAS) is upregulated in patients with chronic heart failure with reduced ejection fraction (HFrEF). Blockade of the RAAS system with ACE inhibitors (ACEi), angiotensin receptor blockers (ARB), or angiotensin receptor neprilysin inhibitors (ARNi) have proven mortality benefit in these patients.   The PARADIGM-HF trial compared sacubitril-valsartan (an ARNi) with enalapril in symptomatic patients with HFrEF. Patients receiving ARNi incurred a 20% relative risk reduction in the composite primary endpoint of cardiovascular death or heart failure hospitalization. Based on these results, the 2022 heart failure guidelines recommend replacing an ACEi or ARB for an ARNi in patients with chronic symptomatic HFrEF with NYHA class II or III symptoms to further reduce morbidity and mortality (Option D). This is a class I recommendation with level of evidence of B-R and is also of high economic value. Making no changes at this time would be inappropriate (Option E).  While it would be reasonable to increase the dose of lisinopril to 40mg (Option A), this should be pursued only if ARNi therapy is not tolerated.   Mineralocorticoid receptor antagonists (MRAs) have a class I (LOE A...

Enhance your understanding of Cardionerds: A Cardiology Podcast with My Podcast Data

At My Podcast Data, we strive to provide in-depth, data-driven insights into the world of podcasts. Whether you're an avid listener, a podcast creator, or a researcher, the detailed statistics and analyses we offer can help you better understand the performance and trends of Cardionerds: A Cardiology Podcast. From episode frequency and shared links to RSS feed health, our goal is to empower you with the knowledge you need to stay informed and make the most of your podcasting experience. Explore more shows and discover the data that drives the podcast industry.
© My Podcast Data